PDA

توجه ! این یک نسخه آرشیو شده میباشد و در این حالت شما عکسی را مشاهده نمیکنید برای مشاهده کامل متن و عکسها بر روی لینک مقابل کلیک کنید : فيزيك جديد



aysa
06-23-2013, 09:45 AM
سلام خدمت همه ي دوستان عزيز خودم:)
راستش اين تاپيك رو ايجاد كردم تا جايي باشه برا بحث در مورد فيزيك جديد...:7:
روند تاپيك: اول در مورد اين موضوع كه از كجا اومده توسط كي چرا وچطوري :confused:بحث ميكنيم!
دوم اين كه كلا چي ميخواد بگه!
وسوم:مباحث علمي ترش!اول از نسبيت شروع كنين لطفا.
تو اين ميان يه عالمه فرمول عجيب غريب خواهيم ديد كه اون هارو هم تاحد امكان سعي كنين به بيان ساده واگه نشد با تشرح كامل بيان كنين.
دوستان در نظر داشته باشين ما يه تاپيكي به اسم فيزيك _محض هم داريم.:دی
سؤالي بود درخدمتم:)

aysa
06-25-2013, 08:29 PM
دوستان ما اين جا ميخوايم يه بحثي درمورد جالب ترين علم دنيا(البته در نظرخودم:دی )يعني فيزيك جديد(فيزيك نوين يا فيزيك مدرن)صحبت كنيم!
همون طور كه ميدونين(شايدم نميدونين:thumbsup:) تا اوايل قرن 20 فيزيك كلاسيك چنان جاشو بين دانشمندا باز كرده بود كه دانشمندي مثل پلانك برگشته گفته :ما به تمام كشفيات مهم موجود در فيزيك رسيده ايم.
اون زمان حرفش خيلي منطقي بود.چون با توجه به امكاناتشون كه فقط دركي از اجسام دورو برشون داشتن اين حرف حتي كاملا درست هم بود.(لازم به ذكره كه اين روزا فيزيك كلاسيك در محدوده ي خودش تقريبا تمام مشكلاتشو حل كرده)تا اين كه ديدن نه نميشه همه چي رو با اون توصيح داد وپاي فيزيك جديد به ميان كشيده شد وداستان از اين جا شروع ميشه...
ژاكوب برونوفسكي رياضيدان و مورخ لهستاني تو آخرين فصل از كتاب (عروج انسان)اش مينويسه:
"يكي از اهداف ديرينه علم فيزيك اين بوده كه تصويري دقيق از جهان ارائه كند.يكي از بزرگترين كاميابي هاي فيزيك قرن بيستم اين بوده كه ثابت كرده رسيدن به چنين هدفي غير ممكن است."
-----------------------------------------------------------------
پ.ن1:لطفا بقيه اش رو شما بياين و...:قلب
پ.ن2:لطفا سعي كنين مطالب رو تاحد امكان ساده توضيخ بدين:)

Ehsan
06-26-2013, 07:14 PM
فیزیک کلاسیک در سه بخش کلی مکانیک (برای پیش بینی حرکت سیاره ها، پرتابه ها، آونگها و... در کل پیش بینی حرکت ذرات)، الکترومغناطیس (پیش بینی کنشهای الکترومغناطیسی، مدارها، اپتیک و نور، پدیده های درون اتمی و...) و ترمودینامیک (برای پیش بینی کنشهای گرمایی-دمایی سیستم ها، کنشهای آماری کلاسیک و...) خلاصه میشه.فیزیک کلاسیک با کارهای دانشمندانی مثل لاپلاس، ماکسول، لاگرانژ، برنولی‌ها و.... پیشرفتهای قابل توجهی داشت و در سالهای انتهایی قرن نوزده‌ام به اوج خودش رسیده بود و آن قدر محکم و خلل ناپذیر نشون می‌داد که خیلی از دانشمندان و فیزیک دانها فکر می کردند فیزیک به زودی تمام میشه و فیزیک دانها قراره برن غاز بچرونن!

تا این که در همان سالها (اواخر قرن نوزده) کم کم مشکلات ِ عجیبی پیش روی فیزیکدانها نقش بست، دونه دونه آزمایش ها و پدیهد هایی پیدا شدند که فیزیک دانها رو کانهو مجسمه ی ابولهول بهت زده کرد! طوری که اصلا با دانسته های گذشته و مفروضات ِ قبلی که توی ذهن داشتند نمی‌تونستند این پدیده ها رو توجیه کنند.

آزمایش مایکلسون-مورلی یکی از مشهورترین ِ این آزمایشهاست، آزمایشی که نسبیت ِ خاص از دل ِ این آزمایش بیرون اومد.

پدیده ی فوتوالکتریک، مشکلات ِ توجیه ساختمان ِ اتمها و طیف ِ اونها، مشکلات ِ مربوط به توصیف ِ طیف تابش جسم سیاه، اثر کامپتون، آزمایش دو شکاف الکترون و.... بقیه ی آزمایشهایی بودند که کم کم مثل ِ تیرهای زهرآگین بر پیکر فلکزده ی فیزیک کلاسیک فرود اومدند.

در سال 1900 ماکس پلانک با ارائه ی مبحث ِ کوانتای نور برای توجیه ِ طیف ِ جسم ِ سیاه اولین ضربه شمشیر رو بر پیکر فیزیک کلاسیک وارد کرد و بعدش اینشیتن با ارائه ی سه مقاله در 1905 که یکیش راجع به نسبیت و توضیح ِ نتیجه ی آزمایش مایکلسون مورلی با مفاهیم کاملا جدید بود و اون یکی کوانتای نور و توضیح ِ اثر ِ فوتوالکتریک با این مبحث ِ تازه بود زد و نصف‌بیشتر ِ فیزیک کلاسیک رو به تنهایی به فنا داد!

و نهایتا در سال 1927 با ارائه شدن ِ مکانیک ِ کوانتمی، فیزیک کلاسیک به طول کامل دچار فنارفتگی شد و فیزیک نوین به جای اون نشست :دی

جا داره به تاریخ ِ هر کدوم از اینها مفصلا بپردازیم اما با توجه به قرار های قبلی اول از تاریخچه ی نسبیت خاص و آزمایش مایکلسون-مورلی شروع می‌کنیم :)
ــــــــــــــ
شروع کنید دیگه :دی

aysa
06-27-2013, 09:42 AM
خب انگار كسي قصد گفتن چيزي رو نداره.پس خودم شروع ميكنم:)
خب.يكي از جنجالي ترين سال ها واسه انيشتين سال1905بود.انيشتين در پي موضوعاتي از قبيل ثابت پلانك كه توسال1900مطرح كرده بود افتاد.به دنبال مقاله هايي معروفش يه عالمه موضوعات عجيب وغريب از فضا اومده(درس مثل مغز خودش)مثل جريان فوتوالكتريك(كه اون زمان ارايه اش كرد)به علم فيزيك بازور وارد كرد.در اين ميان بود كه نظريه ي نسبيت خاص توهمون سال درمورد اندازه گيري اومد به وسط.واين جا بود كه ميدان جنگ برا كلاسيك و نوين به طور عملي تشكيل شد.:fight:
برا ادامه دادن اين مطلب همون طور كه گفته شد اول بايد يه آزمايشي به اسم آزمايش مايكلسون_مورلي رو بشناسيم!اين دونفر نشون دادن كه سرعت نور اصلا ربطي به اجسام ديگه نداره وتحت تاثير اونها نيس.يعني مثل جمع سرعت دوماشين با سرعت هاي مختلف برا به دست آوردن مقدار اونها نسبت به هم نيست وكلا نميشه!:43:اونها در تلاش بودن سيال اتر رو آشكاركنن كه فك ميكردن تو كل دنيا هست وعامل حركت نور در فضا اونه!بيچاره ها خيلي تلاش كردن ولي تيرشون به هدف خودشون نخوردو به يه سمت هدف بهتر ديگه رفت.(تير كج به منزل نميرسد:دی )وبه كشفيات مهمي رسيدن!
اگه شما با ماشينتون با سرعت70km/hدر حال حركت باشين ودوستتون باسرعت 85km/hاز كنار شما رد شه شما فك ميكنين كه دوستتون با سرعت15km/hنسبت به شما در حال حركته!تفريق كنين سرعت هارو:دی
ولي نور اين طوري نيس.اگه شما باسرعت نزديك به c در حال حركت باشين نور از كنار شما باسرعتcدوباره رد ميشه:wut:
تو اين ميان همه ميگفتن تقصير دستگاه هاس كه اشتباه جواب داده ولي يه جووني به اسم انيشتين يه حرف ديگه زد...
حالا بقيه اش رو شما بياين كه چي كار كرد وكلا اصول اين آزمايش چي بود؟!!!:)

mohsen4465
06-27-2013, 01:34 PM
اگه شما باسرعت نزديك به c در حال حركت باشين نور از كنار شما باسرعتcدوباره رد ميشه:wut:
البته مثال رو یخورده بدجور زدید. اگه با سرعت مثلاً نصف سرعت نور در حال حرکت باشیم (یا هر سرعت دیگه ای) و یه چراغ قوه دستمون باشه و اونو روشن کنیم، سرعت چراغ قوه که تو دست ماست به سرعت نور اضافه نمیشه و نور بازهم با همون سرعت c خارج میشه. یعنی نور چراغ قوه با نصف سرعت نور از ما دور میشه و برای یه ناظر خارجی همچنان همون c هست. :have a nice day:
------------------------------------------------------------------
با صحبتی که با آیسا خانم داشتم انگاری قضیه پیچیده تر از این حرفهاست و حق با ایشونه. آقا احسان بیاید کمک. این نسبیت انیشتین رو من آخر نفهمیدیم چی میگه!:grin:

Ehsan
06-28-2013, 12:28 PM
آروم آروم پیش میریم! :دی

تا حدود سالهای 1870 نظریه ی الکترومغناطیس ماکسول کامل شده بود و در پیش بینی پدیده های الکتریکی و مغناطیسی بسیار موفق از آب در اومده بود، یکی از خفن ترین موفقیت های این نظریه توضیح ِ «نور» بود، نظریه ی ماکسول به وضوح وجود ِ امواجی از جنس الکترومغناطیس رو پیش بینی می‌کرد که سرعتشون تطابق ِ عجیبی داشت با سرعت ِ اندازه گیری شده از نور تا اون زمان.

به همین جهت و به خاطر پیشرفت نظریه ی امواج الکترومغناطیسی و کاربرد و آزمایشاتِ گسترده حول این امواج و کلی شواهد ِ دیگه، خیلی ها پذیرفتند که نور هم یک موج ِ الکترومغناطیسی هستش.

نظریه ی ماکسول یک پیش بینی جالب داشت، سرعت امواج الکترومغناطیسی رو دقیقا c پیشبینی می کرد، نه ذره ای کم و نه ذره ای زیاد. اما این سرعت نسبت به کدوم دستگاه بود؟ تجربه ی گذشته ی ما میگه اگر اتوموبیلی با سرعت ِ 80 بره و ما در اتوموبیلی با سرعت 70 موازی قبلی حرکت کنیم، سرعتی که ما از اتوموبیل ِ 80 کیلومتری خواهیم دید، 10 کیلومتر در ساعت خواهد بود، همین تجربه میگه اگر ما به سمت ِ نور حرکت کنیم سرعت ِ نور با سرعت ِ ما جمع میشه و اگر ازش دور بشیم سرعت ِ ما کسر میشه. اما این معادلات دقیقا پیش بینی می کردند که سرعت c هستش، پس این c باید نسبت به محیطی باشه دیگه؟ نمی تونه که همین طوری تو هر دستگاهی c باشه که؟ می تونه؟ دِ نمی تونه! :دی

از این گذشته با توجه به تجربه ی فیزیک دانها از موج، هر موجی برای انتشار نیاز به محیطی داره، بنا بر این دانشمندان تصمیم گرفتند در پی ِ یک محیط ِ فرضی به نام ِ اتر aether باشند تا موج ِ نور رو منتقل کنه و اعلام کردند که یک اتر ِ جهانی وجود داره که موج ِ نور درون ِ اون با سرعت ِ c حرکت می کنه. اتر برای نور درست مثل ِ هواست برای امواج صوتی، امواج ِ صوتی درون ِ هوا حرکت می‌کنند. اگر ما به سمت امواج صوت حرکت کنیم سرعت ِ اونها رو بیشتر خواهیم یافت. مثلا اگر با سرعت 20 متر بر ثانیه درون هوا حرکت کنیم خواهیم دید که سرعت ِ صوتی که به سمت ِ ما میاد به جای 340 متر بر ثانیه، 360 متر بر ثانیه هستش.

اما اتر چطور آشکار می‌شد؟ اتر باید بسیار لطیف باشه چون تاثیر ِ بسیار کمی در محیطهای میانستاره ای و میان سیاره ای داره در عین حال باید بسیار محکم باشه چون سرعت ِ نور به طرز غیر قابل باوری زیاده.

با ایده ی این که نور درون ِ اتر سرعتش ثابت هستش میشه با اندازه گیری ِ سرعت ِ دقیق ِ نور در جهت های مختلف درون ِ یک آزمایشگاه به سرعت ِ زمین درون ِ اتر ِ فضایی پی برد و یه جورایی اتر رو آشکار کرد. این اندازه گیری قرار بود با کمک آزمایش مایکلسون-مورلی انجام بشه که انتشار نتایجش در سال ِ 1887 برای جامعه ی فیزیک یک شوک بزرگ بود:

سرعت ِ زمین درون ِ اتر ِ فضا همیشه صفره!!!

این نتیجه غیر ممکن بود! با توجه به جهت متغییر سرعت زمین در طول سال و حرکت ِ خورشید به دور کهکشان و هزار تا حرکت ِ جور وا جور ِ دیگه این نتیجه غیر ممکنه. قطعا یک حرکتی باید باشه، اما با آزمایش های بیشتری که با دقتهایی معادل هزاران برابر آزمایش مایکلسون-مورلی تکرار شد، دوباره همین نتیجه به دست اومد:

سرعت ِ زمین درون ِ اتر صفره! یا به عبارتی سرعت ِ نور نسبت به آزمایشگاهها همیشه برابر ِ c هستش، این آشکارا با هر آنچیزی که در بالا گفتم در تناقضه!

چی غلطه؟ معادلات ِ ماکسول؟ یا تبدیلات ِ سرعتی که ما انجام میدیم؟

«تبدیلات ِ سرعت نمی‌تونه اشتباه باشه چون اون وقت باید بپذیریم که فضا و زمان منقبض و منبسط میشن و این اصلا قابل قبول نیست، پس احتمالا معادلات ِ ماکسول باید اشکال داشته باشند!» این استدلالی بود که خیلی از فیزیکدانان ِ اون زمان با توجه به موفقیت عظیم ِ مکانیک ِ نیوتونی (که مبتنی بر همین تبدیلات بود) در اون سالها پذیرفتنش و در پی تصحیح نظریه نور بر اومدند اما آلبرت ِ قصه ی ما که اون موقع ها بیست و چند سال داشت جور ِ دیگه ای فکر می کرد :دی ;)

ـــــــــــــ
ابهامی اگر هست در خدمتم :)

narcissus flower
06-29-2013, 11:58 PM
سلام
البته احسان خان همه چیزو گفتند ولی کلا چه تصوری از حضور اتر در ذهن افراد میتونه شکل بگیره؟
یعنی چی که کل جهان رو اتر پر کرده؟؟
تصور میشد که اگر کل جهان از اتر پر شده باشه, زمین در هنگام حرکتش در این ماده , مثل کشتی که در آب حرکت میکنه و آب رو جابه جا میکنه , هست وانتظار داشتند مثل آب یک مقاومتی در برابر ژمین حس بشه.اما حس نشد!!.
از طرفی اگر اتر وجود میداشت باعث میشد که نوساناتی در مسیر حرکت زمین ایجاد بشه. مثل اینکه مثلا در مسیر باد حرکت کنیم یا در جهت مخالفش , و این تو مسیرش اثر میذاشت .و اصلا عبارتی به نام باد اتری هم به وجود اومد.
حالا فکر میکردند که نور هم باید درگیر بشه با این شرایط , اما با ازمایش مایکلسون ومورلی معلوم شد که سرعت نور در هر مسیر وراستایی و در هر شرایطی همونی هست که همیشه بوده.
پس نتیجه گرفتند که سرعت نور که ثابته هیچ , این اتره هم مثل اینکه نیست ...چون اگر بود تاثیری در حرکت ایجاد میکرد

Ehsan
07-03-2013, 04:53 AM
بیاید دوباره برگردیم وسطای قصه و این‌دفعه کمی محض‌تر به داستان نگاه کنیم:

گفتیم دانشمندان چهار تا معادله (معادلات ماکسول) پیدا کردند که پیش بینی می کرد نور دقیقا باید با سرعت c سیر کنه اما از طرفی از روی تبدیلات سرعت این انتظار رو هم داشتند که با حرکت کردن در راستای نور، سرعتهایی غیر از c از نور مشاهده خواهیم کرد. معنی این اتفاق چیه؟

اولا چون سرعت ِ c از نور با معادلات ِ ماکسول به دست اومده بود، وقتی با حرکت کردن سرعت ِ نور دیگه c نباشه، پس معادلات ماکسول تغییر شکل دادند چون اگر شکلشون همون قبلی باقی می‌موند می‌شد نشون داد که سرعت c باقی می‌مونه.

ثانیا حد اقل یک دستگاه وجود داره که سرعت ِ نور اونجا c هستش (معادلات ِ ماکسول به فرمی هستند که اولش ارائه شده بودند)، با توجه به نیاز امواج به ماده ی واسطی برای انتشار (مثلا امواج صوتی به هوا برای انتشار نیاز دارند)، دانشمندان و فیزیکدانان اون زمان وجود ماده ای به اسم اتر رو پیش‌بینی کردند که امواج نور در اونها با سرعت c سیر می‌کنه به عبارتی معادلات ماکسول وقتی درون اون ماده ساکن هستیم، کار می‌کنه.

وجود اتر از چند لحاظ حائز اهمیت بود که شاید مهمترین ِ اونها بحث ِ وجود ِ یک دستگاه ِ مطلق ِ جهانی باشه، چیزی که نیوتون برای توجیه ِ عکس‌العمل مواد در برابر شتاب پیشبینیش کرده بود (برای توضیحات بیشتر به بحث فضای مطلق نیوتونی مراجعه کنید :دی )


اگر ما توی اتر با سرعت v حرکت کنیم، سرعت امواج نور رو بین c+v تا c-v خواهیم دید. با توجه به حرکت پیچیده ی زمین در فضا که هم دور خودش می‌گرده و هم دور خورشید و هم دور کهکشان، حتما باید با آزمایشاتی که به تغییرات سرعت نور حساس هستش، باید بشه سرعت ِ زمین درون ِ اتر رو پیدا کرد.

آزمایشات مایکلسون-مولری و چندین آزمایش دیگه که برای پیدا کردن ِ این سرعت بودند همگی با شکست مواجه شدند، سرعت ِ زمین در همه ی اون آزمایش ها صفر بود! به عبارتی با هر حرکتی درون ِ فضا، سرعت ِ نور c بود که c بود!

از سوی فیزیکدانان اون زمان، تلاشهای زیادی برای توجیه این نظریه و تصحیح نظریه نور انجام شد که همگی با شکست مواجه شدند و آزمایش هایی وجود داشت که این تلاشها رو نقض می‌کرد. غیر از یک نظریه که تا اون زمان هیچ آزمایشی ردش نکرد، اشتباه نکنید! نظریه اینتشتین نه! بلکه نظریه ی لورنتس:

لورنتس فرض کرد که با حرکت درون اتر، طولها منقبض می‌شه و زمان منبسط و توانست با تبدیلات ِ ریاضیی که برای فضا و زمان به دست آورد نشون بده که نتیجه ی آزمایش مایکلسون مورلی کاملا منطقی هستش، اما این نظریه یک اشکال ِ کوچک داشت: با این اتفاق هیچ جوری نمی‌شد سرعت ِ خودمون رو درون ِ اتر آشکار کنیم!*

کمی الکترومغناطیسی تر:

حالا بیاید از یک منظر دیگه به داستان نگاه کنیم، از منظر معادلات ماکسول: گفتیم معادلات ِ ماکسول باید با حرکت کردن درون ِ اتر تغییر شکل بدن، و آزمایش مایکلسون-مورلی به طور ضمنی قرار بود تغییر شکل ِ این معادلات رو بررسی بکنه.

(وقتی معادلات ماکسول از c بودن ِ سرعت ِ نور خبر می‌دن ولی با حرکت کردن سرعت ِ نور دیگه c نیست، پس با حرکت کردن دیگه معادلات ماسکول مثل قبل کار نمی‌کنند و تغییر کردند)

قانون القای فارادی رو همتون شنیدید: اگر من یک آهنربای متغیر داشته باشم و یک حلقه ی رسانا، با تغییر میزان میدان مغناطیسی درون ِ اون حلقه ی رسانا ولتاژی القا می‌شه و جریانی درش به راه می‌افته.

به عبارتی اگر من یک آهن ربا رو به یک حلقه ی آهنی دور و نزدیک کنم، بارهای آزاد ِ توی حلقه شروع به حرکت کردن و جاری شدن خواهند کرد.

ترکیب ِ قانون ِ القای فارادی با بقیه ی قوانین الکتریسیته بزرگترین دستاورد ماکسول بود و نتیجه اش همان معادلات ِ معروف ِ ماکسول ِ ما شد. پس قانون ِ القای فارادی جزئی از قوانین ِ ماکسول هستش.


از طرفی میدان ِ مغناطیسی می‌تونه بارهای متحرکی که درونش هست رو منحرف کنه و بهشون نیرو وارد بکنه، این قانون ِ نیروی لورنتس هستش

حالا بیاید یک آزمایش رو تصور بکنیم:

فرض کنید که یک آهنربا رو به یک حلقه ی ساکن نزدیک می‌کنم، بنا به قانون القای فارادی میدان مغناطیسی که اطراف حلقه تغییر می‌کنه و بارهای الکتریکی درون ِ حلقه حرکت می‌کنند و جریان ایجاد میشه.

حالا این شکل رو ببینید:

http://www.modernrelativitysite.com/fig1.gif

این شکل به ما می‌گه که ما می‌تونیم این آزمایش رو از یک منظر ِ دیگه نگاه کنیم: فرض کنید آهن ربا ساکن هستش و حلقه متحرکه (به عبارتی ما روی آهن ربا نشستیم)، با توجه به این که حلقه درونش بارهای الکتریکی آزاد داره، با عبور از میدان ِ مغناطیسی طبق قانون نیروی لورنتس بهشون نیرو وارد میشه ور درون ِ حلقه شروع می‌کنند به حرکت و یک جریان ایجاد می‌کنند.

http://www.pitt.edu/~jdnorton/teaching/HPS_0410/chapters/origins_pathway/magnet3.gif

دقت کنید این دو تا تعبیر زمین تا آسمان با هم فرق دارن:

در اولی ما از یک قانون ِ بنیادی در باب ِ میدان مغناطیسی متغییر حرف می‌زنیم (قانون فارادی) اما در دومی ما داریم راجع به نحوه ی وارد شدن ِ نیروی مغناطیسی به بارهای الکتریکی متحرک حرف می‌زنیم (قانون نیروی لورنتس) و این دو تا قانون هیـــــــچ ربطی به هم ندارند.

دست ِ بر قضا با استفاده از این دو قانون کاملا مجزا، نتیجه ای یکسان از این آزمایش بیرون میاد، یعنی هر دو تعبیر یک میزان جریان رو پیش‌بینی می‌کنند، به عبارتی با هر حرکتی تو فضا، جریانی که برقرار می‌شد، یکسان بود که یکسان بود! اما کدام تعبیر صحیحه؟

از نظر فیزیکدانان اون زمان، با توجه به تغییر شکل معادلات ماکسول به واسطه ی حرکت و با توجه به فرض اولیه ساکن بودن ِ حلقه، تعبیر اولی صحیصه و این یکی بودن ِ نتیجه ی دو تعبیر صرفا یک تصادفه.

دقیقا همین جاهاست که اینشتین قصه ی ما وارد ِ ماجرا میشه، این جون ِ رعنای ما درست مثل همیشه یک جورایی متفاوت از بقیه فکر می‌کرد.

بقیه اش به خاطر طولانی شدن بمونه واسه بعد!

ـــــــــــــــــــــــــ ــــــ
*البته این نظریه به خاطر ِ تعبیر غیر صحیحش در یک آزمایش جواب نداد اما از اونجایی که این آزمایش در سال 1932 انجام شد (آزمایش کندی-تورندیک)، و این قضایا مال حدود 1900 هستش پس می‌تونیم بگیم نظریه ی لورنتس تا اون زمان کاملا موفق بود.

Ehsan
07-06-2013, 06:20 AM
خوب خوب خوب! قصه داره به جاهای خوبش می‌رسه. (این که همیشه همه به تاریخ ِ نظریه نسبیت اشاره دارند اینه که بخش عظیمی از مفاهیم نظریه نسبیت در تاریخش نهفته است)


ما بالا دو تا مورد رو شرح دادیم: یکم این که آزمایش مایکلسون-مورلی صریحا بیان می‌کرد که علی‌رغم پیشبینی فیزیک کلاسیک سرعت ِ ما درون ِ اتر صفره! این نتیجه غیر قابل توجیه بود، لورنتس نامی ادعا کرد که طول‌ها درون ِ اتر منقبض میشن و زمان با حرکت درون اتر اتساع پیدا می‌کنه بنا بر این نتیجه ی آزمایش مایکلسون مورلی رو تایید و تفسیر کرد، فقط اشکال ِ کار اینجا بود که دیگه نمیشد اتر رو آشکار کرد.

دوم این که یک آزمایش الکترومغناطیسی دو تعبیر ِ مختلف داشت که دست بر قضا هر دو تعبیر به نتیجه ی یکسان منجر می‌شدند اما از نظر فیزیک دانها فقط یکی از این تعابیر صحیح بود.


حالا اینشتین ِ ما وارد ِ قصه میشه:

اینشتین ما این طوری به قضیه نگاه کرد: اون دید که تبدیلات لورنتس به ما می‌گه ما هر سرعتی نسبت به اتر داشته باشیم، نمی‌تونیم سرعت ِ خودمون رو درونش آشکار کنیم، یعنی مثلا اگر من درون ِ اتر ساکن باشم یا با سرعت نصف سرعت نور درونش حرکت کنم، از لحاظ «قوانین فیزیکی قابل آشکارسازی» هیچ تغییری ایجاد نمی‌شه. اما معنی این که «از لحاظ «قوانین فیزیکی» هیچ تغییری ایجاد نمی‌شه» چیه؟ کلید ِ درک نسبیت در درک همین جمله است!

فرض کنید من یک جعبه دارم که توش پر از قوانین ِ فیزیک هستش، این قوانین سنگ ِ محک‌هایی هستند برای اندازه گیری بعضی کمیت ها: مثلا قوانین ِ مکانیک نیوتون برای اندازه گیری شتاب و جرم، قوانین ترمودینامیک برای اندازه گیری دما و حرارت و .....

گاهی با اتفاقاتی که در دستگاهای مختصات رخ میده این قوانین تغییر می‌کنند مثلا اگر من درون ماشینی که در حال ترمز گرفتن هستش بخوام قوانین ِ نیوتون رو پیاده کنم خواهم دید که این قوانین دیگه کار نمی‌کنن! وقتی من ذره ای رو درون ِ این ماشین رها میکنه ذره به عقب کشیده میشه و این با قانون اول نیوتون ناساگاره.

بحث ِ فیزیک ِ کلاسیک این بود که قوانین ماکسول با «حرکت با سرعت ِ ثابت در فضا» عوض می‌شن و تغییر شکل میدن (مثل قوانین نیوتون که توی ماشین ِ در حال ترمز کار نمیکنه)، بنا بر این سنگ ِ محکی به ما میدن تا بفهمیم که داریم حرکت می‌کنم یا ساکن هستیم و به عبارتی سرعت رو اندازه بگیریم (به قرینه: قوانین ِ نیوتون هم سنگ محکی میدن تا بفهمیم که در حال ِ حرکت ِ شتابدار هستیم یا نه به عبارتی شتاب رو اندازه بگیریم)

تعبیر ِ لورنتس حاوی این نکته بود که قوانین ِ ماسکول حداقل در مقام ِ اندازه گیری چنین سنگ ِ محکی نیستند، چون طولها و زمان با حرکت درون اتر طوری تغییر می‌کنه که سرعت ِ نور همیشه سرعت ِ نوره و قوانین ِ ماکسول شکلشون همون شکلِ قبلی هستش. بنا بر این نمیشه با قوانین ِ ماکسول، اتر رو آشکار کرد، از طرفی توی جعبه ی جادویی ما که پر از قوانین ِ فیزیکه، هیچ قانون ِ دیگری هم نیست که حرکت ما درون ِ اتر رو آشکار کنه => پس من هیچ ابزار ِ فیزیکی برای آشکار کردن ِ اتر ندارم=> پس اتر هیچ تاثیری رو قوانین ِ فیزیک نداره=> اتر هیچ تاثیری روی ما نداره=> از لحاظ ِ فیزیکی اتر مفهومی زاید و اضافیه!!!=> اتر وجود نداره!

اینشتین به جای فرض ِ پیچیده ی اتر و حرف ِ پیچیده تر ِ لورنتس که ما درون ِ اتر منقبض میشیم یک فرض ِ بسیار زیباتر و قشنگتر جایگزین کرد که تمام ِ آزمایش های مشابه مایکلسون مورلی رو توجیه می‌کرد:

هیچ دستگاه مطلقی وجود نداره، جعبه ی جادویی من که پر از قوانین ِ فیزیک هستش، با حرکت ِ سرعت ِ ثابت، عوض نمیشه. یعنی قانون ِ ماکسول قانون ِ ماکسول باقی میمونه (این برای تمام ِ قوانین ِ فیزیک صادقه)

فرض کنید که من یک اتوموبیل پیدا کردم که جعبه ی قوانین فیزیکم توش کار می‌کنه (با آزمایش می‌فهمم)، هر ماشین ِ دیگه ای به هر جهتی و با هر سرعتی حرکت بکنه، قوانین ِ پایه ی فیزیک باید درونش کار بکنه و جعبه ی قوانین ِ من برای اون ماشین هم درسته.

حالا اگر یادتون بیاد قانون ِ ماکسول پیش بینی می کرد که سرعت ِ نور c باشه، پس این ناوردا می‌مونه و با هر سرعت در هر دستگاهی ما سرعت ِ نور رو c اندازه گیری خواهیم کرد :دی

شاهدی که اینتشتین بر این مدعا آورد آزمایشی با حلقه و آهنربا بود که آخر ِ پست ِ قبل شرح دادم:

اینشتین قصه ی ما گفت که : تعبیر ِ هر دو ناظر (ناظر ِ روی آهن ربا و ناظر ِ همراه با حلقه) از آزمایش صحیحه، اینها فقط دو نگاه ِ مختلف از یک پدیده ی واحد هستند:قوانین ِ ماکسول!

به عبارتی همون حرف ِ قبلی که قوانین ِ ماسکول با حرکت کردن عوض نمی‌شن.

بنا بر این سرعت ِ نور همیشه c اندازه گیری میشه. به عبارت ِ خیلی ساده چون خود ِ همین قوانینی که ما با خودمون همراه داریم و توی جعبه‌ی جادوییمون این ور و اون ور می‌بریم تا سرعت ِ نور رو بسنجیم، سرعت ِ نور رو تعیین می کنه، بنا بر این نمیشه با این قوانین سرعت ِ نور رو چیز ِ دیگری اندازه گرفت. سرعت ِ نور همیشه ثابته.

ــــــــــــــــــــ
دیگه از پست ِ بعد انشالله از تاریخ میایم بیرون و راجع به نسبیت به طور کاملا منسجم حرف میزنیم :)

ابهامی هست اگر در خدمتم

حتما بپرسید

مداد رنگیهام
07-06-2013, 02:03 PM
میشه در مورد این جمله بیشتر توضیح بدین؟:)
"...فرض کرد که با حرکت درون اتر، طولها منقبض می‌شه و زمان منبسط و توانست با تبدیلات ِ ریاضیی که برای فضا و زمان به دست آورد نشون بده که نتیجه ی آزمایش مایکلسون مورلی کاملا منطقی هستش..."

arashgmn
07-06-2013, 04:16 PM
... بنا بر این نمیشه با قوانین ِ ماکسول، اتر رو آشکار کرد، از طرفی توی جعبه ی جادویی ما که پر از قوانین ِ فیزیکه، هیچ قانون ِ دیگری هم نیست که حرکت ما درون ِ اتر رو آشکار کنه => پس من هیچ ابزار ِ فیزیکی برای آشکار کردن ِ اتر ندارم=> پس اتر هیچ تاثیری رو قوانین ِ فیزیک نداره=> اتر هیچ تاثیری روی ما نداره=> از لحاظ ِ فیزیکی اتر مفهومی زاید و اضافیه!!!=> اتر وجود نداره!
...[/B]
اصلا کل این دو پست یه طرف ، این دو خط یه طرف دیگه ...! تشکر ... تشکر...

فقط یه نکته ای. توی دو پست قبلی یکی از تصاویر نشون داده نمی شه. اگر ممکنه یه جای دیگه آپلود کنید تا کامل درک کنیم آنچه را که نوشتید... :)

aysa
07-06-2013, 11:00 PM
میشه در مورد این جمله بیشتر توضیح بدین؟:)
"...فرض کرد که با حرکت درون اتر، طولها منقبض می‌شه و زمان منبسط و توانست با تبدیلات ِ ریاضیی که برای فضا و زمان به دست آورد نشون بده که نتیجه ی آزمایش مایکلسون مورلی کاملا منطقی هستش..."

اين موضوع از نسبيت سرچشمه ميگيره كه با نزديك شدن به سرعت نور طول جسم رفته رفته كاهش يافته ودر انتها صفرميشه.در اين موقعيت زمان براتون طولاني تر هم ميشه.مثلاتو اين سرعت حالا با فرض اين كه وجود دارين اگه زمان براشما 20 سال مثلا بگذره برا كسي كه رو زمين بوده ودر حال زندگيه عاديش بوده چند صد سال گذشته.در ادامه كه مربوط به نسبيت هست به طور كامل شرح داده خواهد شد.:)
-------------------------------------------------------------
حالا سؤال خودم:اين بالا گفته شده تو الكترو مغناطيس يكي صرفا برحسب تصادفه واون يكي حقيقته.
ودر پست پايين انيشتين گفته هردوتاش يكين وفقط عوامل فرق داره.الان من دقيقا متوجه نشدم اين وسط حرف انيشتين درست بود يا غلط يا كلا اين دوتا يه موضوع اند؟!
باهم فرق دارن؟
2:ميشه در مورد جمله هاي آخرتون در مورد اين كه انيشتين از اين موضوع برا اثبات vنور استفاده كرده بيشتر توضيح بدين؟برام اين جاش گنگ بود!
--------------------------------------------------------------
دوستان اين جا فقط قرار نيست يه نفر پست بذاره بقيه بخونن.يه نفرم بياد وسط گاهي يه چيزي بپرونه(خودمو ميگما:دی)لطفا همكاري كنين ديگه!
بايد برين 8ترم تو دانشگاه كوانتوم بخونين تا قدرشو بدونين؟:دی مفتو مجاني گذاشتن جلوتون. امتحان دادنم نداره.يه پستي چيزي بذارين ديگه.اگه جوابتون درست بود چه بهتر ولي اگه نشد اصلا نگران نباشين.

narcissus flower
07-07-2013, 12:08 AM
میشه در مورد این جمله بیشتر توضیح بدین؟:)
"...فرض کرد که با حرکت درون اتر، طولها منقبض می‌شه و زمان منبسط و توانست با تبدیلات ِ ریاضیی که برای فضا و زمان به دست آورد نشون بده که نتیجه ی آزمایش مایکلسون مورلی کاملا منطقی هستش..."


من یک پرانتز این وسط باز کنم , تا مستر احسان ادامه بدهند :دی!!!
من در مورد انقباض طول یه مطلبی روبگم.که اصلا چه اتقافی رخ میده؟؟

فرض کن من وشما باهم میخوایم اندازه ی یک خط رو بسنجیم .اما من در یک دستگاه مختصات هستم وشما دریک دستگاه دیگه...دستگاه یعنی شما برای خودت یک مجموعه محور مختصات دارید ومن هم برای خودم. هر دوتا مون خط کش دست منو 20 سانت اندازه میگیریم

حالا من تصمیم میگیرم خط به دست همراه با چارچوب مربوط به خودم در راستای x مثلا حرکت کنم...واز شما بخوام که بگی خط کش من چند سانته؟؟
حقیقت امر اینه که قبلا ها اقای گالیله تبدیلات خودشو داشت و تحلیل های مربوط به خودشو.اما دیده شد که وقتی سرعت بالا میره دیگه تبدیلات گالیله ای معتبر نیست.اینکه غلظه یا نه , باید بگم غلظ نیست , بلکه در سرعت بالا هست و حقیقت رو به ما نمیگه...حالا چه کنیم؟
در این موقع ما میایم از تبدیلات لورنتس استفاده میکنیم.به این صورت که هم در سرعت بالا معتبر هست وهم در سرعت پایین به تبدیلات گالیله ای مبدل میشه...

خب وقتی سرعت بالا میره شما دیگه خط کش من رو با انچه من میسنجم برابر نیست.یعنی شما خط کش منو کوتاه تر از حالت اصلی میبینی...کدوم درسته؟هر دوتا .ما هر کدوم نسبت به این خط کش و چارچوبمون داریم درست اندازه میگیریم.منتها من چون نسبت به خکشم ثابت هستم همون 20 سانت رو میگیرم (چون من وخط کشم هر دو تحت تاثیر یک شرایط هستیم.نمیشه که خط کش دستم هی کوچک بشه واونقت من همون اندازه باشم.بنابراین اگر من خط کشم رو نگاه کنم میبینم درسته , حالا مشا هرچی بهم بگی نه کوچک تره من نمیتونم بپذیرم .چون منم درگیر همون شرایط خطکش هستم.)

ولی چون خط کش نسبت به شما ساکن نیست یه چیز دیگه... پس ما با یک دید نسبی رویداد ها رو میبینیم...که اینجا بحث ناظر ها و چار چوب نمود پیدا میکنه...پس وقتی میگیم یک رویداد ,چنین وچنان ویژگی داره ما باید بپرسیم نسبت به کدوم ناظر .؟

در این بین زمان هم نسبت به دستگاه ها دچار تغییر میشه....

حالا در این تبدیلات لورنتسی , سرعت نور خیلی قشنگ حضور به عمل میاره .
و اگر این فکر به سرمون بزنه که مبادا بتونیم از سرعت نور پیشی بگیریم این تبدیلات ,اجازه نمیدهد .چون که یک پدیده در یک چار چوب حقیقی هست ولی ممکنه دریک چارچوب اینگونه نباشه...
اگر یک جستجوی کوچولو در موردش بکنی , فرمولاش یافت میشه :have%20a%20nice%20d

امیدوارم روند صحبت اقا احسان به هم نخورده باشه:)

Ehsan
07-07-2013, 04:05 AM
البته دوستان منظور ِ من انقباض به دیدگاه ِ اتری بود نه دیدگاه ِ نسبیتی :دی


میشه در مورد این جمله بیشتر توضیح بدین؟:)
"...فرض کرد که با حرکت درون اتر، طولها منقبض می‌شه و زمان منبسط و توانست با تبدیلات ِ ریاضیی که برای فضا و زمان به دست آورد نشون بده که نتیجه ی آزمایش مایکلسون مورلی کاملا منطقی هستش..."

آزمایش مایکلسون مورلی کلا داستانش اینه:

این شکل رو ببینید:
http://upload.wikimedia.org/wikipedia/commons/e/e7/Interferometer.svg?uselang=fa

تو این شکل یک پرتوی نور از منبع نور رها می‌شه و به سمت یک آینه ی نیمه اندود میره. آینه ی نیمه اندود پرتو رو میشکافه، قسمتی از پرتو عبور می کنه و قسمتی بازتاب میشه به بالا، طوری که مسیر حرکتشون عمود بر هم باشه، و هر دو پرتو با برخورد به یک آینه ی کاملا اندود، بازتاب شده و برمیگردند و با رسیدن به همون آینه ی نیمه اندود ِ قبلی، یکی میشن و میرن به سمت آشکارساز یا detector حالا اگر آزمایشگاه درون ِ اتر حرکت کنه چه اتفاقی می افته؟ از اونجایی که یک شکل از هزارتا حرف گویا تره شکل ِ اتفاق رو میگذارم:

http://upload.wikimedia.org/wikipedia/en/1/12/MichelsonMorleyAnimation.gif

یکی از پرتوها دیرتر میرسه! (اصطلاحا می گن اختلاف فاز پیدا میشه) این دیرتر رسیدن هیچ وقت تو آزمایش دیده نشد. لورنتس فرض کرد بازویی که در راستای حرکت قرار داره، با حرکت درون اتر منقبض میشه و طولش کم میشه و زمان آزمایشگاه هم کند تر میشه به همین خاطر پرتو بالایی هم کمی دیرتر میرسه و چپی کمی زودتر میرسه و در برگشت هر دو همزمان میرسن به آینه! در نتیجه اختلاف فاز دیده نمیشه، اما اشکالات ِ این دیدگاه رو گفتم :)





-------------------------------------------------------------
حالا سؤال خودم:اين بالا گفته شده تو الكترو مغناطيس يكي صرفا برحسب تصادفه واون يكي حقيقته.
ودر پست پايين انيشتين گفته هردوتاش يكين وفقط عوامل فرق داره.الان من دقيقا متوجه نشدم اين وسط حرف انيشتين درست بود يا غلط يا كلا اين دوتا يه موضوع اند؟!
باهم فرق دارن؟
2:ميشه در مورد جمله هاي آخرتون در مورد اين كه انيشتين از اين موضوع برا اثبات vنور استفاده كرده بيشتر توضيح بدين؟برام اين جاش گنگ بود!
--------------------------------------------------------------


1.مشکل ِ اول این بود که ما یا باید فرض میکردیم بارهای الکتریکی ساکن هستند و با قانون القای فارادی موضوع رو تحلیل می کردیم، یا باید فرض می کردیم بارهای الکتریکی در حال حرکت هستند و بارهای متحرک درون ِ میدان مغناطیسی بهشون نیرو وارد میشه و جریان ایجاد میشه. حالا از نظر محاسباتی نتیجه ی این دو تا فرض یکی بودن اما از دید ِ دانشمندان فقط یکیشون صحیح بود.
اینشتین گفت که هر دو دید صحیحه، فرض کنید دانشمندان می گفتن حلقه متحرکه و اونی که همراه ِ حلقه حرکت میکنه (یعنی حلقه براش ساکنه) و قانون القای فارادی رو واسش می‌نویسه در اشتباهه (البته اون چیزی که تو آزمایش گفتم برعکس ِ این بود!!) اینشتین گفت اونی که قانون القای فارادی رو می‌نویسه هم راست میگه، چون قوانین ِ فیزیک با حرکت کردن عوض نمیشن. همه قانون ِ القای فارادی رو درست خواهند یافت.
در واقع این دو تحلیل درسته ظاهرا متفاوت هستند اما در اصل از یک قانون نشات می‌گیرن پس فرقی با هم ندارند.

2.بحث اینه که حالا که قوانین ِ ماکسول که پیش بینی کننده ی سرعت نور هستند با حرکت کردن عوض نمیشن (شاهدش همون آزمایش هستش :دی) پس سرعت ِ نور هم که نتیجه ی مستقیم قانون ماکسول هستش عوض نمیشه.


اصلا کل این دو پست یه طرف ، این دو خط یه طرف دیگه ...! تشکر ... تشکر...

فقط یه نکته ای. توی دو پست قبلی یکی از تصاویر نشون داده نمی شه. اگر ممکنه یه جای دیگه آپلود کنید تا کامل درک کنیم آنچه را که نوشتید... :)

گفتم که کلید ِ درک نسبیت تو اون چند پاراگرافه چکیده اش هم اون دو تا خط :دی

حالا شما بگو کدوم شکلو می بینی تا درست بشه! البته به من نگو چون توانایی ویرایش ندارم :دی

aysa
07-07-2013, 07:51 AM
:))فك كنم ما كلا منظورو نفهميده بوديم.الان برا همه جا افتاد.ممنون



حالا شما بگو کدوم شکلو می بینی تا درست بشه! البته به من نگو چون توانایی ویرایش ندارم :دی
اولين شكل تو پست اولتون ديده نميشه.البته از اين دوتا پست آخري اولي رو ميگم.چي گفتم من؟همون خلاصه اولين شكل :دی
واسه منم نيوفتاده فك كردم هر دوتا موضوع يه شكل داره.الان دقت كردم.عجب متني خونديم ما.
بهتره تو يه پست ديگه بذارين لطفا:)
والان بحث بعدي چيه؟كي شروع ميكنه؟:)

Ehsan
07-17-2013, 04:37 PM
خیلی خوب! خواهران ای دوستان حال می‌خواهیم کم کم وارد بحث اصلی و مدون نسبیت خاص بشیم :دی


ابتدا باید چند مقدمه خدمتتون عرض بکنم و چند اصطلاح رو معرفی بکنم:

1.دستگاه مرجع یا چارچوب مرجع:

یک دستگاه ِ مختصات سه بعدی دکارتی هستش به همراه یک مختصه ی زمان که رویداد ها در اون دستگاه مختصات با چهار عدد مشخص میشه : سه عدد ِ مربوط به مکان و یک عدد مربوط به زمان.

مثلا تریکدن ترقه ای در فضا این طوری معلوم میشه:

5m,2m,1m,4s
یعنی در زمان چهار ثانیه و در مکان ایکس مساوی پنج متر، وای مساوی دو متر و زِد مساوی 1 متر اتفاق افتاده.

این دستگاه مختصات صرفا برای اندازه گیری رویداد ها به کار میره (همیشه در نظریات فیزیکی یک دستگاه مختصات در نظر گرفته میشه) و بهش میگن دستگاه مرجع یا چهار چوب مرجع.

2.دستگاه مرجع لخت:

قبل از این معرفی، بگذارید قانون اول نیوتون رو یاداوری بکنم:

در فضای آزاد، ذره ای که هیچ عاملی روش تاثیری نمی‌گذاره، یا با سرعت ثابت به سمتی حرکت خواهد کرد و یا اگر ساکن هست، ساکن باقی خواهد ماند. به عبارتی جسمی که بهش نیرو وارد نمیشه یا تا ابد ساکن میمونه یا تا ابد با سرعت ثابت به حرکتش روی یک خط ادامه میده. این قانون اول نیوتون هستش، کلید ِ این قانون هم در دو چیز نهفته است:

1.ذره نباید از چیزی تاثیر بپذیره.
2. ذره حرکتش خطی باقی میمونه.
در واقع قانون اول نیوتون اینه که اگر 1 =>2
سوال اینه که آیا 2=>1 ؟

از لحاظ فلسفی مشکلاتی هست که این قانون رو انتزاعی می‌کنه، مثلا ما از کجا می‌دونیم که هیچ چیزی به ذره تاثیر نمیگذاره، شاید عوامل زیادی روی حرکت ذره تاثیر میگذاره که نهایتا خطی بشه، شاید اصلا عوامل زیادی هست که با هم خنثی میشن! یعنی برایند تاثیرشون صفر میشه.

در واقع در این جا ما داریم یک تعریف انجام میدیم: تعریف ِ «نیروی صفر»! تعریف ِ نیروی صفر اینه که حرکت ذره یکنواخت باشه یعنی یا تا ابد با سرعت ثابت روی خط راست حرکت کنه یا تا ابد ساکن بمونه. در این صورت ما تعریف می کنیم که هیچ نیرویی بهش وارد نمیشه و یا اگر هیچ نیرویی وارد نشه ذره حرکتش یکنواخت باقی میمونه.

صفر ِ نیرو با قانون اول نیوتون مشخص میشه.

توی پرانتز: قانون دوم نیوتون تعریف ِ نیروی غیر صفر رو شامل میشه :دی یعنی قانون دوم نیوتون میگه که خیلی خوب! حالا اگر نیرو وارد بشه چه اتفاقی می افته!


حالا فرض کنید تو یکی از این دستگاه مختصات ها نشستیم، و داریم آزمایش انجام میدیم، آزمایش ِ ما پرتاب ِ یک ذره است، حالا اگر ذره از قانون اول نیوتون تبعیت کرد، یعنی حرکتش یک نواخت بود، ما اسم اون دستگاه مرجع رو میگذاریم دستگاه مرجع لخت یا چارچوب مرجع لخت.

برای مثال یک ماشین ِ در حال ترمز گرفتن چارچوب ِ مرجع ِ لخت نیست، چون اگر ذره ای رو رها کنم اون ذره به سمت جلوی ماشین کشیده میشه، اما یک ایستگاه فضایی در فضای بین کهکشانی دور از ستاره ها تقریبا یک دستگاه مرجع لخت حساب میشه.

3.تبدیلات بین دو دستگاه:

حالا فرض کنید که دو تا دستگاه مرجع دارم (که الزاما لخت نیستند) که ارتباط این دستگاه ها رو نسبت به هم میدونیم، مثلا دو دستگاه الف و دال داریم که موقعیت مبدا دستگاه مختصات دال رو در دستگاه الف می‌دونیم مثلا میدونیم که مبدا دستگاه مختصات دال از دید دستگاه مختصات الف در لحظه ی 2 ثانیه در نقطه ی 3,2,5- قرار داره (این سه عدد رو در هر لحظه میدونیم)

حالا فرض کنید یک رویدادی در دستگاه الف دارای موقعیت 2m,3m,1m,4s بود (m=meter,s=second) حالا سوال اینه که این رویداد در دستگاه ِ دال چه مختصاتی خواهد داشت؟

تبدیلات ِ بین دو دستگاه، یک تابع هستش که به ما میگه اگر رویدادی در دستگاه الف موقعیت x,y,z,t داشت، در دستگاه دال چه موقعیتی خواهد داشت، در واقع اگر در دستگاه دال موقعیت اون رو با 'x',y',z',t نشان بدیم، ربط این هشت عدد به همدیگر رو بهش میگن تبدیلات بین دو دستگاه.

مثلا یک تبدیلات خیلی معروف، تبدیلات گالیله ای هستش:

اگر دو دستگاه به شکل پایین داشته باشیم:

http://upload.wikimedia.org/wikipedia/commons/9/90/Standard_conf.png
(یعنی در راستای محور ایکس، یکی داره با سرعت ثابت نسبت به اون یکی حرکت میکنه)

در این صورت تبدیلات بین دو دستگاه به فرم گالیله ای به شکل پایین خواهد بود:

http://upload.wikimedia.org/math/f/d/a/fda95b59af07deb418cffa0b26aeea14.png
http://upload.wikimedia.org/math/d/2/9/d29a7f126abe0a92f5bb08e8260bd420.png
http://upload.wikimedia.org/math/4/b/7/4b724820c828fab4cd0d9b77b26b1073.png
http://upload.wikimedia.org/math/1/9/a/19a995484a0420ac0aac290eea0017fa.png

(لازم به ذکره که معادله ی چهارم در باب تبدیل زمان، در فیزیک کلاسیک به صورت کاملا ضمنی و زیرپوستی فرض میشه و فقط با اومدن نسبیت خاص بود که فرض چهارم به صورت صریح تبدیل به نماد فیزیک کلاسیک شد :دی)

حالا فعلا این معرفی ها رو داشته باشید، چند تا معرفی ِ دیگه انجام میدیم تا بگیم دقیقا نسبیت خاص چی میگه (این معرفی ها رو دست ِ کم نگیرید، این معرفی ها زبان ِ نسبیت خاص هستند، در صورتی که با اینها آشنا نباشید از نسبیت خاص سر در نمیارید)

Ehsan
07-21-2013, 07:47 PM
حالا بعد از این معرفی ها، لازمه که دو تا مفهوم رو معرفی و بررسی کنیم:

1.همگنی و همسانگردی
2.نسبیت

می‌خواهیم ببینیم معنا و مفهوم این واژه ها چیه؟

1.همگنی و همسانگردی

الف) همگنی:

معنایی که روزانه به همگنی نسبت میدیم چیه؟ مثلا میگیم یک محلول همگن، محلولی که همه جاش به صورت یکنواخت باشه و غلظت ثابتی داشته باشه، یا یک جامعه ی همگن، جامعه ای که فردی با فرد دیگه تفاوت فرهنگی چندانی نداشته باشه و... به طور کلی معنای عامیانه همگن اینه که یکنواخت و بدون تفاوت شدید باشه، یا کمی فیزیکی تر: یک ماده ی همگن، ماده ای که ویژگی های فیزیکی اون (مثل چگالی، یا ضریب پذیرفتاری دی‌الکتریک یا مغناطیسی یا....) در همه جای اون ماده ثابت و یکسان باشه.

معنای فیزیکی همگنی فضا اما کمی متفاوته. ویژگی های فضا چیه؟ دما یا چگالی که نمیشه به فضا نسبت داد، اینها ویژگی های جالبی نیستند، هر ماده ای میتونه این ویژگی رو به فضا تحمیل کنه اما خلا چنین ویژگی نداره، چه چیزی هست که همیشه هست و میشه به فضا نسبت داد؟

قوانین بنیادین فیزیک! بله! قوانین بنیادین فیزیک همیشه قابل دسترس و قابل آزمایش هستند و میشه به عنوان ویژگی ذاتی فضا به آنها نگاه کرد، اما معنای همگنی با قوانین بنیادین فیزیک چیه؟
همگنی در واقع به این معنی هستش که اگر من یک آزمایش الکتریکی یا مکانیکی مثل a رو اینجا انجام بدم و نتیجه b ازش بگیرم، و همین آزمایش رو در یک نقطه ی دلخواه دیگه از فضا انجام بدم و نتیجه ی c رو ازش بگیرم، الزاما b=c ، اگر به ازای هر دو نقطه ی دلخواهی این نتیجه برقرار باشه، فضای من همگن هستش.

در واقع با فرض همگنی، یعنی قوانین فیزیک همیشه و در همه ی نقاط ثابت و تغییر ناپذیر (اصطلاحا ناوردا) خواهند بود.


این شکل با بیان ساده همگنی رو نشان میده، یک آزمایش فیزیکی که یک باطری ده ولتی به یک مقاومت ده اهمی متصل شده و جریانش اندازه گیری میشه، در تمام فضا جریان 1 آمپر اندازه گرفته خواهد شد در غیر این صورت فضا ناهمگن هستش.
http://up.avastarco.com/images/r4rrwu5ez2tly0zj0ea.png



ب) همسان گردی

اما همگنی الزاما به ما نمی‌گه که اگر دستگاه رو بچرخانیم نتیجه ی آزمایش ثابت میمونه یا نه. یعنی اگر من سیم آزمایش بالا رو شرق-غرب ببندم یا شمال-جنوبی آیا نتیجه ی آزمایش باز هم تغییر نخواهد کرد و ثابت خواهد ماند؟ همگنی به این سوال جوابی نمیده اما ما اینجا فرض می‌کنیم جوابش مثبته یعنی نتیجه ی آزمایش تفاوتی نمی‌کنه ولی این یک فرض ِ جدید هستش که اسم این فرض رو می‌گذاریم همسانگردی.

همسانگردی در واقع شبیه همگنی هستش با این تفاوت که این بار به جای این که از تفاوت قوانین فیزیکی برای نقاط مختلف فضا بحث بشه، از تفاوتشون برای جهات مختلف فضا بحث میشه و گفته میشه که فضا همسانگرد هستش، این هم همان آزمایش قبلی با فرض همسانگردی:
http://up.avastarco.com/images/5q2hle66hu1a2rah311.png


در واقع در گذشته (دبیرستان و ...) همیشه به صورت ضمنی و زیرپوستی فرض همگنی و همسانگردی رو برای قوانین فیزیک انجام میدادیم و هیچ وقت ذکرش نمی‌کردیم اما در نسبیت خاص ذکر این فرض واجب خواهد بود و به عنوان یک فرض بدیهی وارد نمیشه بلکه به عنوان یک فرض منطقی وارد میشه و میتونه درست نباشه.



و دو تا نکته ی دیگه: اولا قوانین فیزیک از نظر نسبیت خاص دو تاست: 1.قوانین مکانیک 2.قوانین ماکسول

گرانش در این میان یک استثناست که بعدا در نسبیت عام بررسی میشه.

ثانیا فرض همگنی شامل زمان هم میشه، یعنی اگر آزمایش رو الان انجام بدم و دو سال بعد هم انجام بدم نتیجه ی آزمایش یکسان باقی میمونه (این فرض اصلا بدیهی نیست!) به این میگن همگنی نسبت به زمان، همسانگردی نسبت به زمان البته بی معنی هستش!!!

ــــــــــــ
در پست بعد مفهوم نسبیت رو توضیح می‌دم انشالله (می‌خواستم اینجا توضیح بدم اما دیدم طولانی میشه) چون مفهوم نسبیت کاملا مهمه و این که بدونید قوانین نیوتون در واقع نسبیتی هستند. بعد از مفهوم نسبیت، توضیح خواهم داد که چه طور مفهوم نسبیت به همگنی و همسانگردی منجر میشه و همگنی و همسانگردی هم منجر به نسبیت میشه و بعد نسبیت خاصی که اینشتین ارائه میده رو توضیح خواهیم داد و بعدش نتایج ریاضی نظریه و.... همین طوری آروم آروم پیش میریم! در واقع یک دوره ی درسی نسبیت خاص :دی اصلا عجله نکنید :)

خودم نوشت: من عاشق توضیح دادن این جور مسائلم، روحم شاد میشه :))

aysa
07-21-2013, 08:44 PM
خب من الان چند تا سؤال و حرف دارم كه بگم.
اولين اين كه بگيم فضا همگن و همسانگرد هست غلطه.چون اصلا اين طوري نيست.جهان در مقياس هاي بزرگ همگن و همسانگرد هست.مگه اين طور نيست؟
دليل ديگم واسه اين حرفم اينه كه اگه بگيم در كل جهان جواب آزمايش ها يكي هست يه كم عجيب نيست؟چون يه آزمايش درون سياهچاله هيچ وقت جوابش با آزمايشي كه توي خلاء مطلق انجام گرفته يكي نيست.مگه اين كه بگيم در مقياس بزرگ دوباره.
اگه بگيم فضايي كه ما از زمين داريم بهش نگاش ميكنيم اين طوريه همه شك ميكنن.
دوما چرا اين نظريه ها به وجود اومد؟چه لزومي داشت كه جهانو همسانگرد و همگن معرفي كنن؟
اين ايده اولين بار به ذهن كي اومد؟بعد نسبيت خاص يا قبلش؟چون در اين صورت يه كم ماجرا عوض ميشه.تا اونجايي كه من ميدونم همون سال1905بو توسط فريدمن!:)
وسومين مورد در پي نوشته ها يه موردم قبل نسبيت خاص توضيح بدم كه جمله ي
"جهان در مقياس هاي بزرگ همگن وهمسانگرد هست "رو به عنواناصل كيهان شناختي ميشناسن.(Cosmological Principle)چيز عجيبي نيست فقط يه توضيح اضافيه برا حرفاي بالا.درست عين تانژانت تو رياضي كه معلوم نيست به چه دردي ميخوره.آخه به جاش ميتونيم بگيم سينوس بر كسينوس ديگه!!!:دی
------------------------------------------
و در اخر منظور از اين جمله چي بود؟

من عاشق توضیح دادن این جور مسائلم، روحم شاد میشه

Ehsan
07-22-2013, 02:53 PM
خب من الان چند تا سؤال و حرف دارم كه بگم.
اولين اين كه بگيم فضا همگن و همسانگرد هست غلطه.چون اصلا اين طوري نيست.جهان در مقياس هاي بزرگ همگن و همسانگرد هست.مگه اين طور نيست؟
دليل ديگم واسه اين حرفم اينه كه اگه بگيم در كل جهان جواب آزمايش ها يكي هست يه كم عجيب نيست؟چون يه آزمايش درون سياهچاله هيچ وقت جوابش با آزمايشي كه توي خلاء مطلق انجام گرفته يكي نيست.مگه اين كه بگيم در مقياس بزرگ دوباره.


این همگنی و همسانگردی که داریم ازش حرف میزنیم با همگنی و همسانگردی کیهان شناسی فرق داره، این همگنی و همسانگردی خیلی بنیادی‌تره.

در کیهان‌شناسی ما راجع به چگالی کیهان حرف میزنیم و منظورمون از همگنی و همسانگردی، ویژگی جزئی کیهان مثل چگالی و توزیع کهکشان ها و... است، ولی اینجا منظور ما همون طور که گفتم آزمایشات و قوانین فیزیکی هستش، و باز هم تاکید می کنم گرانش در این میان یک استثناست، سیاه چاله و نزدیکی ماده و هر مثالی شبیه به این، وارد کردن گرانش در بحث و نقض کردن ِ این استثنایی هست که ما قائل شدیم. گرانش در بحث نسبیت خاص نمی‌گنجه، پس همگنی و همسانگردی که ما داریم ازش حرف می زنیم غلط نیست، (دست کم تا جایی که به آزمایشات مربوط میشه غلط نیست)
گرانش به نحو زیبایی در نسبیت عام بحث میشه.


اگه بگيم فضايي كه ما از زمين داريم بهش نگاش ميكنيم اين طوريه همه شك ميكنن.

منظورتون رو نفهمیدم.



دوما چرا اين نظريه ها به وجود اومد؟چه لزومي داشت كه جهانو همسانگرد و همگن معرفي كنن؟
اين ايده اولين بار به ذهن كي اومد؟بعد نسبيت خاص يا قبلش؟چون در اين صورت يه كم ماجرا عوض ميشه.تا اونجايي كه من ميدونم همون سال1905بو توسط فريدمن!:)


همگنی و همسانگردی ِ فریدمان باز مربوط به همان کیهان شناسی هستش، از لحاظ تاریخی اینشیتن مفهوم نسبیت رو مقدم بر مفهوم همگنی و همسانگردی مطرح کرد اما بعدا دیدند که این دو مفهوم از همدیگر قابل استنتاج هستند، یعنی اگر فرض کنیم نسبیت برقرار هست، به همگنی و همسانگردی خواهیم رسید و اگر فرض کنیم جهان همگن و همسانگرد هستش، به نسبیت خواهیم رسید.

به درد بخور بودن فرض همگنی و همسانگردی رو هم در استخراج تبدیلات لورنتس خواهیم دید :)

اما این که چه کسی اولین بار به همگنی و همسانگردی فکر کرد، من اطلاعی ندارم.



و در اخر منظور از اين جمله چي بود؟

من عاشق توضیح دادن این جور مسائلم، روحم شاد میشه


توضیح دادن مفاهیم بنیادی فیزیک همیشه برام لذت بخش بوده. این مسائل اصلا بدیهی نیستند، کافیه یه ذره راجع بهشون فکر کنید، چه لزومی داره قانون اهم همه جای کیهان به این صورت جواب بده؟ چه لزومی داره ضریب دی تراوای مغناطیسی در خلا همیشه همین مقدار رو داشته باشه و....

اینا واقعا قشنگ هستند.

Ehsan
08-08-2013, 04:13 PM
2.نسبیت

صرف ِ نظر از هر بار ِ فلسفی که واژه ی نسبیت داره ( و گاها دردسر ساز هم هست) در فیزیک معنای جالبی در پی داره، نسبیت در فیزیک یک اصل محسوب میشه (یعنی بدون استدلال پذیرفته میشه). بیان های مختلفی برای این اصل وجود داره

فرض کنید یک چارچوب ِ مرجع لخت داریم، ناظری درون این چارچوب ِ مرجع ِ لخت هست که با آزمایشاتی، قوانین ِ فیزیک رو به دست میاره و میریزه درون ِ یک جعبه ی جادویی و یک چِک لیست هم از قوانین ِ فیزیکی توی جعبه تهیه می‌کنه و می‌گذاره توی جیبش.

بعد یک چارچوب ِ دیگه داریم که با سرعت ِ ثابت نسبت به چارچوب ِ مرجع ِلخت ِ اولیه در حال ِ حرکت هست، (مثلا یک قطار که با سرعت ِ ثابت داره حرکت می‌کنه) ناظر ِ قبلی جعبه ی جادوییش رو برمی‌داره و می‌ره درون ِ چارچوب ِ متحرک می‌شینه طوری که نسبت به چاچوب ِ در حال ِ حرکت، ساکن باشه (درون ِ قطار میشینه) و بعدش چک لیست رو درمیاره و شروع می‌کنه به آزمودن قوانین ِ فیزیکی توی جعبه، هر قانونی که آزمایشات دوباره تاییدش کنه، یک تیک جلوش می‌زنه و هر قانونی که نقض میشه یک ضربدر جلوش قرار می‌ده.

اصل ِ نسبیت تضمین می‌کنه که در چکلیست ِ ما هیچ ضربدری قرار نخواهد گرفت!

یک آزمایش ِ ملموس‌تر:

فنری با ضریب ِ سختی 10 در واحد ِ SI در نظر بگیرید که یک میله به جرم ِ یک دهم کیلوگرم به سرش وصل شده، من این فنر رو ده سانتیمتر از حالت تعادل دور کرده و بعدش رهاش می‌کنم، اتفاقی که میافته
اینه که فنر با معادله ی 0.1cos(10t) lنوسان میکنه.

همین آزمایش رو درون ِ یک قطار ِ متحرک انجام می‌دم و باز همین نتیجه حاصل میشه، نتیجه: قانون ِ هوک (نیروی فنر) و قانون دوم نیوتون ناورداست و در هر دو دستگاه صادقه و کار می‌کنه.

یک مقاومت ِ 100 اهمی دارم و یک منبع تغذیه ی (باتری) دویست ولتی. روی زمین که ساکن هستم این مقاومت رو به این منبع تغذیه وصل می‌کنم و مشاهده می‌کنم که جریان ِ 2 آمپر از مقاومت عبور می‌کنه، همین آزمایش رو درون ِ قطار انجام میدم و باز هم جریان ِ 2 آمپر عبور می‌کنه.

نتیجه: قانون ِ اهم ناورداست و در هر دو دستگاه ِ ساکن و متحرک صادقه و کار می‌کنه.

اصل ِ نسبیت می‌گه که تمام ِ قوانین ِ فیزیک در هر دو دستگاه ِ ساکن و متحرک یکسان هستند و هیچ تفاوتی بین ِ این دو دستگاه دیده نخواهد شد.

سوال: وقتی از لحاظ ِ فیزیکی نمیشه هیچ تفاوتی بین ِ دستگاه ِ ساکن و دستگاه ِ متحرک قائل شد و توی جعبه ی جادویی ما هیچ قانون ِ فیزیکی نیست که با حرکت کردن تغییر بکنه، پس چه طوری فرق ِ بین ِ سکون و حرکت رو بفهمیم و از کجا بدونیم که دستگاه‌مون ساکنه یا متحرک؟

یعنی اگر یک قطار باشه که روی خط ِ راست حرکت بکنه و یکی از کوپه های قطار، یک آزمایشگاه ِ بسیار مجهز ِ فیزیکی باشه که پنجره ای نداره، آزمایشگر چه طوری می‌تونه بفهمه که قطار ایستاده یا در حال ِ حرکته؟

جواب: هیچ جور! اصل ِ نسبیت همینه: هیچ چارچوب ِ لخت ِ مرجح و ممتازی وجود ندارد و فیزیک برای تمام ِ ناظران ِ لخت یکسانه.

پس مفهومِ نسبیت این شد که قوانین ِ فیزیکی برای چاچوبهای لختی که با سرعت ِ ثابت نسبت به هم حرکت می‌کنند یکسان هستش.

چارچوب های لخت نسبت به قوانین ِ فیزیکی کاملا متقارن هستند و هیچ تفاوتی با هم ندارند.

این استدلال شبیه ِ همین چند خطه:


بنا بر این نمیشه با قوانین ِ ماکسول، اتر رو آشکار کرد، از طرفی توی جعبه ی جادویی ما که پر از قوانین ِ فیزیکه، هیچ قانون ِ دیگری هم نیست که حرکت ما درون ِ اتر رو آشکار کنه => پس من هیچ ابزار ِ فیزیکی برای آشکار کردن ِ اتر ندارم=> پس اتر هیچ تاثیری رو قوانین ِ فیزیک نداره=> اتر هیچ تاثیری روی ما نداره=> از لحاظ ِ فیزیکی اتر مفهومی زاید و اضافیه!!!=> اتر وجود نداره!


در واقع بهتره پست ِ مربوط به اون چند خط رو دوباره بخونید.

ـــــــــــــــــــــــــ ـ
در پست ِ بعدی راجع به این حرف می‌زنیم که چه طور دو مفهوم ِ همگنی و همسانگردی و نسبیت با هم معادل هستند.

ابهامی اگر هست در خدمت‌ام، اگر مفهوم ِ نسبیت رو درک نکردید حتما بپرسید چون خیلی خیلی مهمه، احساس می‌کنم خیلی بد توضیح دادم.

مداد رنگیهام
08-10-2013, 03:53 PM
مثله آسانسور که وقتی با سرعت ثابت بالا یا پایین بره یا ایستاده باشه تفاوتی برای وزن فرد درونش ایجاد نمیشه...
حالا یه سوال چرا سرعت ثابت؟اگر حرکت شتابدار باشه نسبیت چه چیزیو برای معادلات فیزیکی پیش بینی میکنه؟
در مورد آسانسور میدونم که وزن فرد نسبت به حالت اول فرق میکنه ولی در موزد بقیه قوانین چه اتفاقی میوفته؟در مورد قانون اهم؟

ممنون:)

Ehsan
08-10-2013, 08:50 PM
مثله آسانسور که وقتی با سرعت ثابت بالا یا پایین بره یا ایستاده باشه تفاوتی برای وزن فرد درونش ایجاد نمیشه...
حالا یه سوال چرا سرعت ثابت؟اگر حرکت شتابدار باشه نسبیت چه چیزیو برای معادلات فیزیکی پیش بینی میکنه؟
در مورد آسانسور میدونم که وزن فرد نسبت به حالت اول فرق میکنه ولی در موزد بقیه قوانین چه اتفاقی میوفته؟در مورد قانون اهم؟

ممنون:)

درسته، سوالتون بسیار بجاست :)

اولین قانونی که به صورت ِ بدیهی در چارچوب های نالخت نقض میشه قانون ِ اول ِ نیوتون هستش، چرا که اصلا تعریف ِ یک چاچوب ِ لخت صادق بودن ِ قانون ِ اول ِ نیوتون هستش!

وقتی جسمی رها می‌شه باید ساکن بمونه اما در چاچوبهای شتابدار این اتفاق نمی‌افته. مثلا در ماشینی که در حال ِ ترمز هستش (سرعت ِ غیر ثابت یا به عبارتی یک چارچوب ِ شتابدار) شما احساس می‌کنید که نیرویی باعث میشه شما رو به جلو کشیده بشید، در حالی که هیچ خبری از هیچ نیرویی نیست. بنا بر این حتی قانون ِ دوم ِ نیوتون هم نقض میشه چون یک ذره شتاب داره، در حالی که نیرویی بهش وارد نمیشه.

یا مثال ِ دیگه اش همون رابطه ی جرم و وزن در آسانسور های شتابدار هستش.

اما قانون ِ اهم: نقض ِ مستقیم ِ قانون ِ اهم بستگی بعضی مسائل داره، مثلا اگر رسانای دارای مقاومت ِ شما تقریبا یک بعدی باشه (حالت ِ طولی داشته باشه) قانون ِ اهم اختلاف ِ بسیار جزئی پیدا می‌کنه و نسبت ِ ولتاژ به جریان به راستای قرار گیری ِ سیم بستگی پیدا می‌کنه. مثلا در ماشین ِ در حال ِ ترمز، در یک جهت الکترونها نیروی الکتریکی بیشتری لازم دارند تا حرکت کنند اما در جهت ِ مخالف با نیروی الکتریکی صفر هم می‌تونن حرکت کنند چون شتاب ِ مرجع حرکتشون میده، در جهت ِ عمود، اتفاقی نمی‌افته و قانون ِ اهم مثل ِ قبل هستش و به طور ِ کلی دستگاه ناهمسانگرد میشه. اما این اختلاف خیلی خیلی خیلی جزئی و ضعیف هستش.

قوانین ِ ماکسول در چارچوبهای شتابدار صحیح نیستند چون پیشبینی می‌کنند که نور مستقیم حرکت کنه اما در چارچوب ِ شتابدار نور خمیده حرکت می‌کنه اما این مورد هم بسیار ضعیف آشکار میشه.

به طور ِ کلی بررسی قوانین ِ فیزیک در چارچوبهای شتابدار به نسبیت عام مربوط میشه که سالها بعد از نسبیت ِ خاص به وجود اومد :)

سوال ِ بجای دیگه اینه که چرا چارچوبهای بیشتاب رده ی ممتازی از چارچوبها رو تشکیل می‌دن و به عبارتی چرا ما نمی‌تونیم سرعت ِ دستگاه رو اندازه بگیریم و نمی‌تونیم بین ِ سکون و حرکت تفاوتی قائل بشیم اما می‌تونیم شتاب رو اندازه بگیریم و بین ِ شتابدار بودن و بیشتابی تفاوت قائل بشیم! جوابش به نسبیت ِ عام، نظریات ِ ماخ و کمی فلسفه مربوطه و البته کامل نیست :دی

aysa
08-10-2013, 11:05 PM
2.

ابهامی اگر هست در خدمت‌ام، اگر مفهوم ِ نسبیت رو درک نکردید حتما بپرسید چون خیلی خیلی مهمه، احساس می‌کنم خیلی بد توضیح دادم.
ابهام و سؤال كه زياد هست.حالا مونده شما چه قد حوصله ي جواب دادن داشته باشين:دی
اين كه تو پست بالايي گفتين[quot
در حال ِ ترمز، در یک جهت الکترونها نیروی الکتریکی بیشتری لازم دارند تا حرکت کنند اما در جهت ِ مخالف با نیروی الکتریکی صفر هم می‌تونن حرکت کنند چون شتاب ِ مرجع حرکتشون میده، در جهت ِ عمود، اتفاقی نمی‌افته و قانون ِ اهم مثل ِ قبل هستش و به طور ِ کلی دستگاه ناهمسانگرد میشه. اما این اختلاف خیلی خیلی خیلی جزئی و ضعیف هستش.
من نفهميدم.شما ميگين كه يعني قانون اهم توي همه ي دستگاه ها به يه مدله؟منظورم همون ثابته؟
اگه اين طوريه پس چرا گفتين نا همسانگرده؟
هر وقت كه بگيم يه جسمي شتابداره حتما اون جسم ديگه نا لخته؟مثلا يه جسم شتابداري درون يه قطار كه با شتاب ثايت در حال حركته حركت مي كنه.الان واسه يه ناظر كه خودش شتابداره اين جسم و قطار چه حالتي دارن؟اگه طرف شتاب نداشته باشه چي؟ببخشيد اگه جوابشو گفته باشين .چون در اين صورت من نفهميدم.
3.من از تعريف هاتون به اين نتيجه رسيدم:وقتي ميگيم نسبيت در واقع اينو قبول كرديم كه همسانگرده.و همم چنين همگن.اولي كه معلومه دومي واسه اين كه ما اونو نسبت به يه دستگاه لخت ميسنجيم و درنتيجه همه جا يه مفهموه!پس اين سه تا موضوع يكين.درسته؟
4:اين نسبيت درست همون حرفايي شد كه دانشمند هاي قبلي هم گفتين.آيا اين وسط چيزي تغيير كرده ويا نه فقط يه اسم جديد واسش گذاشتن؟اصلا چه لزومي داشت كه همون موضع هاي قبليرو بگن.چيش متفاوته؟بعد اين وسط دستگاه نا لخت چي ميشه؟
---------------------------------------------------------
خدارو شكر اين جا ميشه بيشتر از يه سؤال پرسيد:دی :))

مداد رنگیهام
08-11-2013, 04:27 PM
در مورد قانون اهم من اینجوری فهمیدم که چون اختلافی که ایجاد میشه خیلی کمه و مثله مثال تغییر وزن در آسانسوری که حرکت شتابدار داره محسوس نیست میشه گفت قانون اهم باز هم صادق میمونه.
در مورد اینکه اگر دو متحرک با یک شتاب یکسان از لحاظ اندازه و جهت حرکت کنن من اینجوری استدلال کردم که این دو متحرک در واقع نسبت به هم شتابی ندارن و بسته به v0 شون انگار دارن با سرعت ثابت نسبت به هم حرکت میکنن و اگه سرعت اولیشونم یکسان باشه اینگونه به نظر میررسه که انگار نسبت به هم ساکنن بنابر این باید قوانین فیزیکی جواب های یکسانی برای هر دو پیشبینی کنه در واقع نوع حرکت یه متحرک به تنهایی تعیین کننده نیست بلکه مهم اینه که نسبت به هم چه نوع حرکتی داشته باشن و این همون نسبیته!:)

Ehsan
08-11-2013, 04:51 PM
ابهام و سؤال كه زياد هست.حالا مونده شما چه قد حوصله ي جواب دادن داشته باشين:دی

ما حوصله مون زیاده به شرطی که سوالاتونو یه جوری بپرسید که بشه فهمید چی پرسیدید :))



اين كه تو پست بالايي گفتين
من نفهميدم.شما ميگين كه يعني قانون اهم توي همه ي دستگاه ها به يه مدله؟منظورم همون ثابته؟
اگه اين طوريه پس چرا گفتين نا همسانگرده؟
---------------------------------------------------------
خدارو شكر اين جا ميشه بيشتر از يه سؤال پرسيد:دی :))



توی همه ی دستگاه‌های لَخت ثابته نه هر دستگاهی، در دستگاهای نالخت این قانون با ناهمسانگردی مواجه میشه و در راستاهای مختلف مقاومت ِ مختلف اندازه گیری میشه اما در کل ناهمسانگردی و ناهمگنی اثرات ِ الکترومغناطیسی در دستگاهای نالخت بسیار بسیار سخت آشکار میشه و اصلا واضح نیست.


هر وقت كه بگيم يه جسمي شتابداره حتما اون جسم ديگه نا لخته؟مثلا يه جسم شتابداري درون يه قطار كه با شتاب ثايت در حال حركته حركت مي كنه.الان واسه يه ناظر كه خودش شتابداره اين جسم و قطار چه حالتي دارن؟اگه طرف شتاب نداشته باشه چي؟ببخشيد اگه جوابشو گفته باشين .چون در اين صورت من نفهميدم.

شتاب رو به چارچوب نسبت می‌دیم نه به جسم، چارچوبی که شتابدار باشه و قانون ِ اول ِ نیوتون توش کار نکنه بهش می‌گیم نالخت.
بستگی به جهت و اندازه های شتابهای ناظر داره، اگر جهتشون ربطی به هم نداشته باشه در حالت ِ کلی می‌تونه حرکت ِ همه چیز شتابدار باشه اما اگر جهت و اندازه ی شتابها از دید ِ یک ناظر ِ لخت ِ بیرونی یکسان باشه، اون موقع در چارچوب ِ نالخت اجسام بیشتاب هستند.


.من از تعريف هاتون به اين نتيجه رسيدم:وقتي ميگيم نسبيت در واقع اينو قبول كرديم كه همسانگرده.و همم چنين همگن.اولي كه معلومه دومي واسه اين كه ما اونو نسبت به يه دستگاه لخت ميسنجيم و درنتيجه همه جا يه مفهموه!پس اين سه تا موضوع يكين.درسته؟

بعدا نشون میدیم که نسبیت می‌تونه به همگنی و همسانگردی منجر بشه و همگنی و همسانگردی هم به نسبیت منجر بشه، در واقع یک مفهوم نیستند بلکه هم ارز هستند و میشه از هم نتیجه گرفتشون.

مثل ِ «برابر بودن ِ دو ضلع و زاویه ی بینشان در مثلث الف و ب» و «برابر بودن ِ سه ضلع مثلث الف و ب» که هر دو با هم هم ارز هستند اما یکی نیستند.

کمی صبر داشته باشید :دی



4:اين نسبيت درست همون حرفايي شد كه دانشمند هاي قبلي هم گفتين.آيا اين وسط چيزي تغيير كرده ويا نه فقط يه اسم جديد واسش گذاشتن؟اصلا چه لزومي داشت كه همون موضع هاي قبليرو بگن.چيش متفاوته؟بعد اين وسط دستگاه نا لخت چي ميشه؟

اگه یه کمی دیگه صبر کنید(!) نشون میدیم که تنها با فرض ِ همگنی و همسانگردی و بدون ِ دخالت ِ اصل ِ ثابت بودن ِ سرعت ِ نور، میشه شکل ِ تبدیلاتِ لورنتس رو استخراج کرد و تبدیلات ِ گالیله یک حالت ِ حدی از تبدلیات ِ لورنتس خواهد بود، این دستیابی به فرم ِ تبدیلات-اون هم با فرضهایی بسیار ساده- قدرت ِ این فرضها رو نشان میده، تنها با یک فرض ِ اضافی هستش که حرفهای کلاسیک تبدیل میشه به نسبیت ِ خاص.

بررسی دستگاه ِ نالخت همون طور که گفتم مربوط میشه به نسبیت ِ عام :)

Ehsan
08-17-2013, 09:00 PM
لازمه راجع به این مفهوم کمی بیشتر صحبت کنیم

معنای همگنی رو این شرح دادیم که قوانین ِ فیزیک برای تمام ِ نقاط ِ فضا یکسانند،برای همگنی می‌شه معنای دیگری هم ذکر کرد:

فرقی نمی‌کند که مبدا دستگاه ِ مختصات را کجا انتخاب کنیم، هیچ نقطه ی مرجح و ممتازی وجود ندارد

چرا فرقی نداره؟ چون هیچ قانون ِ فیزیکی وجود نداره که بشه تشخیص داد ما کجای فضا هستیم!

برای همسانگردی هم میشه حرفهای مشابهی زد:

گفتیم همسانگردی یعنی آزمایش های فیزیکی نسبت به چرخیدن تغییری در نتیجه‌شون رخ نمیده، وقتی چرخش معنای برای آزمایشهای فیزیکی نداره-به عبارتی تاثیر فیزیکی نداره- یعنی نمیشه مشخص کرد که تو کدام جهت از فضا هستیم پس:

فرقی نمی‌کند که جهت ِ محور ها را به کدام سو فرض کنیم، هیچ جهت ِ مرجح و ممتازی وجود ندارد

از آنجایی که یعرف الاشیا بالاضدادها (:دی یعنی موجودات با متضادهاشون شناخته میشن) دو سه تا مثال از فضاهای ناهمگن و ناهمسانگرد می‌زنم تا مفهومش بهتر جا بی‌افته:

فرض کنید یه سطح دارید که هر چه قدر پیش‌روی می‌کنید، لیزتر میشه:
(سطح=فضا، آزمایش= احتمال ِ لیز خوردن (اندازه گیری ضریب اصطکاک))

http://up.avastarco.com/images/4ysgyr2o5202h9n8yrfp.png

خوب با آزمایش ِ فیزیکی می‌شه فهمید که این فضا ناهمگنه، کافیه جایی که هستید ضریب اصطکاک رو اندازه بگیرید اگه زود لیز خوردید سمت ِ چپ هستید اگر نسبتا پایدار هستید و به زور لیز می‌خورید، سمت ِ راست هستید. به عبارتی برای قوانین ِ فیزیکی فرق داره که مبدا رو کجا می‌گذارید (وضعیت می‌تونه از این هم بدتر باشه)

تناقض: آیا این فضا همسانگرده؟

1) این فضا همچنان همسانگرده، به این معنا که من چه در جهت ِ y لیز بخورم و چه در جهت ِ x ضریب ِ اصطکاک تفاوتی نمی‌کنه، با تغییر ِ جهت فضا، آزمایش ِ فیزیکی به طور ِ صریح عوض نمیشه و شکلش ثابته

2)همسانگرد نیست چون جهت ِ مرجحی مثل ِ x وجود داره.

جواب: نمی‌دونم! ولی می‌پرسم و فکر می‌کنم، شما هم فکر کنید :دی (یک قضیه ای هست به اسم شور که می‌گه فضایی که در هر نقطه همسانگرده، حتما همگنه (عکس ِ قضیه درست نیست) به استناد ِ این قضیه می‌شه گفت که فضا قطعا ناهمسانگرده، چون اگر در هر نقطه همسانگرد بود حتما همگن هم بود، شاید یک جواب ِ دیگه از این قرار باشه که اندازه گیری ِ تغییر ِ ضریب ِ اصطکاک یک «آزمایش با محتوای فیزیکی (قابل اندازه گیری)» هستش که با این آزمایش، فضا ناهمسانگرده، به هر حال در هر دو استدلال فضا ناهمسانگرده، به عبارتی تناقض ِ عبارت ِ 1 اینه که آزمایش ِ دیگری هم هست که نسبت به فضا ناهمسانگرده)

اما یک مثال ِ مشابه برای فضای همگن اما ناهمسانگرد:

فرض کنید که باز یک سطح داریم، اما روی این سطح هر کجا که با‌یستیم در راستای محور چپ-راست فضا به شدت لیزه، اما در راستای جلو-عقب فضا کاملا عادیه:

http://up.avastarco.com/images/vibbyxbbhsla80nes4g.png

این فضا ناهمسانگرده، به این معنا که من می‌تونم با آزمایش ِ فیزیکی بفهمم که کدوم جهت x هستش و کدوم جهت y و انتخاب ِ جهت کاملا شکل ِ قوانین ِ فیزیک رو عوض می‌کنه اما این فضا کاملا همگنه، یعنی من در انتخاب ِ مبدا مختصات آزاد هستم و با آزمایش نمی‌تونم بفهمم که کجای صفحه قرار دارم.


تذکر: طبق ِ اصل ِ همگنی و همسانگردی هر چارچوب ِ لَختی که به هر طریقی بیابیم، همگن و همسانگرد است.

ـــــــــــــــــــ
می‌خواستم لِم دستگاهِ میانی رو هم بگم اما دیگه طولانی میشد :دی

بعد از لِم دستگاه میانی، میشه به چیزی که می‌خواهیم بپردازیم: همگنی و همسانگردی<=>نسبیت

solh
08-18-2013, 06:10 PM
خوب در مورد اصطکاک که همچنان در مورد قانون همگن و همسانگرده ، قطعا وابسته به شرایط داده های مسئله فقط تفاوت میکنه اما قوانین همچنان همسانگرد هستند . اگر بخایم بگیم که ناهمسانگرده قاعدتا باید این دو آزمایش در دو قاعده ی متفاوت فیزیکی صدق کنن در حالی که در هر دو از یک ضابطه استفاده میشه و نتیجه ی نهایی مثبته . مگه نباید به این ترتیب همسانگردی رو قبول داشت؟

قطعا کسی به ذهنش نیومده بوده که معنای همسانگردی باید با نتایج آزمایش ها مورد شک قرار بگیره ، حد اقل تصور ذهنی من اینه که کوانتوم به وضوح اشاره میکنه که اتفاقا نباید هرگز آزمایش ها دوباره تکرار بشن . فکر میکنم که باید همسانگردی رو بیشتر روی روابط ریاضی شناخت . یعنی تا وقتی که معادلات از لحاظ کارکرد و نتایج مشاهده شده به خوبی کار میکنند و نیازی به دگرگونی در آن ها نیست همچنان فضا همسانگرد هست .

Ehsan
08-20-2013, 01:25 PM
خوب در مورد اصطکاک که همچنان در مورد قانون همگن و همسانگرده ، قطعا وابسته به شرایط داده های مسئله فقط تفاوت میکنه اما قوانین همچنان همسانگرد هستند . اگر بخایم بگیم که ناهمسانگرده قاعدتا باید این دو آزمایش در دو قاعده ی متفاوت فیزیکی صدق کنن در حالی که در هر دو از یک ضابطه استفاده میشه و نتیجه ی نهایی مثبته . مگه نباید به این ترتیب همسانگردی رو قبول داشت؟

بله، قوانین ِ پایه همگن همسانگرد هستند و مسلما این مثالها تناقضی با اصل ِ همگنی و همسانگردی ندارند گرچه می‌تونند در واقعیت وجود داشته باشند ( و دارند) من فقط گفتم اگر فرض کنیم قانون ِ پایه ی ما اصطکاک باشه این اتفاق میافته، و الا «فیزیک» همچنان همگن و همسانگرده.


قطعا کسی به ذهنش نیومده بوده که معنای همسانگردی باید با نتایج آزمایش ها مورد شک قرار بگیره ، حد اقل تصور ذهنی من اینه که کوانتوم به وضوح اشاره میکنه که اتفاقا نباید هرگز آزمایش ها دوباره تکرار بشن . فکر میکنم که باید همسانگردی رو بیشتر روی روابط ریاضی شناخت . یعنی تا وقتی که معادلات از لحاظ کارکرد و نتایج مشاهده شده به خوبی کار میکنند و نیازی به دگرگونی در آن ها نیست همچنان فضا همسانگرد هست .


متاسفانه درست نفهمیدم منظورتون چیه، مخصوصا وقتی از کوانتم استدلال کردید (اونجا رواصلا نفهمیدم) اما نکته اینه که همگنی و همسانگردی معناش اتفاقا کاملا ریاضیه، چون منظور ِ ما از ناوردایی نسبت به چرخش و انتقال (همان همگنی و همسانگردی) ناوردایی ِ معادلات ِ پایه ی فیزیک (مثل ِ قوانین ِ ماکسول) هستش (حالا سر ِ این که «فیزیک» دقیقا یعنی چی یه ذره بحث خواهم کرد، مخصوصا وقتی بخواهیم اصول ِ موضوعه ی نسبیت رو مطرح کنیم)

smhm
08-22-2013, 08:50 AM
ببخشید وسط بحث میپرم. یک سوال همیشه تو ذهنم هست که ما چطوری می تونیم سرعت را احساس کنیم؟
فرض کنید شخصی درون یک اتومبیل نشسته در حالیکه منظره بیرون را نمی تونه ببینه و اتومبیل هم کاملا روان و بدون لرزش هست. حالا این شخص درون اتومبیل چگونه می تواند بفهمد که آیا در حال حرکت هست یا نه و اگر در حال حرکته با چه سرعتی؟؟
ناظر همواره نسبت به اتومبیل، ساکن هست پس فرقی نمی کنه که اتومبیل با چه سرعتی حرکت کنه. چه با سرعت 10 کیلومتر در ساعت و چه با سرعت 1000 کیلومتر در ساعت و چه با سرعت 10000 تا ... برای ناظر هیچ تفاوتی ندارد و بالتبع هیچ احساسی هم نسبت به سرعت ندارد.

حالا چطوری هست که در فیزیک نسبیت می گوییم اگر این ناظر به سرعت نور نزدیک شود تغییراتی در جرم بوجود میاد و اثرات ناشی از این سرعت را احساس می کند؟!
اصلا این سرعت مربوط به اتومبیل هست که نسبت به زمین در حال حرکت هست اما ناظر درون اتومبیل که همواره ساکن هست و سرعتش صفره. پس چرا این سرعت را به ناظر هم نسبت می دهیم؟ این سرعت فقط مربوط به اتومبیله (اون هم نسبت به زمین) نه ناظر. تازه زمین هم که در فضا بی حرکت نیست. پس سرعت واقعی اتومبیل هم معلوم نیست. از کجا می تونیم بفهمیم اتومبیل داره به سرعت نور نزدیک میشه؟؟

بنابراین سرعت همیشه یک مفهوم نسبی دارد و هیچوقت معنای واقعی نخواهد داشت. در حالیکه در نسبیت خاص برای سرعت نور یک سرعت واقعی و ذاتی تصور میشود!! چرا باید برای سرعت نور چنین تصوری داشته باشیم و چرا سرعت نور نسبت به چیزی نیست یا نسبت به هرچیزی هست؟؟ از لحاظ منطقی مفهوم سرعت بدون نسبت اصلا بی معناست.

solh
08-22-2013, 09:40 AM
ببخشید وسط بحث میپرم. یک سوال همیشه تو ذهنم هست که ما چطوری می تونیم سرعت را احساس کنیم؟
فرض کنید شخصی درون یک اتومبیل نشسته در حالیکه منظره بیرون را نمی تونه ببینه و اتومبیل هم کاملا روان و بدون لرزش هست. حالا این شخص درون اتومبیل چگونه می تواند بفهمد که آیا در حال حرکت هست یا نه و اگر در حال حرکته با چه سرعتی؟؟
ناظر همواره نسبت به اتومبیل، ساکن هست پس فرقی نمی کنه که اتومبیل با چه سرعتی حرکت کنه. چه با سرعت 10 کیلومتر در ساعت و چه با سرعت 1000 کیلومتر در ساعت و چه با سرعت 10000 تا ... برای ناظر هیچ تفاوتی ندارد و بالتبع هیچ احساسی هم نسبت به سرعت ندارد.

حالا چطوری هست که در فیزیک نسبیت می گوییم اگر این ناظر به سرعت نور نزدیک شود تغییراتی در جرم بوجود میاد و اثرات ناشی از این سرعت را احساس می کند؟!
اصلا این سرعت مربوط به اتومبیل هست که نسبت به زمین در حال حرکت هست اما ناظر درون اتومبیل که همواره ساکن هست و سرعتش صفره. پس چرا این سرعت را به ناظر هم نسبت می دهیم؟ این سرعت فقط مربوط به اتومبیله (اون هم نسبت به زمین) نه ناظر. تازه زمین هم که در فضا بی حرکت نیست. پس سرعت واقعی اتومبیل هم معلوم نیست. از کجا می تونیم بفهمیم اتومبیل داره به سرعت نور نزدیک میشه؟؟

بنابراین سرعت همیشه یک مفهوم نسبی دارد و هیچوقت معنای واقعی نخواهد داشت. در حالیکه در نسبیت خاص برای سرعت نور یک سرعت واقعی و ذاتی تصور میشود!! چرا باید برای سرعت نور چنین تصوری داشته باشیم و چرا سرعت نور نسبت به چیزی نیست یا نسبت به هرچیزی هست؟؟ از لحاظ منطقی مفهوم سرعت بدون نسبت اصلا بی معناست.

در واقع ترازویی که اون آدم داره تغییر جرم رو نشون نمیده ، اون شخص همچنان سرعت نور رو همون مقدار اولیه اندازه میگیره و طبق معادلات کلاسیک نسبت به نور ساکنه . این افزایش جرم از دید ناظره که رخ میده و در واقع با وجود ناظر سومی که داره این شخص رو میبینه که با سرعتی نسبت به خودش حرکت میکنه میشه افزایش جرم رو توسط اون اندازه گیری کرد .

ثابت بودن سرعت نور یه اصل فیزیکه . این صدق میکنه مستقل از ناظر ، یعنی بدون اینکه اهمیتی بدیم ناظر با چه سرعتی حرکت میکنه. در واقع همونطور که قبلا آقا احسان توضیح میدادن در این تلاش هستیم که بگیم سرعت ناظر رسما اهمیت فیزیکی نداره چون راهی برای فهمیدنش اصلا وجود نداره (ناظر سوم وجود نداره !) .این طور فکر میکنم .

smhm
08-24-2013, 10:36 AM
سرعت ناظر رسما اهمیت فیزیکی نداره چون راهی برای فهمیدنش اصلا وجود نداره
پس با این حساب سرعت نور هم یک سرعت واقعی و ذاتی نیست بلکه نسبی است (نسبت به ناظر)
و چون ناظر هیچ راهی برای درک سرعت واقعی خودش نداره میشه نتیجه گرفت که سرعت کلا یک پدیده خنثی و بی اثر است و سرعت یک جسم هرچقدر هم که باشد هیچ تأثیری در ذات ماده و ذرات آن ندارد.!

Ehsan
08-24-2013, 03:11 PM
پس با این حساب سرعت نور هم یک سرعت واقعی و ذاتی نیست بلکه نسبی است (نسبت به ناظر)
و چون ناظر هیچ راهی برای درک سرعت واقعی خودش نداره میشه نتیجه گرفت که سرعت کلا یک پدیده خنثی و بی اثر است و سرعت یک جسم هرچقدر هم که باشد هیچ تأثیری در ذات ماده و ذرات آن ندارد.!


سرعت مستقل از این که سرعتِ چی باشه، یک امرِ مطلق نیست و نسبی هستش. سرعتِ نور هم به این معنا که بالاخره «سرعت» هستش و باید نسبت به چیزی سنجیده بشه نسبی محسوب میشه، اما از طرفی سرعتِ نور یک کمیت ِ بنیادین هستش به این معنا که در هر دستگاهی که اندازه گیری بشه مقدارش ثابته و به همین دلیل یک خطکشِ بنیادین برای جهان حساب میشه و به نحوی با ارزشه.

تو فیزیک معمولا راجع به «ذات» حرف نمی‌زنند و بیانِ معمول و عُرفی گزاره ای که فرمودید اینه که سرعتِ دستگاه ِ مختصات هیچ اثرِی روی قوانین ِ فیزیک (مشاهده پذیرها) نداره و قوانین ِ فیزیک مستقل از سرعتِ ناظر، یکسان و بلاتغییر هستند و این همون اصل ِ نسبیت هستش که در این پست (http://forum.avastarco.com/forum/showthread.php?1443-%D9%81%D9%8A%D8%B2%D9%8A%D9%83-%D8%AC%D8%AF%D9%8A%D8%AF&p=66383&viewfull=1#post66383) بهش اشاره شده :)

smhm
08-26-2013, 11:49 AM
سرعتِ نور هم به این معنا که بالاخره «سرعت» هستش و باید نسبت به چیزی سنجیده بشه نسبی محسوب میشه، اما از طرفی سرعتِ نور یک کمیت ِ بنیادین هستش به این معنا که در هر دستگاهی که اندازه گیری بشه مقدارش ثابته و به همین دلیل یک خطکشِ بنیادین برای جهان حساب میشه و به نحوی با ارزشه.
خوب این مطلب یک مقدار تناقض داره اگر سرعت نور نسبی است پس چرا مستقل از سرعت ناظر هست؟! نسبی بودن با مستقل بودن تضاد دارد.
در همه سرعتهایی که کمتر از نور هست و بطور روزمره با آنها مواجهیم نسبی بودن کاملا صدق می کند یعنی اگر من با سرعت 20 عقب بروم و شما با سرعت 40 جلو بروید نهایتا من سرعت شما را 60 اندازه گیری می کنم. (مفهوم نسبی بودن یعنی همین) اما در مورد سرعت نور این مسئله صدق نمی کند و نور همواره با سرعت ثابت اندازه گیری می شود. این یعنی نقض نسبیت.! این مسئله نشون میده که سرعت نور یک ویژگی متفاوتی نسبت به بقیه سرعتها داره. در حالیکه سرعت، سرعته چرا باید یک حالت خاصی از سرعت وجود داشته باشه؟
و یک سوال دیگه اینکه اگر من با سرعت 150 هزار کیلومتر در ثانیه عقب بروم و شما 150 هزار کیلومتر در ثانیه جلو بروید آیا من شما را با سرعت نور می بینم؟ در اینصورت آیا شما نباید به نور تبدیل شوید؟ حالا اگر سرعت شما کمی بیشتر شود آیا من شما را با سرعتی بیشتر از نور مشاهده خواهم کرد؟

Ehsan
08-26-2013, 05:39 PM
خوب این مطلب یک مقدار تناقض داره اگر سرعت نور نسبی است پس چرا مستقل از سرعت ناظر هست؟! نسبی بودن با مستقل بودن تضاد دارد.
در همه سرعتهایی که کمتر از نور هست و بطور روزمره با آنها مواجهیم نسبی بودن کاملا صدق می کند یعنی اگر من با سرعت 20 عقب بروم و شما با سرعت 40 جلو بروید نهایتا من سرعت شما را 60 اندازه گیری می کنم. (مفهوم نسبی بودن یعنی همین) اما در مورد سرعت نور این مسئله صدق نمی کند و نور همواره با سرعت ثابت اندازه گیری می شود. این یعنی نقض نسبیت.! این مسئله نشون میده که سرعت نور یک ویژگی متفاوتی نسبت به بقیه سرعتها داره. در حالیکه سرعت، سرعته چرا باید یک حالت خاصی از سرعت وجود داشته باشه؟
و یک سوال دیگه اینکه اگر من با سرعت 150 هزار کیلومتر در ثانیه عقب بروم و شما 150 هزار کیلومتر در ثانیه جلو بروید آیا من شما را با سرعت نور می بینم؟ در اینصورت آیا شما نباید به نور تبدیل شوید؟ حالا اگر سرعت شما کمی بیشتر شود آیا من شما را با سرعتی بیشتر از نور مشاهده خواهم کرد؟

تضادی در کار نیست، معنای نسبی بودن کمی دردسر ساز شده، سرعت همیشه نسبت به سکون سنجیده میشه وقتی در دستگاهی اعلام می‌کنیم که سرعتِ فلان شی 100 کیلومتر در ثانیه است منظورمون اینه که در دستگاه ِ سکونِ ما این جسم در هر ثانیه 100 متر جا به جا میشه. نه این که نسبت به یک جسم ِ متحرکِ دیگه بگیم 100 متر در ثانیه، البته میشه این کار رو کرد (یعنی بگیم نسبت به یک جسم ِ متحرک ِ دیگه سنجیده میشه) اما باید دقت کنیم وقتی این بیان رو استفاده می کنیم از یک اشتباه جلوگیری کنیم، فرض کنید میگیم سرعتِ جسم ِ A نسب به جسم ِ B در دستگاه ِ سکونِ ما (که الزاما دستگاهِ سکونِ a یا b نیستش) Vab هستش باید دقت کنیم این با سرعتِ جسمِ A در دستگاه ِ سکونِ B تفاوت داره یعنی:

«Vab» مساوی نیست با «سرعتِ A در دستگاه ِ سکونِ B»

حالا فرض کنید که دو تا ذره داریم که در یک آزمایشگاه یکی با سرعتِ 250000 کیلومتر در ثانیه به سمت ِ چپ میره و یکی دیگه با سرعتِ 250000 کیلومتر بر ثانیه به سمت ِ راست میره، سرعت ِ نسبی به اون معنایی که شما گفتید برای این دو ذره میشه 500000 هزار کیلومتر بر ثانیه یعنی خیلی بیشتر از سرعتِ نور اما آیا این نقض ِ نسبیته؟ نه!

حدِ سرعت ِ نور برای سرعتِ یک ذره در چاچوبِ سکون هستش، قاعده ای هست به اسمِ جمع ِ سرعت های نسبیتی که می‌گه برای دو جسم که در راستای متضاد حرکت می‌کنند سرعتشون از دیدِ هم دیگر از این رابطه حساب میشه:

http://upload.wikimedia.org/math/2/0/3/2035aab1ba5af2e1ff296512b6a57779.png

(به زودی اثباتش خواهیم کرد) این قاعده میگه سرعتِ این دو تا جسم در چارچوبِ سکون ِ یک دیگر برابر خواهد بود با 0.98 برابر ِ سرعت ِ نور یا حدودا 295000 کیلومتر بر ثانیه.

این رابطه در سرعت های بسیار کم تبدیل میشه به اون چیزی که شما برای سرعتِ نسبی فرمودید
ــــــــــــــــ
فعلا بحث ِ ما به اینجاها نرسیده لطفا مطالبِ قبلی رو مطالعه کنید و صبر کنید تا بحثها برسه و بعد من در خدمتم :)

Ehsan
08-26-2013, 08:48 PM
از نسبیت چه طور به همگنی و همسانگردی می‌رسیم؟

ساده است

اول فرض می‌کنیم نسبیت برقراره.

سه چارچوب ِ پایین رو در نظر بگیرید:


http://up.avastarco.com/images/k0it6vnempvsfllq6edp.png

ما در دستگاهِ s1 ساکن نشستیم، در هر کدوم از دو دستگاه ِ دیگه یک آزمایش داره انجام میشه E و 'E که در s2 و s3 ساکن هستند، چون نسبیت برقرار هست پس من مطمئن هستم که نتیجه ی دو آزمایش یکی خواهد بود و چون می‌تونم s2 رو به میزانِ دلخواه جا به جا کنم و s3 رو به میزان ِ دلخواه بچرخونم و طبق ِ نسبیت نتیجه ی آزمایش نباید عوض بشه پس قوانین ِ فیزیک و آزمایشها نسبت به چرخش و انتقال ناوردا هستند بنا بر این هر دستگاه ِ لختی همگن و همسانگرد هستش :دی

اما برعکسش چه طور؟ قبل از اثبات ِ برعکسش باید لم ِ دستگاه ِ میانی رو معرفی کنم:

لم دستگاه ِ میانی (یا لم ِ چارچوب ِ میانی)

فرض کنید دو تا چارچوبِ لخت داریم، یکی s و دیگری 's ، چارچوبِ 's با سرعتِ v نسبت به s در حالِ حرکت هستش، لم ِ دستگاهِ میانی میگه که وجود دارد چارچوبی مثلِ "s که مشاهده می‌کنه s و 's با سرعتِ یکسان به دو جهتِ مخالف در حالِ حرکتند.


http://up.avastarco.com/images/kp0ij3jwoo2shwvx0852.png

اثبات:

فرض کنید دستگاهِ r رو داریم که می‌تونیم سرعتش رو تغییر بدیم، w ، یعنی سرعتِ دستگاه ِ r از دیدِ چارچوب ِ s می‌تونه از صفر تا v تغییر کنه، وقتی سرعتِ r صفر باشه r دقیقا منطبق بر s هستش و وقتی برابر با v باشه r دقیقا منطبق بر s’ هستش.

دو تابع ِ پایین رو در نظر بگیرید:

f(w) l سرعتِ دستگاه ِ s از دیدِ ناظر ِ ساکن درون ِ r هستش
g(w) l سرعت ِ دستگاه ِ 's از دید ِ ناظر ِ ساکن درون ِ r

میدونیم که g(w) l به صورتِ پیوسته از v تا صفر تغییر می‌کنه یعنی g(0)=v و g(v)=0 ، اما f(w) l از صفر تا یک سرعت ِ مثبتی مثل ِ u تغییر می‌کنه یعنی f(0)=0 و f(v)=u حالا تابعِ اختلاف ِ سرعت ِ بین ِ دو دستگاه میشه D(w)=f-g

تابع ِ D(0)=-v و D(w)=+u ، ما دنبال ِ اثبات ِ وجود ِ سرعتی مثل ِ a هستیم که در اون D(a)=0 باشه (یعنی ناظر سرعت ِ یکسان و مخالف الجهتی از s و 's آشکار کنه. چون D پیوسته است و از منفی تا مثبت تغییر میکنه پس حتما دستِ کم یک جا (سرعت) وجود داره که D حتما تغییر علامت میده (صفر میشه) بنا بر این دستگاه ِ "s وجود داره :دی

این لم با نام midframe lemma معروفه.

حالا با این لم میشه برعکس ِ قضیه رو هم اثبات کرد اما قبل از اثبات یه کمی روش فکر کنید :دی نظری نیست؟

ــــــــــــــــــ

ابهامی هست اگر در خدمتم :)

Ehsan
08-29-2013, 11:25 AM
حالا چه طور از همگنی و همسانگردی دستگاه‌های لخت، نتیجه بگیریم که اصلِ نسبیت صحصیحه؟


حالا فرض کنید دو تا دستگاهِ لختِ دلخواه داریم که می‌خواهیم اثبات کنیم فیزیکِ یکسانی بر اونها حاکمه، طبقِ لمِ دستگاهِ میانی "s ی وجود داره که سرعتِ هر دو دستگاه رو یکسان می‌بینیه. فرض کنید رفتیم توی "s و اونجا دو تا آزمایش در جهتِ مخالف ِ هم رو رصد می‌کنیم که هر دوی آزمایش ها در دستگاه‌های اول و دوم قرار داره، یعنی آزمایش های E و 'E


http://up.avastarco.com/images/yglssjnggxeapfwa4v0.png


حالا طبقِ همگنی و همسانگردی ِ دستگاه‌های لخت، دستگاهِ "s هم همگن و همسانگرده، در نتیجه دو آزمایش ِ E و 'E باید نتیجه ی دقیقا یکسانی داشته باشند، نتیجه:

فیزیک برای تمامِ دستگاه‌های لخت یکسان است

نکته: اگر فرض کنیم که تنها یک دستگاهِ همگن و همسانگرد وجود داره و بعد بخواهیم برای بقیه ی چارچوبها (لخت یا غیرِ لخت) اظهار ِ نظر داشته باشیم، می‌تونیم به صورتِ منطقی همگنی بقیه ی چاچوبها رو اثبات کنیم، اما همسانگردی بقیه چارچوبها مستلزمِ رصد و آزمایش هستش. اگر در گزاره های بالا، به جای سرعت بگذاریم: شتاب، ممکنه این ابهام پیش بیاد که اثباتِ ما همچنان جواب میده و فیزیک برای چارچوبهای شتابدار هم یکسانه اما می‌دونیم این طور نیست چون آزمایشها نشون میده چارچوبهای شتابدار ناهمسانگرد هستند :)


ـــــــــــــــــــــــــ ــ
ابهامی اگر هست در خدمتم، حالا می‌تونیم با همین فرضها واردِ استخراجِ تبدیلاتِ لورنتس بشیم :)

aysa
09-15-2013, 09:36 PM
با سلام به همه دوستان :)

اين تاپيك ، يكي از تاپيك هاي تالار مباحث غير تخصصي(غير نجومي) بود كه فقط اعضاي ممتازو مديران به مباحث آن دسترسي داشتند .
با ايجاد زيرتالار فيزيك اين تاپيك هم به اينجا منتقل شد.بنابراين از اين پس همه ي اعضا ميتوانند در مطالب آن شركت كنند.براي اطلاعات بيشتر به پست اصلي نگاه كنيد وصفحات قبلي را بخوانيد.
با تشكر از همگي:)

kamy
09-16-2013, 01:36 PM
کسی در باره نظریه f مطلبی داره که در اختیارم بزاره؟

gandom
09-16-2013, 11:02 PM
کسی در باره نظریه f مطلبی داره که در اختیارم بزاره؟

نظریه f در اینجا مخفف کلمه Father هستش. این نظریه یکی از شاخه های نظریه ریسمان محسوب میشه. (شخصاً چون خیلی موهومی و رو کاغذ تعریف میشه خوشم نمیاد ازش :دی). کامران وفا اومد و یک بعد جدید به نظریه M اضافه کرد و اون رو به بعد دوازدهم تعمیم داد. اما این بعدی که ایشون اضافه کرد یک بعد زمانی بود نه مکانی به خاطر همینم بود که به چالش خورد. چون ما در جهانی زندگی می کنیم که فقط یک بعد زمان رو تنها حس می کنیم و از اینکه دو بعد زمانی رو درک کنیم یکم سخته و به خاطر همین قضایای فلسفی میان وسط که چالش کار رو بیش تر می کنن. یکم واضح تر میگم. اگر گذشته و آینده و حال رو روی یک خط راست فرض کنیم و اسمشو بذاریم بعد زمان،گذشته و آینده و حال رو توجیه کردیم اما وقتی زمان رو به ابعاد بالاتر تعمیم بدیم مثلاً دو بعد، یک صفحه زمانی دارم که توجیه آینده و... سخت میشه!
این از بعدی که اضافه شد. اما نکته جالبی که تو این نظریه هستش اینه که دیگه قوانین فیزیک برای ناظران یکسان باقی نمی مونن و این یعنی بی اعتبار بودن نسبت. خود کامران وفا هم معتقد هستش که این نظریه در حال حاضر به صورت انتزاعیه و با توجیه این که خیلی از پدیده های فیزیکی ابتدا روی کاغذ پدید اومدن و بعد تو طبیعت پیدا شدن،ممکن هستش که روزی این ابعاد زمانی بالاتر هم قابل درک و مشاهده باشن. در اون صورت دیگه پدیده های فیزیکی این طوری که ما می بینیم رفتار نخواهند کرد. همه اینا ینی دنیایی جدید با قوانین فیزیکی جدید!

این تاپیک رو هم بخونین :)
http://forum.avastarco.com/forum/showthread.php?1465-%D9%86%D8%B8%D8%B1%DB%8C%D9%87-%DB%8C-%D8%B1%DB%8C%D8%B3%D9%85%D8%A7%D9%86-%D9%87%D8%A7

آیو
09-17-2013, 12:06 AM
تجربه ی گذشته ی ما میگه اگر اتوموبیلی با سرعت ِ 80 بره و ما در اتوموبیلی با سرعت 70 موازی قبلی حرکت کنیم، سرعتی که ما از اتوموبیل ِ 80 کیلومتری خواهیم دید، 10 کیلومتر در ساعت خواهد بود، همین تجربه میگه اگر ما به سمت ِ نور حرکت کنیم سرعت ِ نور با سرعت ِ ما جمع میشه و اگر ازش دور بشیم سرعت ِ ما کسر میشه. اما این معادلات دقیقا پیش بینی می کردند که سرعت c هستش، پس این c باید نسبت به محیطی باشه دیگه؟ نمی تونه که همین طوری تو هر دستگاهی c باشه که؟ می تونه؟ دِ نمی تونه! :دی


سلام
میدونم که عقبم چون امروز شروع کردم
نمیدوننم که الان درباره چه تو صفحه 4 صحبت میشه فقط این جمله رو برام تفسیر کنه یه دوست عزیز ممنون میشم...

arashgmn
09-17-2013, 01:14 AM
[B]لم دستگاه ِ میانی (یا لم ِ چارچوب ِ میانی)

فرض کنید دو تا چارچوبِ لخت داریم، یکی s و دیگری 's ، چارچوبِ 's با سرعتِ v نسبت به s در حالِ حرکت هستش، لم ِ دستگاهِ میانی میگه که وجود دارد چارچوبی مثلِ "s که مشاهده می‌کنه s و 's با سرعتِ یکسان به دو جهتِ مخالف در حالِ حرکتند.


http://up.avastarco.com/images/kp0ij3jwoo2shwvx0852.png

اثبات:

فرض کنید دستگاهِ r رو داریم که می‌تونیم سرعتش رو تغییر بدیم، w ، یعنی سرعتِ دستگاه ِ r از دیدِ چارچوب ِ s می‌تونه از صفر تا v تغییر کنه، وقتی سرعتِ r صفر باشه r دقیقا منطبق بر s هستش و وقتی برابر با v باشه r دقیقا منطبق بر s’ هستش.

دو تابع ِ پایین رو در نظر بگیرید:

f(w) l سرعتِ دستگاه ِ s از دیدِ ناظر ِ ساکن درون ِ r هستش
g(w) l سرعت ِ دستگاه ِ 's از دید ِ ناظر ِ ساکن درون ِ r

میدونیم که g(w) l به صورتِ پیوسته از v تا صفر تغییر می‌کنه یعنی g(0)=v و g(v)=0 ، اما f(w) l از صفر تا یک سرعت ِ مثبتی مثل ِ u تغییر می‌کنه یعنی f(0)=0 و f(v)=u حالا تابعِ اختلاف ِ سرعت ِ بین ِ دو دستگاه میشه D(w)=f-g

تابع ِ D(0)=-v و D(w)=+u ، ما دنبال ِ اثبات ِ وجود ِ سرعتی مثل ِ a هستیم که در اون D(a)=0 باشه (یعنی ناظر سرعت ِ یکسان و مخالف الجهتی از s و 's آشکار کنه. چون D پیوسته است و از منفی تا مثبت تغییر میکنه پس حتما دستِ کم یک جا (سرعت) وجود داره که D حتما تغییر علامت میده (صفر میشه) بنا بر این دستگاه ِ "s وجود داره :دی

این لم با نام midframe lemma معروفه.

حالا با این لم میشه برعکس ِ قضیه رو هم اثبات کرد اما قبل از اثبات یه کمی روش فکر کنید :دی نظری نیست؟

ــــــــــــــــــ

ابهامی هست اگر در خدمتم :)

من اون قسمت رنگی رو متوجه نشدم. چرا (f(v باید یه مقدار مثبت داشته باشه ؟ مگه سرعت به سمت جهت منفی نیست ؟

و اینکه چرا باید یه مقدار (نامشخص) u داشته باشه؟ مگه نباید مقدارش همون v باشه؟ الان تو این تیکه (که سرعت باید احتمالا u ای باشه که مساوی با v نیست) به طور ضمنی تبدیلات لورنتس رو سرعت ها رو وارد نکردیم ؟!

یه نکته ای هم که متوجه نشدم اینه که در تعریف توابع f و g چرا به خط عمودی ( " | " ) سمت راست این دو گذاشته شده ؟ منظور چی بوده ؟!

کلا گیج شدم ... ممنون میشم اگه توضیح بیشتری بدید . :دی

Ehsan
09-17-2013, 07:52 PM
من اون قسمت رنگی رو متوجه نشدم. چرا (f(v باید یه مقدار مثبت داشته باشه ؟ مگه سرعت به سمت جهت منفی نیست ؟



مگه قرار نیست اثبات کنیم دستگاهِ میانی وجود داره که خواهد دید هر دو دستگاه داره با سرعتِ یکسانی اما در دو جهتِ مخالف دور میشه؟ دستگاههای مختلف هم با چی از هم جدا میشن و تمییز داده میشن؟ با w (یا سرعتشون)

حالا من باید نشون بدم سرعتی (چارچوبی) وجود داره که در اون، مشاهده گرِ درونِ چارچوب، سرعتِ یکسانی رو به هر دو چارچوبِ اطرافش نسبت میده، یعنی اگر یکی رو با سرعت n ببینه دیگری رو با n- خواهد دید، حالا برای این کار فرض میکنم سرعتِ یکی با تابعِ f(w) l مشخص بشه و دیگری با g(w) l (علامتِ هر دو سرعت رو مثبت در نظر میگیریم تا ببینیم کجا با هم برابر میشن، به جای این که ببینیم کجا منفیِ یکی با اون یکی برابر میشه، کارِ راحتتری هستش، نه؟ :دی )



و اینکه چرا باید یه مقدار (نامشخص) u داشته باشه؟ مگه نباید مقدارش همون v باشه؟ الان تو این تیکه (که سرعت باید احتمالا u ای باشه که مساوی با v نیست) به طور ضمنی تبدیلات لورنتس رو سرعت ها رو وارد نکردیم ؟!

نه الزاما
با همگنی و همسان گردی میشه نشون داد که v رو میبینه و با اصلِ نسبیت هم میشه نشون داد که سرعتِ ما در v/2 مساوی خواهد شد اما ما هیچ کدوم از اینها رو نمیدونیم :دی یعنی می‌خواهیم کلی ترین حالتِ ممکن باشه.




یه نکته ای هم که متوجه نشدم اینه که در تعریف توابع f و g چرا به خط عمودی ( " | " ) سمت راست این دو گذاشته شده ؟ منظور چی بوده ؟!

اگر نمیگذاشتم میشد این طوری : f(w(

خوب منم نمی‌خوام پارانتز ها به هم بخوره :))

Ehsan
09-18-2013, 07:28 PM
برای استخراجِ ریاضی تبدیلاتِ لورنتس دو مطلبِ زیر رو هم باید اضافه کنیم

وضعیتِ استاندارد:

اگر دو چارچوب طوری قرار بگیرند که:

1.در زمانِ صفر برای هر دو دستگاه، مبدا دو چارچوب روی هم منطبق باشد

2.در زمانِ صفر محورهای مختصات روی هم منطبق باشد

3.سرعت در راستای محورِ x هر دو دستگاه باشد

میگیم که دو دستگاه در وضعیتِ استاندارد هستند. وضعیتِ استاندارد، استخراجِ تبدیلاتِ لورنتس رو ساده می‌کنه.


قضیه ی انعکاس:

چهار دستگاهِ مختصاتِ لختِ زیر را در نظر بگیرید:

http://up.avastarco.com/images/66k6qzwsdetlkeakp9q2.png

دو چارچوبِ S و '''S نسبت به دو چارچوبِ 'S و ''S در وضعیتِ استاندارد قرار دارند.

از دیدِ چارچوبهای 'S و ''S دو چارچوبِ S و'''S با سرعت v در حالِ دور شدن هستند و از دیدِ '''S و S هم وضعیت مشابه هستش یعنی اونها هم میبینند که 'S و ''S در حالِ دور شدن با سرعتِ v هستند. (در غیر این صورت همسانگردی نقض میشه (چرا؟))


قبول دارید که وضعیتِ S نسبت به 'S دقیقا مشابهِ وضعیتِ ''S نسبت به '''S هستش؟ (نحوه ی قرار گیری محورها و سرعتِ دور شدن همگی مشابه هستش)

حالا اگر تبدیل از S به'S با تابعی مثلِ x'=f(x,y,z,t) l و t'=g(x,y,z,t) l بیان بشه ("l" به خاطرِ خوشگلی معادله گذاشته شده :دی) اون وقت تبدیل از دستگاهِ ''S به '''S هم دقیقا با همین تابع تبدیل بیان خواهد شد (اصلِ نسبیت؛ چارچوبهای لخت هیچ برتری نسبت به هم دیگر ندارند) به عبارتِ ریاضی x'''=f(x'',y'',z'',t'') l و t'''=g(x'',y'',z'',t'') l برقرار خواهد بود.

حالا از روی شکل کاملا واضحه که x'''=-x , y'''=-y , z'''=-z و البته مختصه های زمانی با هم برابر هستند یعنی t'''=t

همین وضعیت برای 'S و ''S هم صادقه یعنی: x''=-x' ,y''=-y' , z''=-z' l و البته همون موضوعِ مختصه ی زمانی همچنان سرِ جاشه: t''=t' l

در نتیجه ما معادلاتِ زیر رو برای استخراجِ تبدیلات داریم که فقط بینِ مختصه های 'S و S برقرار هستند:

http://up.avastarco.com/images/pvzu21cs44wlc6mjs3ye.png

این معادلات که برای تبدیلِ لورنتس و عکسِ تبدیلِ لورنتس به کارمون خواهد اومد رو داشته باشید (به این قضیه reciprocity هم میگن)

ــــــــــــــــ

ایشاللا از پستِ بعدی دیگه تبدیلاتِ لورنتس رو استخراج می‌کنیم (دو تا سه پست طول می‌کشه) بعدش مقادیرِ زیادی راجع به مفهومِ فضا، اثراتِ نسبیتی و تبدیلات لورنتس و نتایجشون حرف می‌زنیم

بعد از این تا دلتون می‌خواد حرف هست :دی فقط یه کمی تحمل کنید که این تیکه های ریاضیِ لازم بگذره :دی

Ehsan
09-22-2013, 08:01 PM
خوب حالا فرض کنید دو دستگاهِ S و 'S رو در وضعیتِ استاندارد نسبت به هم داریم. 'S با سرعتِ v در راستای x در حالِ دور شدن از S هستش و در زمانِ t=0 مبدا دو دستگاه روی هم منطبق است.


http://upload.wikimedia.org/wikipedia/commons/9/90/Standard_conf.png

برای به دست آوردنِ تبدیلات از کجا شروع کنیم؟ خوب، از قانونِ اولِ نیوتون، چه جوری؟ معلومه دیگه اگه یه ذره توی یک دستگاهِ لخت روی یک خط حرکت کنه، توی بقیه ی دستگاه های لخت هم روی یک خط حرکت خواهد کرد (دقت کنید هنوز حرفی از قوانینِ همگنی و همسانگردی نزدم فقط دارم از تعریفِ دستگاه های لخت استفاده می‌کنم) بنا بر این خواهم دید که کلی ترین تبدیلاتی که خطِ راست رو خطِ راست نگه می‌دارند، تبدیلاتِ خطی-کسری هستند (اثباتش یه کمی پیچیده و ریاضی هستش و مربوط به بحثِ معادلات دیفرانسیل، برای اثبات کردنش به اَپِندیکسِ اول این مقاله (http://arxiv.org/pdf/physics/9909009v1.pdf) رجوع کنید)

این تبدیلات این شکلی‌اند:


http://up.avastarco.com/images/vo9t3531y8ehh1knf2k.png

خوب، ما این تبدیلات رو دوست نداریم! چون ممکنه مخرج صفر بشه و ذره بره به بی‌نهایت و این اتفاق فیزیکی نیست، علاوه بر این تبدیلات همگن نیستند و به جایی که تبدیل انجام میشه بستگی دارن، مثلا دو نقطه که x متفاوتی دارند بعد از تبدیل شدن این اختلافِ فاصله شون به زمان وابسطه میشه که ناقضِ همگنیه، پس همگنی و همسانگردی لزوما به میگه ضرایبِ متغییر ها توی مخرج صفره و تبدیلاتِ پایین رو به دست می‌ده که تماما خطی هستند:


http://up.avastarco.com/images/6p86w96d2ilhr3hrlx6.png

خیلی هم خوب! :دی حالا ذره ای رو در نظر بگیرید که روی صفحه ی z=0 قرار داره (یعنی روی صفحه ی xy)، اگر z’ ذره (یعنی مولفه ی z بعد از تبدیل مختصات) غیر ِ صفر باشه اتفاقِ بدی می‌افته! همسانگردی و همگنی نقض میشه، چون این یعنی صفحه ی z=0 به یک سمتی چرخیده یا تغییر موضع داده که میشه این سوال رو پرسید که چرا به اون سمت و چرا در اون نقطه! پس قطعا z’=0 برای y هم داستانِ مشابه‌ای داریم که مستقیما نتیجه می‌ده:

y'=a22*y
z'=a33*z

با استفاده از قضیه ی انعکاس خواهیم داشت که a22^2=a33^2=1 که میشه با اطمینان گفت هر دو ضریب برابر با یک هستند.

حالا ذراتی رو در نظر بگیرید که روی صفحه ی x=vt حرکت می‌کنند، این ذرات باید لزوما همراهِ صفحه ی x’=0 حرکت کنند که یعنی x’ همه شون بی برو برگرد باید صفر بشه، یعنی کلیترین تبدیلِ 'x میشه این:


http://up.avastarco.com/images/nhpcrrzmioskcfaxn782.png

که گاما یک ضریبِ ثابت و نامعلومه که می‌تونه تابعِ سرعت باشه (که جلوتر میبینیم واقعا هست)، با استفاده از قضیه ی انعکاس برای x داریم:


http://up.avastarco.com/images/gjoi3ffj3rnf9yyrness.png

حالا ما کلی‌ترین شکلِ تبدیلاتِ بینِ دستگاه های مختصات رو فعلا با اختلافِ یک ضریبِ ثابت (همون گامای نامعلوم) داریمش:


http://up.avastarco.com/images/8d3unfye1x8wzxax3rwu.png

و دقت کنید که هیچ حرفی غیر از همگنی و همسانگردی نزدیم، نه خبری از سرعتِ نور هست و نه چیزِ دیگه :دی



ادامه بمونه برای پستِ بعد
ابهامی هست در خدمتم :)

تذکر: الکی خوف نکنید! توی معادله های بالا غیر از ضرب و تقسیم هیـــــــــــچ عملِ ریاضی خاصی انجام ندادم! :دی

Ehsan
09-23-2013, 10:03 PM
حالا ما بحث رو تا حدِ یک ضریبِ نامعلومِ گاما که تابعِ سرعت هم بود پیش بردیم. بیاید به جای تابعِ گاما این تابع رو تعریف کنیم (که بُعدش توانِ دومِ سرعته) :


http://up.avastarco.com/images/oqzuei8istjmb8bpw13.png

دقت کنید که هــــیـــچ کارِ عجیب و غریبی انجام ندادیم فقط تابعِ جدیدی از سرعت رو بر حسبِ تابعِ گاما تعریف کردیم. گاما بر حسبِ این تابع میشه این:


http://up.avastarco.com/images/0i30mmen9mditzmslp.png


این طوری شکلِ تبدیلِ چهارم (تبدیلِ زمان) میشه این:


http://up.avastarco.com/images/h90gl3p1ujuh26702nii.png

بیایید دو تا تبدیلِ لورنتسِ پشتِ سرِ هم بزنیم، یک بار از ساکن به یه دستگاه با سرعتِ v یک بار هم از اون دستگاه به یک دستگاهِ دیگه با سرعتِ u نسبت به دستگاهِ متحرک اما در همون راستا (یعنی بریم به دستگاهِ ''S که با سرعتِ u داره از 'S دور میشه) خواهیم داشت:


http://up.avastarco.com/images/ug43eqv34e0ao13s5f7.png

با جایگذاری دو رابطه ی اولی در دو رابطه ی دومی تبدیلِ مستقیم از S به ''S رو خواهیم داشت:


http://up.avastarco.com/images/xwfruf8nrstqaagg7o49.png

با مرتب سازی معادلاتِ بالا خواهیم داشت:


http://up.avastarco.com/images/drnd7ftpkam0tyly01gi.png

خوب! حالا ما می‌دونیم که از دستگاهِ S رفتیم به یک 'S که با سرعتِ v داره از ما دور میشه و از 'S هم رفتیم به یک ''S که داره با سرعتِ u از 'S دور میشه، پس مثلِ اینه که ما با یک سرعتِ w که از جفتِ v و u بزرگتره داریم به سمتِ راست حرکت می‌کنیم، پس می‌تونیم تبدیلاتِ بالا رو با تبدیلاتِ مستقیم از S به ''S مقایسه کنیم :دی


http://up.avastarco.com/images/ljn96znnfy5k9jhhv0fr.png

این کار رو میگذاریم واسه پستِ آینده :دی

تذکر: همچنان هیچ کاری بیشتر از ضرب و تقسیم انجام ندادیم! :دی پس بازم الکی خوف نکنید!

Ehsan
09-24-2013, 09:39 PM
خوب بحث رو تا اینجا پیش بردیم که قرار شد چهار تا معادله ی پایین رو با هم مقایسه کنیم:


http://up.avastarco.com/images/5crgypbem8d2yvjeo1a.png

با مقایسه ی رابطه ی بین مختصه های فضایی و زمانی و برابر قرار دادنِ گاماهای دو رابطه به دست میاریم که:


http://up.avastarco.com/images/20eh1dqeo3777eljkfrp.png

خُب، دو تا گاما که باید حتما برابر باشند دیگه؟! بنا بر این حتما باید f(u)=f(v) l باشد تا این برابری صحیح باشه که نتیجه می‌ده که چی؟ ها؟ صدا نمیاد؟!


نتیجه می‌ده که f تابع نیست بلکه حتما یک عددِ ثابته! پس تبدیل زمان هم میشه این:


http://up.avastarco.com/images/7agptw8m59ezoynqsh9.png

حالا با مقایسه ی هر دو رابطه می‌شه گفت که سرعتِ دستگاهِ ''S از دیدِ S میشه:


http://up.avastarco.com/images/pbvsttwpmvg35xglka7.png

این رابطه ی جمعِ سرعتِ نسبیتی هستش، با استفاده از این رابطه میشه گفت که f حتما یک عددِ مثبته چون در غیر این صورت (منفی بودنِ ثابتِ f ) سرعتِ مجموع ،w، میتونه به ازای یک سری از سرعتهای v و u مثبت، منفی بشه (یعنی من سرعتمو به سمتِ "اونور" زیاد میکنم بعد یه هو میبینم که دارم از "این ور" به دستگاهِ ساکنِ قبلی نزدیک میشم و این اتفاق فیزیکی نیست و ناقضِ همگنی و همسانگردیه)


پس حالا که ثابتِ f مثبته من می‌تونم اون رو بر حسبِ توانِ دومِ یک کمیت با بعدِ سرعت بیان کنم:

f=V^2

و همچنان دقت کنید که من هیچ حرفی غیر از همگنی و همسانگردی نزدم اما شکلِ کلی تبدیلاتِ لورنتس رو به دست آوردم:


http://up.avastarco.com/images/y08ruufroc1kcxtn6yo8.png

تنها مجهولِ من برای این تبدیلات یک ثابت با بعدِ سرعت، V ، هستش که تمامی ناظران در اندازه گیری مقدارِ اون اتفاقِ نظر دارند و هیچ ناظری نمی‌تونه سریعتر از اون حرکت کنه، این حرفا آشناست نه؟!

و باز هم به خاطرِ اهمیتِ زیادش تاکید می‌کنم ما تا اینجا هیچ حرفی غیر از همگنی و همسانگردی فضا نزدیم!

این تاکید، یعنی من تا حدِ بسیار خوبی می‌تونم ساختارِ نسبیتِ خاص رو بدونِ استفاده از اصلِ ثابت بودنِ سرعتِ نور یا بدونِ درخواستِ ناوردا بودنِ ساختارِ نسبتا پیچیده ای مثلِ قوانینِ ماکسول به دست بیارم اون هم تنها با یک سری فرضِ کاملا منطقی راجع به فضا که حتی زمانِ نیوتون هم همین فرضها رو راجع به فضا انجام داده بودند، حالا فرض کنید من کنار این فرضِ منطقی (نسبیتِ فضا یا به طورِ هم ارز: همگنی و همسانگردی) یک فرضِ اضافی انجام بدم:

تمامی ناظران در اندازه گیری زمان اتفاقِ نظر خواهند داشت، به عبارتی:

t'=t

(یه به طورِ معادل: «انتقال در فضا طولها رو تغییر نمی‌ده (فضا صلبه)» یا «حدِ بالایی برای سرعتِ اجسام وجود نداره» و یا هر فرضِ دیگری که با این ها معادل باشه، دقت کنید تمامِ این فرضها به طرزِ بسیار عجیب و زیبایی با شهودِ ما از فضا یا زمان هماهنگ هستند)


اون وقت هر کدام از این فرضها بلا فاصله تبدیلاتِ بالا رو به شکلِ تبدیلاتِ معروفِ گالیله ای در میاره که همه ی ما باهاش آشناییم اما معنی ِ این اتفاق چیه؟!

معنیش اینه که قوانینِ نیوتون هم نسبیتی بود! اما در حوزه ی مشاهدات و فرضهای اون زمان.

حالا داستانِ هیجان انگیزِ ما از اینجا به بعد تازه شروع میشه، این که چه طور شکلِ یکتای تبدیلاتِ لورنتس رو به دست بیاریم و ثابتِ V رو چه طور تعیین کنیم بحثِ بعدی ماست، میشه خیلی راحت به آزمایشات ارجاع داد و گفت که این ثابت برابر با سرعتِ نور هستش اما من نمی‌خوام این قدر راحت راجع بهش قضاوت کنم :دی می‌خوام از یه زاویه دیگه نگاه کنیم و از یه پس‌کوچه ای بریم تا یه چیزایی جالبی ببینیم با هم :دی :دی

بحثهای بعدی‌ترِ ما مربوط به پدیده های عجیبِ نسبیتی خواهد بود، خبرِ خوب اینه که فعلا خبری از ریاضیات نیست :دی سوالی هست در خدمتم :)

aysa
09-26-2013, 12:54 AM
آقا گفتين خوف نكنين ما هم نكرديم.ولي ديديم هيچي متوجه نشديم:دی رفتم از صفحه ي اول خوندم ديدم تا حالا اصلا هيچ چي نفهميده بودم.ولي الان فهميدم ولي بعد صفحه ي3/5همون قضايا تكرار شد.:blink:
اون اثباتارو كه زورزوركي به خودم فهموندم ولي بعد پست41به بعد گير كردم.دوتا آخريم نخوندم
به هوش خودمم شك كردم چون اين همه تشكر خورده وهيچكي هيچ سوالي نپرسيده.يعني همه فول فهميدن.يا ديدن كلا نفهميدن هيچي نگفتن؟:دی

(در غیر این صورت همسانگردی نقض میشه (چرا؟))

واقعا چرا؟:دی (پست43)


با استفاده از قضیه ی انعکاس خواهیم داشت که a22^2=a33^2=1 که میشه با اطمینان گفت هر دو ضریب برابر با یک هستند.

حالا ذراتی رو در نظر بگیرید که روی صفحه ی x=vt حرکت می‌کنند، این ذرات باید لزوما همراهِ صفحه ی x’=0 حرکت کنند که یعنی x’ همه شون بی برو برگرد باید صفر بشه، یعنی کلیترین تبدیلِ 'x میشه ا





كلا چي؟؟؟
همچنين قرمزه؟؟؟

و دقت کنید که هیچ حرفی غیر از همگنی و همسانگردی نزدیم، نه خبری از سرعتِ نور هست و نه چیزِ دیگه :دی
مطمئنيد؟:crazy:

مداد رنگیهام
09-26-2013, 09:46 PM
تا آقا احسان جواب آیسا جونو بده من یه خلاصه کوچولو بگم؟!:)

ما اومدیم دوتا چارچوب ِاستاندارد S و 'S رو در نظر گرفتیم که 'S در راستای محور x با سرعت v در حال دور شدن از S بود چون فضا زمان رو همسانگرد در نظر گرفته بودیم انتظار دارشتیم که S هم احساس کنه 'S داره با همون سرعت v ازش دور میشه چون در غیر این صورت ما برای حرکت در راستای محور x تمایز قایل شدیم که قوانین فیزیکی درموردش نتایج یکسانی نمیده و این به معنای نا همسانگردیه.بعد اومدیم گفتیم روابطی وجود داره که بر اساس اون میتونیم مولفه های X Y Z T یکی از چارچوبا رو بر اساس مولفه های چارچوب دیگر بنویسیم که اسم این روابطو گذاشتیم فضیه انعکاس بعد چون در نظر گرفته بودیم 'S با سرعت V در جهت محور X از S دور بشه انتظار داشتیم اگر جسمی درون S در نظر بگیریم میزان Y وZ جسم با 'Y و 'Z در چهارچوب دوم برابر بشه بعد رفتیم سراغ محور X و ضابطه ( x'=r(x-vt رو براش تعریف کردیم دنبال گاما بودیم که گریزی زدیم به یه رابطه جدید که تابع گاما رو بر حسب سرعت نشون میداد که درون خودش یه (f(v داشت!بعد اومدیم یه چارچوب سوم "S نامی رو در نظر گرفتیم که با سرعت u از 'S دور میشد و چون 'S هم با سرعت v از s دور میشد در کل گفتیم "S با سرعت کلی W از S دور میشه با کمک قضیه انعکاس یه بار مستقیم از "S به S رفتیم یه بارم اول از "S به 'S بعد از 'S به S رفتیم و روابط به دست اومده رو با هم برابر قرار دادیم و اینجا بود که به نکته جالبی رسیدیم که: f(v) =f(u و نتیجه گرفتیم تابع f در واقع یک تابع ثابته که میتونه برابر با یک عدد ثابت از جنس سرعت باشه سرعتی که همه جا یکسانه در همه چارچوبا...خیلی راحت میتونستیم بگیم این همون سرعت نوره ولی خب این کار رو نکردیم گفتیم یه دور میزنیم بر میگردیم :دی :)
......................
هرجاشو بد گفتم اصلاح کنین:)

Ehsan
09-26-2013, 09:59 PM
آقا گفتين خوف نكنين ما هم نكرديم.ولي ديديم هيچي متوجه نشديم رفتم از صفحه ي اول خوندم ديدم تا حالا اصلا هيچ چي نفهميده بودم.ولي الان فهميدم ولي بعد صفحه ي3/5همون قضايا تكرار شد.
اون اثباتارو كه زورزوركي به خودم فهموندم ولي بعد پست41به بعد گير كردم.دوتا آخريم نخوندم
به هوش خودمم شك كردم چون اين همه تشكر خورده وهيچكي هيچ سوالي نپرسيده.يعني همه فول فهميدن.يا ديدن كلا نفهميدن هيچي نگفتن؟


اشکال نه از منِ نه از هوشِ شما نه از تشکر کنندگان :دی

مشکل اینه که شما هنوز به کاربستِ ریاضیاتِ پارامتری (یعنی جبر بدون استفاده ی صریح از اعداد) کاملا عادت ندارید؛ کاری که من در سرتاسرِ این چند پستِ پایانی انجام دادم!

اگر می‌خواستم خیــــــــلی با تفضیل حرف بزنم کار از سه چهار تا پست به سی چهل تا پست میرسید! واسه همین دیگه خلاصه گفتم هر وقت خیلی به این نگاه از ریاضیات عادت کردید اونوقت این پستها و اثباتها براتون واضح می‌شه (این اتفاق آخرِ دبیرستان می‌افته نگران نباشید :) )



واقعا چرا؟ (پست43)

به این خاطر که اگر این طوری نباشه اون وقت من تنها با چرخشِ دستگاه ِ مختصات نتیجه ی آزمایشم (خوانشِ سرعتِ دستگاهِ دیگر) عوض شده! پس باید v ببینه.



كلا چي؟؟؟
همچنين قرمزه؟؟؟


اگر توی قضیه انعکاسی که به دست آوردیم به جای x بگذاریم z و تبدیلات رو بررسی کنیم اون روابط به دست میان :) قضیه ی انعکاس رو توی پستِ 43 توضیح دادم. (کلا نکته همونه که شما هنوز به این نحوه ی برخورد با ریاضیات به خوبی عادت نکردید! :دی



مطمئنيد؟


پیدا کنید ببینم غیر از همگنی و همسانگردی (یا به طورِ معادل: اصلِ نسبیت) فرضِ دیگه ای انجام دادیم؟! اصلا کلِ زیبایی قضیه اینه که وجودِ یک ثابتِ جهانی رو داریم پیش بینی می‌کنیم :دی




تا آقا احسان جواب آیسا جونو بده من یه خلاصه کوچولو بگم؟!:)

ما اومدیم دوتا چارچوب ِاستاندارد S و 'S رو در نظر گرفتیم که 'S در راستای محور x با سرعت v در حال دور شدن از S بود چون فضا زمان رو همسانگرد در نظر گرفته بودیم انتظار دارشتیم که S هم احساس کنه 'S داره با همون سرعت v ازش دور میشه چون در غیر این صورت ما برای حرکت در راستای محور x تمایز قایل شدیم که قوانین فیزیکی درموردش نتایج یکسانی نمیده و این به معنای نا همسانگردیه.بعد اومدیم گفتیم روابطی وجود داره که بر اساس اون میتونیم مولفه های X Y Z T یکی از چارچوبا رو بر اساس مولفه های چارچوب دیگر بنویسیم که اسم این روابطو گذاشتیم قضیه انعکاس بعد چون در نظر گرفته بودیم 'S با سرعت V در جهت محور X از S دور بشه انتظار داشتیم اگر جسمی درون S در نظر بگیریم میزان Y وZ جسم با 'Y و 'Z در چهارچوب دوم برابر بشه بعد رفتیم سراغ محور X و ضابطه ( x'=r(x-vt رو براش تعریف کردیم دنبال گاما بودیم که گریزی زدیم به یه رابطه جدید که تابع گاما رو بر حسب سرعت نشون میداد که درون خودش یه (f(v داشت!بعد اومدیم یه چارچوب سوم "S نامی رو در نظر گرفتیم که با سرعت u از 'S دور میشد و چون 'S هم با سرعت v از s دور میشد در کل گفتیم "S با سرعت کلی W از S دور میشه با کمک قضیه انعکاس(1) یه بار مستقیم از "S به S رفتیم یه بارم اول از "S به 'S بعد از 'S به S رفتیم و روابط به دست اومده رو با هم برابر قرار دادیم و اینجا بود که به نکته جالبی رسیدیم که: f(v) =f(u و نتیجه گرفتیم تابع f در واقع یک تابع ثابته که میتونه برابر با یک عدد ثابت از جنس سرعت(2) باشه سرعتی که همه جا یکسانه در همه چارچوبا...خیلی راحت میتونستیم بگیم این همون سرعت نوره ولی خب این کار رو نکردیم گفتیم یه دور میزنیم بر میگردیم :دی :)
......................
هرجاشو بد گفتم اصلاح کنین

خیلی هم خوب، دستتون درد نکنه :) خوب جاهای اصلاحی رو شماره می‌گذارم:

1. اونجا از تبدیلاتِ به دست آمده ی لورنتس استفاده کردیم، قضیه ی انعکاس مالِ یه جایِ دیگه بود.

2. البته جنسِ f توانِ دومِ سرعت بود اما چون مثبت بود می‌تونستیم به صورتِ f=V^2 بنویسیم که V همون ثابتِ از جنسِ سرعتِ نور هستش.

Ehsan
10-03-2013, 06:36 AM
خوب ما تنها با توسل به یک فرضِ کاملا معقول و منطقی راجع به فضا («همگنی و همسانگردی قوانینِ فیزیک در همه ی دستگاه‌های لخت» == «یکسان بودنِ قوانینِ فیزیکی در تمامی دستگاهای لخت») نشون دادیم که «باید یک سرعتِ جهانی مثلِ V وجود داشته باشد که همه ی ناظران در اندازه گیری آن اتفاق نظر داشته باشند و هیچ ناظری در هیچ دستگاهی نمی‌تواند بیشتر از این سرعت حرکت کند»

یه نکته وجود داره

ما منظورمون رو قوانینِ فیزیک به طورِ دقیق مشخص نکردیم! یعنی در طیِ استخراجِ تبدیلاتِ بینِ دستگاه‌ها ما فقط از یک و نیم(!) تا از قوانینِ فیزیک استفاده کردیم:

الف) از قانونِ اول نیوتون (که باید استفاده می‌کردیم، یعنی وقتی داریم از «دستگاه‌های لخت» حرف میزنیم منظورمون دستگاه‌هایی هستش که ذراتِ آزاد توی اون روی خط حرکت می‌کنند و در واقع این تعریفِ دستگاهِ لخت هستش بنا بر این نمیشه به اون معنا گفت که استفاده کردیم، در واقع اصلا شروعِ نسبیت با همین دستگاه‌های لخت بود و استفاده از تعریفِ دستگاه‌های لخت، واجب)

ب) اندازه گیری طول به عنوانِ یک آزمایش ِ فیزیکی ( این همون نیم‌تا قانونِ فیزیک هستش :)) میشه به این مورد نگفت قانون! در واقع وقتی داریم از فضا حرف می‌زنیم منظورمون قطعا اینه که «طول» یک کمیتِ فیزیکی هستش پس باید اندازه گیری طول هم آزمایشی باشه که نسبت به فضا همگن و همسانگرده پس این قانون هم به اون معنا قانون نیست، بدیهیه!)

اما با استفاده از فقط همین قوانین ما شکلِ ِ تبدیلات رو استخراج کردیم و اون گزاره رو صادر کردیم (همون وجودِ سرعت و این داستانا) حالا اگر یک قانونِ فیزیکی قدیمی اضافه کنیم مثلِ این که

حدی برای سرعتِ متحرک وجود ندارد (V بی نهایت است)

زمان اسکالر است

انرژی اسکالر است

فضا صلب است (یعنی فضا، چارچوبی هستش که طولها توش عوض نمیشه)

یا هر چیزِ دیگه، به تبدیلاتِ قدیمی و آشنای گالیله ای می‌رسیم پس ما برای جدا کردنِ نسبیتِ خاص از دنیایِ نیوتون نیاز داریم منظورمون رو از قوانینِ فیزیک دقیقا مشخص کنیم.

این جا جایی هست که مولفینِ کتبِ نسبیت گیر هستند:

بعضی از مولفین تاکید می‌کنند که اگر منظورِ ما از قوانینِ فیزیک، «قوانینِ الکترومغناطیسِ ماکسول» هم باشه ما کارمون راه می‌افته و اصلِ ثابت بودنِ سرعتِ نور یک اصلِ اضافی هستش (یعنی ما به جای این که قوانینِ ماکسول رو به عنوانِ یک اصلِ دوم بیان کنیم، بیاییم داخلِ «خورجینِ قوانین» فرضش کنیم و بعد بتونیم بگیم که ما تنها با یک اصل تبدیلاتِ لورنتس رو استخراج کردیم) اما بعضی ها می‌گن که ما منظورمون رو از قوانین همون یک ونیم تا در نظر بگیریم و بعد اصلِ ثابت بودنِ سرعتِ نور رو به عنوانِ اصلِ دوم بپذیریم تا ساختارِ نسبیتِ خاص بیرون بیاد و این اصل اصلا اضافی نیست.

گروهِ دوم می‌گن که وجودِ اتر به خودی خود نافی اصلِ اول نیست، چرا که اگر قوانینِ حاکم بر اتر رو بشه پیدا کرد و فهمید که معادلاتِ ماکسول چه طور ایجاد میشن و معادلاتِ ماکسول هم اون رفتارِ قدیمی (یعنی تغییر با حرکت کردن) رو داشته باشن باز هم اصلِ همگنی و همسانگردی دست نخورده باقی می‌مونه، مثلِ این که وجودِ امواجِ صوتی توی آب و تغییرِ سرعتِ این امواج یک چارچوبِ مطلق برامون ایجاد کنه که می‌دونیم نمی کنه یا وجودِ شکلِ خاصِ قانونِ اصطکاک توی یک دستگاهِ خاص دلیل بر مطلق و مرجح بودنِ اون دستگاه نیست.

خوب کی راست می‌گه؟

نمی‌دونم! اما گروهِ دوم رو دوست‌تر می‌دارم! چرا؟!

ناوردا بودنِ ساختارِ ریاضی پیچیده ای مثلِ معادلاتِ ماکسول، اون هم توسط ِ تبدیلات لورنتسی که میشه تنها با توسل به دو تا فرض( «یک فرضِ منطقی راجع به فضا»+« V برابرِ 300000 کیلومتر بر ثانیه است» ) به دست آوردشون خیلی عجیب و جالبه و کاملا غیر بدیهی هستش. یعنی ممکن بود معادلاتِ ماکسول شکلِ دیگه ای پیدا کنه و اصلا قیافه اش عوض بشه و این که عوض نشده تصادفِ عجیب و جالبیه (چرا باید فرضهایی که هیچ ربطی به معادلاتِ ماکسول ندارند، شکلِ اونها رو ثابت نگه دارند؟)

اما شاید بشه اثبات کرد که اگر ما هیچ چیزی (مطلقا هیچ چیزی) غیر از این دو فرضی که گفتم ندونیم و فقط همینا رو بدونیم می‌تونیم نشون بدیم که ساختارِ پیچیده ای مثلِ معادلاتِ ماکسول وجود داره و قابلِ به دست آوردن هستند و وجودِ موجی مثلِ نور و معادلاتی مثلِ معادلاتِ ماکسول نتیجه ی فرضِ وجودِ چنین تبدیلاتی هستش، این بی‌نهایت زیباست!

به یاد داشته باشید برهانِ ما برای یافتنِ نسبیت از لحاظِ تاریخی بر دو چیز استوار بود:

1.مشاهداتی راجع به سرعتِ نور که بر خلافِ انتظارِ همه اون رو ثابت می‌دید.

2.آزمایشهایی که می‌خواست اختلافاتِ قوانینِ ماکسول رو مستقیما پیدا کنه که «تصادفا» نتیجه‌ی همه‌ی اونها منتفی بود.

اگر ما تنها دسته ی اولِ مشاهدات رو انتخاب کنیم و از اون دسته مشاهدات، قانونِ بعدی و اصلِ بعدی رو اضافه کنیم می‌تونیم نتیجه ی همه ی مشاهداتِ 2 رو تبیین و پیش بینی کنیم، چون توی فیزیک اصولِ ساده تر همیشه برگزیده هستند پس ما اصلِ دومی رو از آزمایش انتخاب می‌کنیم و اون اینه که «V=c» که c سرعتِ نور هستش و بعد ساختارِ پیچیده ای مثلِ معادلاتِ ماکسول رو پیش بینی می‌کنیم اما این تنها به شرطی هست که بشه واقعا اثبات کرد معادلاتِ ماکسول وجود دارند!

دکتر ثبوتی در این مقاله (http://arxiv.org/pdf/1301.4695.pdf)یک حرکتی در راستای این اثبات زده، من هم تو این یادداشت (http://ehsaneb.persiangig.com/document/maxwell.pdf/download) یه زوری در این راستا زدم و نتیجه گرفتم که به احتمالِ خیلی زیاد تنها نیرویی که تابعِ مشتقاتِ سرعت نیست نیرویی هستش که معادلاتِ ماکسول اون رو پیش بینی می‌کنن، البته از نظرِ من هر دو نوشتار متقن نیست و جاهایی هست که نادقیقه، دستِ کم به نظرِ من (مخصوصا یادداشتِ من!)

(و جالب اینجاست که فاینمن هم اثباتی برای معادلاتِ ماکسول داره که تنها از فرضهای منطقی راجع به فضا ، اما در دنیای کوانتم (جابه‌جا گرها و غیره)، نشان میده تنها نیرویی که تابعِ مشتقاتِ سرعت نباشه نیرویی هستش که از معادلاتِ ماکسول پیروی کنه و لاغیر، و جالبتر اینجاست که فعلا تنها نیروی بزرگ‌مقیاس (غیر از گرانش که مربوط به ذاتِ فضاست) نیروی الکترومغناطیس هستش!)





شکلی از معادلاتِ ماکسول که نهایتا منجر به نور میشه:
http://mstatic.mit.edu/nom150/items/maxwell.jpg



نتیجه ی این بحثها اینه که ما با دو تا اصل نسبیتِ خاص رو ساختیم:

1.قوانینِ فیزیک برای تمامِ چارچوبهای لخت یکسانند (منظورمون از قوانینِ فیزیکی مینی‌مالترین حالتِ ممکن هستش یعنی همون یک و نیم تا قانون!)

2.سرعتی وجود دارد مثلِ سرعتِ نور که تمامی ناظران در اندازه گیری آن اتفاق نظر دارند و هیچ متحرکی در هیچ چارچوبی سریعتر از آن حرکت نمی‌کند

و با فرضِ این دو تا اصل به دنیایی قدم خواهیم گذاشت که به طرزِ وحشتناکی زیباست!

ــــــــــــــ

از این جا به بعد مبتنی بر تبدیلاتی که به دست آوردیم شروع به شرحِ پدیده های عجیب و غریبِ نسبیتی خواهیم کرد انشاالله :)

ابهامی اگر هست بنده در خدمتم (نمی‌دونم چرا حس می‌کنم خیلی مبهم حرف زدم! منظورم واضح نرسید!)

narcissus flower
10-10-2013, 09:09 PM
خیلی ممنون از احسان خان بزرگوار :)
من سوال دارم اقا :دی

چه دلیل فیزیکی و حقیقی وجود داره که تبدیلات رو باید خطی در نظر بگیریم.مثلا اگر اول تبدیلات لورنتس ایجاد میشدند بازهم خودمون رو مجبور به این کار میگردیم؟؟
هندسه فضا و اقلیدسی بودن چه تاثیری تو این روابط داره؟اصلا ربط داره؟؟
یه سوال دیگه
چه دلیلی داره که وقتی سرعت ما خیلی کم باشه به تبدیلات گالیله میرسیم .و سرعت نور هم که بی نهایت میشه باز تبدیلات گالیله میشه؟؟غیر از خودفرمول...دلیل فیزیکی داریم؟؟

یک چیزی , ما میگیم میدان الکتریکی مستقل از چشمه اش هست , یعنی وقتی درست میشه ما دیگه کاری ندارم منشا اش چی بوده و اینا , خودش هست .یک شخصیت مستقل داره و هویت دار هست., از طرفی نور یک جورایی عجیب به میدان الکتریکی وصل هست , حالا میشه بگیم این استقلال سرعت از چارچوب و ناظرش هم به این دلیل هست ؟؟:دی

Ehsan
10-14-2013, 10:54 PM
....

من سوال دارم اقا :دی

چه دلیل فیزیکی و حقیقی وجود داره که تبدیلات رو باید خطی در نظر بگیریم.مثلا اگر اول تبدیلات لورنتس ایجاد میشدند بازهم خودمون رو مجبور به این کار میگردیم؟؟

بله بله! :دی

توی این پست (http://forum.avastarco.com/forum/showthread.php?1443-%D9%81%D9%8A%D8%B2%D9%8A%D9%83-%D8%AC%D8%AF%D9%8A%D8%AF&p=69197&viewfull=1#post69197) که شروع می‌کردیم تبدیلات رو استخراج کنیم گفتیم که بالاخره تبدیلاتِ ما بینِ دو دستگاهِ لخت نوشته میشه دیگه؟! دستگاه‌های لخت هم این ویژگی رو دارند که ذره ی آزاد روی خط راست حرکت می‌کنه، پس اگر متحرکی درونِ دستگاهی با سرعتِ ثابت حرکت کنه باید تبدیلاتِ ما طوری باشه که توی دستگاهِ دیگه هم حرکتش خط باقی بمونه، از اینجا یک سری معادله به وجود میومد که تبدیلاتِ کسری-خطی رو نتیجه میداد و بعد با کمی استدلالِ فیزیکی و مطرح کردنِ «اندازه گیری طول» به عنوانِ آزمایش نتیجه گرفتیم که تبدیلات حتما باید خطی باشند تا هم همگنی رعایت بشه و هم این که ذراتِ آزاد در هر دستگاهی آزاد باشند.



هندسه فضا و اقلیدسی بودن چه تاثیری تو این روابط داره؟اصلا ربط داره؟؟

این سوال خیلی خوبه،

این قضیه وقتی وارد میشه که دارم از «اندازه گیری طول» حرف میزنم (مثلِ همون موردِ قبلی که گفتید) ما فضای اقلیدسی رو با اندازه گیری طول می‌شناسیم فضاهای غیر اقلیدسی روابطِ عجیبی برای طولها و اندازه ها دارند که موردِ بحثِ ما نیست و ما دوست داریم طولها به صورتِ اقلیدسی رفتار کنند، مثلا وقتی من از x=1 میرم به x=2 واقعا یک متر طی کرده باشم، در فضاهای اقلیدسی جابه جایی تو این حالت «ممکنه» یک نباشه.

البته طنزِ ماجرا اینجاست که فقط بخشِ فضایی اقلیدسی محسوب میشه، فضا-زمان ساختارِ شبه-اقلیدسی داره



چه دلیلی داره که وقتی سرعت ما خیلی کم باشه به تبدیلات گالیله میرسیم .و سرعت نور هم که بی نهایت میشه باز تبدیلات گالیله میشه؟؟غیر از خودفرمول...دلیل فیزیکی داریم؟؟


در واقع تمامِ چیزی که تبدیلات لورنتس رو از گالیله ای دور می‌کنه نسبتِ v/c هستش که v سرعتِ متحرک و c سرعتِ نور هستش، از نظرِ ریاضی وقتی c بره به بینهایت (سرعتِ نور بی‌نهایت بشه) یا v بره به سمتِ صفر (متحرک با سرعتِ کم حرکت کنه) در هر دو حالت نسبتِ v/c به سمتِ صفر میره و معادله ها می‌تونید ببینید که تبدیلاتِ گالیله ای به دست میاد

اما از لحاظِ فیزیکی یه چیزی هست که فکر کنم بهش می‌گن اصلِ همخوانی (که از کوانتم اومده) قضیه از این قراره که ما بالاخره 300 سال از معادلاتِ نیوتون استفاده می‌کردیم و همه چیز خوب و عالی بود! پس هر نظریه ای که مغایر با نظریه ی نیوتون باشه (مثلِ نسبیت و کوانتم) در حدودی که مربوط به دنیای ما قبل از 1900 بوده (یعنی سرعتهای معقول و ابعادِ ماکروسکوپی و .... در یک کلام: شرایطِ روزمره) نتایجش باید با تقریبِ خوبی مطابق با نظریه نیوتون باشه (یعنی تبدیلات گالیله هستند و .....) در غیرِ این صورت نظریه رو دور می‌ندازیم! چون به وضوح داره در حوزه ی مشاهداتیی که به راحتی در دسترسِ ماست (دنیای روزمره) غلط عمل می‌کنه :دی



یک چیزی , ما میگیم میدان الکتریکی مستقل از چشمه اش هست , یعنی وقتی درست میشه ما دیگه کاری ندارم منشا اش چی بوده و اینا , خودش هست .یک شخصیت مستقل داره و هویت دار هست., از طرفی نور یک جورایی عجیب به میدان الکتریکی وصل هست , حالا میشه بگیم این استقلال سرعت از چارچوب و ناظرش هم به این دلیل هست ؟؟:دی


نمی‌دونم منظورتون رو درست فهمیدم یا نه اما اگر اونی باشه که من فکر می‌کنم:

نه! صوت وقتی منتشر میشه سرعتش مستقل از چشمه است و هویتِ مستقل داره، اما اما این استقلال باعث نمیشه توی هوا ناظران چاچوبِ ممتازی پیدا نکنند :)

ــــــــــــــــ
من همچنان اگر وقت پیدا کنم راجع به پدیده های نسبیتی حرف خواهم زد :دی

یزدان بابازاده
10-15-2013, 06:07 PM
سلام
من یه سوال نسبتا مفهومی دارم که مدت هاست با خودم حملش می کنم ولی حل نمی شه !!
ببینید قبل از ارائه نسبیت خاص ، فیزیک به چندین دلیل به وجود اتر نیاز داشت ، ولی دو تا از برجسته ترین دلایل :
1- به وجود آوردن مرجعی برای سنجش سرعت نور نسبت به آن
2- توجیح یک ماده که بستر حرکتیه نور باشه ، مثلا هوا بستر حرکت موج صوتیه ...
و دلایل دیگه ...
حالا من دلیل اول و چگونگی رفعش توسط نسبت رو درک می کنم ولی هنوز چگونگی رفع دلیل دوم رو درک نمی کنم !!
اگر به نور از دیدگاه ذره ای نگاه کنیم بازم شهود مند تر از حالت موجیه !
در واقع سوالم اینه که :
الآن تلاطم های الکترومغناطیسی دارند تو "چی " حرکت می کنند ؟؟؟ یعنی بستر حرکتیشون چیه ؟؟
لطفا یکی از دوستان بنده رو توجیح کند !

Ehsan
10-17-2013, 09:33 PM
سلام

.....

اگر به نور از دیدگاه ذره ای نگاه کنیم بازم شهود مند تر از حالت موجیه !

در واقع سوالم اینه که :

الآن تلاطم های الکترومغناطیسی دارند تو "چی " حرکت می کنند ؟؟؟ یعنی بستر حرکتیشون چیه ؟؟

لطفا یکی از دوستان بنده رو توجیح کند !


علیکِ سلام

راستش سوال سوالِ جالبیه! جوابش رو نمی‌دونم :دی اما یه سری توضیحاتِ جالب سراغ دارم:

منظورتون دقیقا از بسترِ حرکت چیه؟

بگذارید سوالتون رو باز تولید کنم تا شاید بهتر متوجه بشیم:

تمامِ تجربیاتِ ما از موج شاملِ این مفهومه یک محیطِ پیوسته از «اجزا»یی وجود داره که ارتباطِ این اجزا با هم به گونه ای تنظیم شده که اختلالی در «حالت» بخشی از اجزای کنارِ هم، در باقی محیط منتشر میشه.

مثلا وقتی یک قسمت از طناب رو کمی «جا به جا» می‌کنم این «جا به جایی» در باقی اجزای طناب منتشر میشه (طناب رو میشه با جرمهای کوچکی که به وسیله ی فنر به هم وصل شدند تقریب زد، پس اجزای طناب اتمها هستند)

یا وقتی یک اختلالِ «فشار» در قسمتی از هوا به وجود میاریم این اختلال منتشر میشه و «فشارِ» اطراف شروع به نوسان میکنه و موجِ صوت شکل می‌گیره (هوا هم از اتمها تشکیل شده)

الگوی کلی که در این مثال ها وجود داره اینه:

یک ساختار وجود داره که هر نقطه از این ساختار یک «ویژگی» یا «حالت» داره و این ویژگی هستش که منتشر میشه:

ساختار & حالت => موج
طناب & جابه جایی => موج ِ طناب
هوا & فشار => موجِ صوت



یه جابه جایی که داره در طی طناب منتشر میشه:
http://upload.wikimedia.org/wikipedia/commons/1/1f/Wave_equation_1D_fixed_endpoints.gif


حالا برمیگردیم به سوالِ شما

برای امواجِ الکترومغناطیسی ما ویژگی رو می‌دونیم: میدانِ الکترومغناطیسی، موج رو هم میشناسیم: موجِ الکترومغناطیسی ، اما سوالتون به طورِ دقیقتر اینه که این «ساختار»ه چیه!؟

یعنی ما باید محیطی دارای «اجزا» پیدا کنیم که میدانِ الکترومغناطیسی ویژگی های اون اجزا باشن، مثلا یه عده ذره ی کنارِ هم پیدا کنیم که کوچیک و بزرگ شدنِ این ذرات میدانِ الکترومغناطیسی ایجاد کنه!

تصویرِ نسبیتِ خاص اینه که چنین محیطی دارای اجزا وجود نداره! نمی‌دونم تصویرِ نظریه ریسمان چیه، شاید نظریه ریسمان واقعا چنین محیطی رو پیش بینی می‌کنه اما من ازش بی‌خبرم، اما یه دیدگاهِ جالبتر هم هست:

میدانهای الکترومغناطیسی ویژگی ذاتی فضا هستند! یعنی اجزایی که قراره تغییرِ حالتشون موجبِ انتشارِ امواجِ الکترومغناطیسی بشه، بافتِ فضا-زمانه و ویژگی و حالتِ فضا-زمان، میدانهای الکترومغناطیسی هستند.

این که یک ماهیتِ مستقلِ بافت‌-گونه برای فضا-زمان قائل باشیم کاری هست که توی نسبیتِ عام انجام میشه :)

اگر روزی بشه اجزای فضا-زمان رو پیدا کرد احتمالا جوابِ سوالِ شما هم کاملا پیدا میشه :)

از طرفی تو انتهای این پست (http://forum.avastarco.com/forum/showthread.php?1443-%D9%81%D9%8A%D8%B2%D9%8A%D9%83-%D8%AC%D8%AF%D9%8A%D8%AF&p=69620&viewfull=1#post69620) تو همین تاپیک یه بخشی گذاشتم که میگه شاید بشه اثبات کرد که ساختارِ فضا-زمانِ نسبیتی و فرضهایی که نسبیت روی فضا-زمان اعمال می‌کنه الزاما وجودِ معادلاتِ ماکسول و در نتیجه موجِ الکترومغناطیسی رو پیش بینی می‌کنه :)

ــــــــــــــــ
از این سوال و این بررسی به شدت لذت بردم :دی

یزدان بابازاده
10-18-2013, 10:28 AM
علیکِ سلام

راستش سوال سوالِ جالبیه! جوابش رو نمی‌دونم :دی اما یه سری توضیحاتِ جالب سراغ دارم:

منظورتون دقیقا از بسترِ حرکت چیه؟

بگذارید سوالتون رو باز تولید کنم تا شاید بهتر متوجه بشیم:

تمامِ تجربیاتِ ما از موج شاملِ این مفهومه یک محیطِ پیوسته از «اجزا»یی وجود داره که ارتباطِ این اجزا با هم به گونه ای تنظیم شده که اختلالی در «حالت» بخشی از اجزای کنارِ هم، در باقی محیط منتشر میشه.

مثلا وقتی یک قسمت از طناب رو کمی «جا به جا» می‌کنم این «جا به جایی» در باقی اجزای طناب منتشر میشه (طناب رو میشه با جرمهای کوچکی که به وسیله ی فنر به هم وصل شدند تقریب زد، پس اجزای طناب اتمها هستند)

یا وقتی یک اختلالِ «فشار» در قسمتی از هوا به وجود میاریم این اختلال منتشر میشه و «فشارِ» اطراف شروع به نوسان میکنه و موجِ صوت شکل می‌گیره (هوا هم از اتمها تشکیل شده)

الگوی کلی که در این مثال ها وجود داره اینه:

یک ساختار وجود داره که هر نقطه از این ساختار یک «ویژگی» یا «حالت» داره و این ویژگی هستش که منتشر میشه:

ساختار & حالت => موج
طناب & جابه جایی => موج ِ طناب
هوا & فشار => موجِ صوت



یه جابه جایی که داره در طی طناب منتشر میشه:
http://upload.wikimedia.org/wikipedia/commons/1/1f/Wave_equation_1D_fixed_endpoints.gif


حالا برمیگردیم به سوالِ شما

برای امواجِ الکترومغناطیسی ما ویژگی رو می‌دونیم: میدانِ الکترومغناطیسی، موج رو هم میشناسیم: موجِ الکترومغناطیسی ، اما سوالتون به طورِ دقیقتر اینه که این «ساختار»ه چیه!؟

یعنی ما باید محیطی دارای «اجزا» پیدا کنیم که میدانِ الکترومغناطیسی ویژگی های اون اجزا باشن، مثلا یه عده ذره ی کنارِ هم پیدا کنیم که کوچیک و بزرگ شدنِ این ذرات میدانِ الکترومغناطیسی ایجاد کنه!

تصویرِ نسبیتِ خاص اینه که چنین محیطی دارای اجزا وجود نداره! نمی‌دونم تصویرِ نظریه ریسمان چیه، شاید نظریه ریسمان واقعا چنین محیطی رو پیش بینی می‌کنه اما من ازش بی‌خبرم، اما یه دیدگاهِ جالبتر هم هست:

میدانهای الکترومغناطیسی ویژگی ذاتی فضا هستند! یعنی اجزایی که قراره تغییرِ حالتشون موجبِ انتشارِ امواجِ الکترومغناطیسی بشه، بافتِ فضا-زمانه و ویژگی و حالتِ فضا-زمان، میدانهای الکترومغناطیسی هستند.

این که یک ماهیتِ مستقلِ بافت‌-گونه برای فضا-زمان قائل باشیم کاری هست که توی نسبیتِ عام انجام میشه :)

اگر روزی بشه اجزای فضا-زمان رو پیدا کرد احتمالا جوابِ سوالِ شما هم کاملا پیدا میشه :)

از طرفی تو انتهای این پست (http://forum.avastarco.com/forum/showthread.php?1443-%D9%81%D9%8A%D8%B2%D9%8A%D9%83-%D8%AC%D8%AF%D9%8A%D8%AF&p=69620&viewfull=1#post69620) تو همین تاپیک یه بخشی گذاشتم که میگه شاید بشه اثبات کرد که ساختارِ فضا-زمانِ نسبیتی و فرضهایی که نسبیت روی فضا-زمان اعمال می‌کنه الزاما وجودِ معادلاتِ ماکسول و در نتیجه موجِ الکترومغناطیسی رو پیش بینی می‌کنه :)

ــــــــــــــــ
از این سوال و این بررسی به شدت لذت بردم :دی

به شدت ممنون !! :دی
راستش واسه من بیشتر جالب بود که چرا هیچ کدوم از کتاب های نسبیت خاص به این قضیه هیچ اشاره ای نکردند ! اولش فکر می کردم جوابش انقدر بدیهیه که نیاز ندونستن اصن اشاره کنن !!!!
ولی الآن فهمیدم که خودشون هم نمی دونستن و اشاره ای نکردن !!
ممنون !

mohsen4465
10-18-2013, 11:10 AM
سوالم اینه که :
الآن تلاطم های الکترومغناطیسی دارند تو "چی " حرکت می کنند ؟؟؟ یعنی بستر حرکتیشون چیه ؟؟
تلاطم‌های الکترومغناطیس در واقع دومین ماهیت از وجود ذره‌ای بنام فوتون‌ها هستن (دوگانگی ذره-موج). ذره حرکت میکنه (فوتون) و موج وجود خودش (الکترومغناطیس) رو با خودش جابجا میکنه. :)

Ehsan
10-18-2013, 12:36 PM
به شدت ممنون !! :دی
راستش واسه من بیشتر جالب بود که چرا هیچ کدوم از کتاب های نسبیت خاص به این قضیه هیچ اشاره ای نکردند ! اولش فکر می کردم جوابش انقدر بدیهیه که نیاز ندونستن اصن اشاره کنن !!!!
ولی الآن فهمیدم که خودشون هم نمی دونستن و اشاره ای نکردن !!
ممنون !

نه خوب! کتابای نسبیت نمی‌گن که ما نمی‌دونیم :دی

می گن که چه اشکالی داره ما امواجی داشته باشیم که توی «خلا» منتشر می‌شن؟! کدوم قانونِ مسلّمِ فیزیک نقض میشه؟

پس قبول می‌کنیم امواجی وجود دارند که توی خلا منتشر میشن و برای انتشار نیازی به محیطِ واسطه ندارند :) اما این حرفایی که زدم کمی برای فرار از این مفهوم بود.

aysa
10-18-2013, 03:20 PM
[B]

میدانهای الکترومغناطیسی ویژگی ذاتی فضا هستند! یعنی اجزایی که قراره تغییرِ حالتشون موجبِ انتشارِ امواجِ الکترومغناطیسی بشه، بافتِ فضا-زمانه و ویژگی و حالتِ فضا-زمان، میدانهای الکترومغناطیسی هستند.

وَو.چه حرف جالبي.خيلي خيلي باحال بود.
ببخشيد ميون كلامتون مي پرم.ولي من به اين نتيجه رسيدم كه در واقع ما درعين اين كه گفتيم به محيطي نياز نداريم به صورت كاملا زيركانه فضا-زمان روبه صورت جايگزين(فك نكنم كلمه ي مناسب باشه! ) استفاده كرديم.
اگه اين طوري باشه كه دقيقا يعني هيچ قانون فيزيك رو هم نقض نكرديم.با اين حساب ما تونستيم اين موج رو هم يه جورايي وارد ميدون كنيم.خب در اين صورت خيلي از مسائلم حل ميشن ديگه.
بعد يه ؤال:نور چه طوري عجيب به ميدان الكتريكي وصله؟؟؟
اميدوارم كه برداشت هام درست باشن.
-------------------------------------------
فعلا كه داريم خيلي خوب پيش ميريمو بحث داره جالب تر ميشه.فقط من از اون اثبات هاي صفحه ي4چيزي نفهميدم.اميدوارم تو ادامه ي مطالب دردسر درست نكنن:)
ممنون:)

gandom
10-18-2013, 08:42 PM
البته اگر که راجع به هیگز بوزون هم سرچ کنین کمک میکنه به پاسخ سؤالتون :)
.........................................
البته من سؤالم یه چیز دیگه است...کسی در رابطه با فوتون جرم دار کتابی سراغ داره؟ الان چند وقته که ذهنمو درگیر کرده...ممنون :)

narcissus flower
10-19-2013, 12:53 AM
وَو.چه حرف جالبي.خيلي خيلي باحال بود.
ببخشيد ميون كلامتون مي پرم.ولي من به اين نتيجه رسيدم كه در واقع ما درعين اين كه گفتيم به محيطي نياز نداريم به صورت كاملا زيركانه فضا-زمان روبه صورت جايگزين(فك نكنم كلمه ي مناسب باشه! ) استفاده كرديم.
اگه اين طوري باشه كه دقيقا يعني هيچ قانون فيزيك رو هم نقض نكرديم.با اين حساب ما تونستيم اين موج رو هم يه جورايي وارد ميدون كنيم.خب در اين صورت خيلي از مسائلم حل ميشن ديگه.
بعد يه ؤال:نور چه طوري عجيب به ميدان الكتريكي وصله؟؟؟
اميدوارم كه برداشت هام درست باشن.
------------------------------------------
ممنون:)
ببین ایسا جان اینکه بگیم زیرکانه فضازمان وارد شده این ابهام رو ممکن ایجاد کنه که ما یک مکان و جایی داریم که جزفضا زمان نیست والان داریم تفاوت وامتیاز براش قائلم میشیم که خب مناسب نیست.در صورتی که حضور فضازمان به صورت پیش فرض و بستر همه چیز هست...

در مورد ارتباط عجیب نور ومیدان الکتریکی ,فکر کنم بدونی که امواج الکترومغناطیسی متشکل از ترکیب میدان های الکتریکی و مغناطیسی هست که بر هم و راستای حرکتشون عمود هستند .بنابراین نور. و میدان به هم مربوط میشوند :)

http://t3.gstatic.com/images?q=tbn:ANd9GcRsXCCBIwA4qzgayEvkGLJXvNBJRE75T OLKZjOZ-LK8lRrzweJcIzqc_Agt

Ehsan
11-01-2013, 06:34 PM
قبل از این که بخواهیم پدیده های نسبیتی رو بررسی کنیم، ابتدا لازمه بینِ «اندازه گیری» و «دیدن» تفاوت قائل بشیم.

با چند آزمایش می‌خوام این دو تا رو شرح بدم:

1.سلسله ی ترقه!

یک چارجوبِ لخت تصور کنید، کردید؟ خوب حالا ده ترقه به طورِ همزمان در نقاطِ 1c ، 2c ، تا 10c روی محورِ x منفجر می‌شن (c همون سرعتِ نوره). در مقامِ یک ناظرِ انسانی، شما در مرکزِ مختصاتِ چارچوب نشستید، وقتی نورِ انفجارِ این طرقه ها به شما رسید، شما دو تا کار انجام می‌دید:

اولا با استفاده از شدتِ نور ِ ترقه ها فاصله ها رو به دست میارید،

ثانیا زمانِ دیدنِ انفجار رو ثبت می‌کنید.

بعد این رویدادها رو این طوری می‌نویسید:
(مکان، زمان)
(چون به خاطرِ سرعتِ محدودِ نور، زمانی طول می‌کشه که تصویرِ ترقه ها به شما برسه، به همین خاطر شما زمانِ انفجارِ ترقه ها رو با تاخیر خواهید «دید»)

(1s ، 0,0,1c )
(2s ، 0,0,2c )
.
.
.
(10s ، 0,0,10c)


http://up.avastarco.com/images/d7x852r0r5urihs1zy9.png

بالفرض شما فیزیکدانِ تنبلی هستید و حال ندارید این زمانهای «دیده شدن» رو با توجه به مکان تبدیل کنید به زمانهای «ترکیدن» پس همین ها رو به عنوانِ نتیجه ی اندازه گیری رد می‌کنید!
مشخصا این «اندازه گیری» کارِ اشتباهی بوده چون ما از اول می‌دونیم رویدادها همزمان هستند اما چیزی که الان می‌بینیم اینه که رویدادها همزمان نیستند!


اما یه فیزیک دانِ بلند همت داریم که اومده یه کارِ خفن انجام داده، این فیزیک دان اومده و در هر نقطه ی فضا یک «بیننده» کارگذاشته و برای هر کدوم یک ساعت نصب کرده و همه ی ساعتها همزمان شدند، هر کدوم از این بیننده ها هم یک برچسب دارند که مکانشون رو مشخص می‌کنه، این بیننده ها هم تنها به رویدادهایی واکنش نشون میدند که درست و دقیق در محلِ حضورشون رخ داده باشه و اگه رویدادی دو متر اونطرف‌تر رخ داده باشه این بیننده ها هیچ چیزی ثبت نمی‌کنند:


http://up.avastarco.com/images/nm2ud9j9vjvcw7sq2lv9.png

حالا اتفاقی که می‌افته اینه که بیننده ی حاضر در مکانِ 1c رویدادِ انفجارِ ترقه رو با زمانِ صفر ثبت می‌کنه، بیننده ی حاضر در مکانِ 2cرویدادِ انفجارِ ترقه رو با زمانِ صفر ثبت می‌کنه و الی آخر، و فیزیکدانِ ما تهِ قضیه سر می‌رسه و اعداد رو ازشون می‌پرسه به به فرمِ قبلی این طوری گزارش می‌ده:

(مکان، زمان)
(0s ، 0,0,1c )
(0s ، 0,0,2c )
.
.
.
(0s ، 0,0,10c)

کاری که فیزیکدانِ دومی انجام داده اسمش اندازه گیری هستش و گزارشی هم که می‌ده قابلِ استناده! در واقع وقتی می‌گیم «ناظر» منظورمون یک شخصِ نشسته در یک گوشه ی مختصات نیست، منظورمون مجموعه ای از ناظرانِ کوره که گوشه گوشه ی دستگاهِ مختصات نشستند و ساعتهای همزمانی دارند و مشغولِ ثبتِ رویدادهایی هستند که در نقطه ی حضورشون رخ میده

در پستِ بعدی یک مثالِ دیگه و مهمتر از تفاوتِ اندازه گیری و دیدن رو شرح می‌دم و بعدش واردِ پدیده های نسبیتی می‌شیم.

ـــــــــــــــــــــــــ ـ

پ.ن: دلیلِ تاکیدم به تفاوتِ این دو تا رو تو پستِ بعدی می‌گم و اونجا هم می‌فهمید که چرا مهمه حتما تفاوتِ بین این دو تا یادمون باشه، فعلا همین قدر بگم که یه بار یه عده منجم به خاطرِ عدمِ دقت بینِ این دو تا مقوله گزارش کرده بودند که متحرکهایی سریعتر از نور رصد کردند به همین خاطر کلی فیزیکدانان رو به دردسر انداختند :))

Ehsan
11-05-2013, 09:19 PM
2.اندازه گیری طول

فرض کنید که می‌خواهید طولِ یک قطار رو که داره حرکت می‌کنه رو اندازه بگیرید، فرض کنید می‌دونیم که طولِ قطار چهارصد متر هستش و با سرعتِ 72 کیلومتر بر ساعت هم در حالِ حرکته.
بازم دو تا فیزیکدان داریم!

فیزیک دانِ اولی این طوری طولِ قطار رو اندازه می‌گیره:

اون منتظره تا ابتدای قطار بهش برسه، بعد که ابتدای قطار رسید سریع محل رو علامتگذاری می‌کنه و با سرعتِ 5 متر بر ثانیه می‌دوه به سمتِ انتهای قطار و وقتی رسید به انتهای قطار محلِ رسیدنش رو علامت می‌زنه! و بعد اختلافِ بینِ دو محلی که علامت زده رو اعلام می‌کنه، چیزی که به دست آورده مقدارش به جای چهارصد متر، 80 متر خواهد بود!!


http://up.avastarco.com/images/2wkne11lsazxtktk4ug.png

فیزیک دانِ دوم، فیزیک دانِ بلند همتِ ماست که توی هر نقطه از ریل یک بیینده قرار داده که فقط رویدادهای محلی که در اون قرار دارند رو ثبت می‌کنند، حالا فقط کافیه دو تا بیننده پیدا کنه «هم‌زمان» رویدادِ
گذرِ ابتدا و انتهای قطار رو ثبت کردند و بعد مکانِ این دو بیننده رو از هم کم کنه، این فیزیکدان طولِ قطار رو 400 متر به دست خواهد آورد!


http://up.avastarco.com/images/29033nal22l5fbaqtevu.png

این دو آزمایش نشون میده که برای اندازه گیری طول حتما و حتما باید «اندازه گیری» ِ «همزمان» انجام بشه و الا این کار بیهوده است! و البته بی معنی! طول یعنی چی؟

مقدار فضایی که جسم «در یک لحظه» در دستگاهِ مختصاتِ من اشغال می‌کنه. «در یک لحظه» یعنی من باید تمامِ نقاط رو همزمان نگاه کنم و الا اندازه گیری طولِ من مثلِ کارِ فیزیکدانِ اولی بی‌معنی خواهد بود.

حالا یک آزمایشِ اندازه گیری طول با دیدن:

فرض کنید یک قطار داریم به طولِ یک سالِ نوری! که با سرعتِ نصفِ نور در حالِ حرکته (فعلا از اثراتِ نسبیتی صرفِ نظر کنید، اگر هم صرفِ نظر نمی‌کنید به این فکر کنید که قطار در دستگاهِ مختصاتِ من طولش یک سالِ نوری هستش فارغ از هر طولی که در دستگاهِ سکونش داشته باشه)

باز یک فیزیک دانِ تنبل داریم که دو بار اندازه گیری انجام میده: یک بار وقتی ابتدای قطار بهش رسید «نگاه می‌کنه» تا ببینده انتهای قطار کجاست، اما چون نوری که از انتهای قطار بهش میرسه مربوط به گذشته است، اون مکانِ انتهای قطار رو در نقطه ای خواهد دید که قطار الان اونجا نیست، یعنی انتهای قطار رو در دو سالِ نوری اون طرف تر خواهد دید و در نتیجه عددِ اشتباهی رو به عنوانِ طول اعلام می‌کنه: دو سالِ نوری!

یک بارِ دیگه وقتی انتهای قطار بهش رسید، «نگاه می‌کنه» تا ببینه ابتدای قطار کجاست، اما نوری که از ابتدای قطار بهش رسیده هشت ماهِ قبل راه افتاده، در نتیجه مکانِ هشت ماهِ قبلِ ابتدای قطار رو خواهد دید و طولی هم که اعلام می‌کنه تقریبا شصت درصدِ طولِ واقعی خواهد بود!

اما اگر کسی به حالتی که فیزیکدانِ بلند همتِ ما «اندازه گیری» انجام داده بود، اندازه گیری انجام میداد طولِ واقعی قطار رو به دست می‌آورد (یعنی مکانِ ابتدا و انتهای قطار رو همزمان علامت بزنه)

پس حالا کاملا باید درک کرده باشید که:

اندازه گیری و دیدن دو فرایندِ کاملا متفاوت در نسبیتِ خاص هستند و هر کدام نتیجه ی خودشون رو دارند و اصلا نباید با هم اشتباه گرفته بشند!

اهمیتِ تفکیکِ بینِ این دو فرایند هم اینه که درصدِ بالایی از متناقض‌نماهای موجود در نسبیتِ خاص به خاطرِ اینه که در مکانیکِ کلاسیک ما اصلا تفکیکی بینِ این دو قائل نمی‌شدیم اما الان مجبوریم تفکیک قائل بشیم، مخصوصا وقتی نسبی بودنِ همزمانی رو در ذیلِ اثراتِ نسبیتی توضیح بدم اونجا کاملا متوجه خواهید شد که دلیلِ این تفکیک چیه.

Ehsan
11-20-2013, 11:40 PM
خوب حالا وقتشه پدیده هایی نسبیتی رو بررسی کنیم، پدیده هایی که فقط توی نسبیتِ خاص معنی دارند.

یکی از اولین اختلافاتی که از نسبیتِ خاص خیلی توی چشم می‌زنه اینه که دیگه تبدیلِ زمان مثلِ گذشته به صورتِ
t=t’
نیست و از اون بدتر اینه که توی فرمولِ تبدیلِ زمان، مکان ظاهر شده! پس یک چیزهایی که ما قبلا راجع به زمان می‌دونستیم الان باید عوض شده باشه، یکی از این چیزها بحثِ «همزمانی» دو رویداد هستش:

ما در دو پستِ گذشته راجع به این حرف زدیم که اندازه گیری‌ها در دستگاهِ مختصات توسطِ ناظرانِ کوری انجام میشه که فقط رویدادهای نقطه ی حضورشون رو با ساعتهایی که «همزمان» شده اندازه گیری می‌کنند، این که این ساعتها چه طور همزمان بشن با نسبیتِ خاص کارِ سختی نیست، فرض کنید توی یک نقطه یک لامپ داریم و فاصله ی ناظری هم ازش d باشه اون موقع اگر ساعتِ ناظر روی d/c تنظیم شده و ثابت مونده باشه، ما لامپ رو روشن می‌کنیم و به هر ناظر می‌گیم به محضِ رسیدنِ نورِ لامپ، ساعتت رو روشن کن، این طوری ساعتِ همه ی ناظرها همزمان شده

(مثلا ساعتِ ناظر رو در 300000 کیلومتری از لامپ، روی 1 ثانیه، ساعتِ ناظرِی در 600000 کیلومتری از لامپ روی دو ثانیه و .... تنظیم می‌کنیم و میگیم هر وقت نورِ لامپ بهتون رسید ساعتتون رو روشن کنید حالا بعد از یک ثانیه ناظرِ اول ساعتش رو روشن میکنه بعد از دو ثانیه ناظرِ دوم و.... و این طوری همه ی ناظرها همزمان میشن)

اما آیا دو تا ناظری که توی یک دستگاهِ مختصاتِ ساعتِ همزمان دارند، آیا توی یک دستگاهِ مختصاتِ دیگه هم ساعتشون همزمان خواهد بود؟


خوب این سوال رو میشه یک نحوِ دیگه پرسید: اگر دو رویداد در یک دستگاهِ مختصات همزمان باشند، آیا در دستگاهِ مختصاتِ دیگری هم همزمان خواهند بود؟

با توجه به تبدیلاتِ لورنتس میشه خیلی سریع به این سوال جواب داد اما با اصولِ موضوعه هم می‌شه همین رو دید:

سه تا ناظر در نظر بگیرید (دقت کنید که منظورمون از ناظر یک شخصِ نشسته نیست، بلکه مجموعه ای از دستگاه‌های اندازه گیری هستش)

ناظرِ اولی درونِ یک واگنِ قطار قرار داره، مهم نیست سرعتش چه قدره، ناظر نسبت به واگن ِ قطار ساکنه، یک لامپ دقیقا وسطِ واگن وجود داره که ناگهان روشن می‌شه، حالا چون سرعتِ نور در هر دو جهت ثابته بنا بر این نورِ لامپ از دیدِ ناظرِ درونِ قطار «همزمان» به سر و تهِ واگن خواهد رسید.

حالا ناظری رو تصور کنید که روی زمینه و داره میبینه که واگن رو به جلو حرکت می‌کنه، با روشن شدنِ لامپ، ناظر از بیرون مشاهده می‌کنه که نور با سرعتِ ثابتِ c به عقب و جلو منتشر می‌شه (این ثابت بودنِ سرعتِ نور اصلِ ماست)، اما چون تهِ واگن به سمتِ نور میره و سرِ واگن از نور دور میشه، بنا بر این نور زودتر به تهِ واگن میرسه و دیرتر به سرِ واگن:

پس رویدادِ رسیدنِ نور به سر و تهِ واگن که از دیدِ یکی همزمان بود، از دیدِ دیگری نا همزمان هستش!

حالا ناظرِ سومی رو هم در نظر بگیرید که داره سریعتر از واگن حرکت می‌کنه، می‌تونید با تحلیل هایی شبیهِ تحلیلِ بالا ببینید که از دیدِ ایشون نور اول به سرِ واگن میرسه و بعد به تهِ واگن! یعنی حتی برای ایشون ترتیبِ زمانی رویدادها هم عوض شده!

پس همزمانی امری نسبی هستش و دو تا رویداد که توی یک دستگاهِ مختلف همزمان هستند لزوما در هر دستگاهِ دیگری همزمان نیستند.

این اتفاق مطلقا نسبیتی هستش و هیچ ما به ازای کلاسیکی نداره :)



همه ی این داستانها توی یک شکل:

http://www.phys.vt.edu/~takeuchi/relativity/notes/lightbulb.gif

مداد رنگیهام
11-26-2013, 11:38 PM
آقا گفتین که "دقت کنید که منظورمون از ناظر یک شخصِ نشسته نیست، بلکه مجموعه ای از دستگاه‌های اندازه گیری هستش"
ولی خب چیزی که می خوام بگم مربوط به ناظران زندس...
نسبیت خیلی موضوع جالبیه نه تنها به موقعیت و شرایط ناظر بستگی داره بلکه خود ناظرم خیلی میتونه تو نتیجه مطالعاتمون تاثیر داشته باشه که البته این کاملا بدیهیه.

مدت ها بود که این موضوع ذهنمو درگیر کرده بود که چرا مگس ها این قدر سریع واکنش نشون میدن اول اینطوری توجیه کردم که شامد سرعت انتقال پیامهای عصبی مگسها بالاست و همین باعث تسریع عملشون میشه تا اینکه این مطلب رو خوندم زمان برای حیوانات کوچک کند تر میگذرد(اگه دوس داشتین بخونینش) http://jahannews.com/vdcb9fbf5rhb80p.uiur.html

اینکه کند گذشتن زمان رو چی تعریف کنیم بحث خودش رو داره ولی چیزی که میخواستم بگم اینه که اگر به جای دستگاه 3 ناظر زنده داشته باشیم بسته به اینکه سه ناظرمون چه موجودی باشه در چه سنی باشه چه جنسیتی داشته باشه میتونیم نتایج متفاوتی رو ثبت کنیم یا حتی اگر یک نفر رو در سه موقعیت پست بالا قرار بدیم و آزمایش رو چند بار تکرار کنیم بسته به حالت روحی فردمون و شاید ساعتی از روز که داره آزمایش انجام میشه ممکنه نتیجه ها با هم اختلاف داشته باشه چیزی که خیلی ساده بهش میگیم خطای اندازه گیری و خیلی ساده از کنارش میگذریم درصورتی که شاید حقیقتی درون همین خطاهای اندازه گیری نهفته باشه.:)

حالا یه سوال یه ناظر انسان داریم یه ناظر مگس!:دی با فرض اینکه زمان برای مگس 4 برابر کند تر از انسان میگذره تحت چه شرایطی دو ناظر یکی از موقعیت های پست بالا رو به صورت همزمان درک میکنن؟!!:دی

Ehsan
11-28-2013, 11:25 PM
بعد از نسبیتِ همزمانی، بحثِ جالبِ دیگه بحثِ ساعتهای متحرک هستش که باز هم مربوط به زمان می‌شه

دیدید که طبقِ اصلِ نسبیت، فاصله ی زمانی بینِ روشن شدنِ لامپ تا رسیدنِ نورش به انتهای واگن، برای سه ناظرِ مختلف کاملا متفاوته، پس گذرِ زمان برای سه ناظر باید متفاوت باشه، این پدیده در کل در قالبِ بحثِ ساعتهای متحرک بیان میشه

اصلا بیاید یه ساعتِ این طوری بسازیم:


http://up.avastarco.com/images/zdscxyxmu6e49igc1cfz.png

دو تا آینه ی رو به رو در نظر بگیرید که ابتدا در دستگاهِ ساکن قرار دارند، بینِ این آینه ها یک نور حرکتِ رفت و برگشتی انجام می‌ده و با هر بار رفت و برگشت، ساعتِ ما تیک تاک می‌کنه، اگر فاصله ی آینه ها 1 باشه، هر تیک و تاک 2/c طول می‌کشه که c سرعتِ نور هستش. با شمردنِ این تیک تاکها می‌تونیم گذرِ زمان رو حس کنیم


حالا بیاید آینه ها رو در راستای عمود بر جهتِ حرکتِ نور، با سرعتِ V حرکت بدیم، این باعث میشه که نور به جای یک مسیرِ مستقیم، برای حرکتِ رفت و برگشتی یک حرکتِ مورب انجام بده:


http://up.avastarco.com/images/zn94h6s0rrbxab9f7fw.png

حالا مسیرِ حرکتِ نور به خاطرِ حرکتِ مورب بودنش طولانی تر شده، نور باید به جای 2 متر،


http://up.avastarco.com/images/me99axzw54lijn417jv4.png

متر رو طی کنه که چون خودِ L برابر است با سرعتِ نور ضرب در زمانِ تیک تاک، در نتیجه با حلِ معادله ی بالا بر حسبِ مدت زمان ِ تیک تاک خواهیم داشت:


http://up.avastarco.com/images/97vqroxhhxiiucpi4g.png


می‌بینید که تیک تاکِ این ساعت از دیدِ ما به اندازه ی ضریبِ بزرگتر از یکِ


http://upload.wikimedia.org/math/e/5/b/e5ba7993261a158ec4bf4ab49094b865.png

طولانی‌تر از تیک تاکِ همین ساعت در دستگاهِ سکونش هستش، یعنی اگر این ساعت در دستگاهِ سکونش هر ثانیه یک بار تیک تاک میکرد، حالا از دیدِ ناظری که با سرعتِ چهار پنجم سرعتِ نور نسبت بهش حرکت می‌کنه هر پنج ثانیه به جای پنج بار، سه بار تیک تاک می‌کنه! و این یعنی ساعتی که متحرکه، کندتر کار می‌کنه! یعنی اگر من نسبت به زمین با سرعتِ زیاد حرکت کنم، یک slow motion از زمینی ها مشاهده خواهم کرد چون همه ی ساعتهای زمینی باید کند کار کنند.

به این پدیده می‌گن اتساعِ زمان و فرمولی که به دست آوردیم هم همیشه برای همه ی سرعتها و همه ی ساعتها (چه مکانیکی چه نوری ) درسته.


این پدیده خیلی عجیبه و اصلا بدیهی نیست.

از این گذشته ظاهرا یک تناقض در این بین وجود داره، اگر ما می‌بینیم که ساعتِ متحرکی کند شده، پس از دیدِ اون ساعت ما باید سریعتر کار کنیم دیگه، اگر ما یک ناظری رو در حالِ slow motion میبینیم قاعدتا اون باید ما رو fast motion ببینه، یعنی اگر از دیدِ ما ساعتِ اون هر پنج ثانیه یک بار تیک تاک می‌کنه اون باید ببینه ما هر ثانیه پنج بار تیک تاک می‌کنیم! خوب حالا می‌گید تناقضِ داستان کجاست؟! مگه نگفتیم دستگاههای لخت نسبت به هم کاملا متقارن هستند و نمی‌شه بینشون تمایز قائل شد؟! این طوری که من توی یک دستگاه ساعتم کند کار میکنه تو اون یکی تند، پس من می‌تونم بینِ دو دستگاهِ ساکن و متحرک تفاوت قائل بشم

به نظرتون جوابش چیه؟


آقا گفتین که "دقت کنید که منظورمون از ناظر یک شخصِ نشسته نیست، بلکه مجموعه ای از دستگاه‌های اندازه گیری هستش"
ولی خب چیزی که می خوام بگم مربوط به ناظران زندس...
نسبیت خیلی موضوع جالبیه نه تنها به موقعیت و شرایط ناظر بستگی داره بلکه خود ناظرم خیلی میتونه تو نتیجه مطالعاتمون تاثیر داشته باشه که البته این کاملا بدیهیه.

مدت ها بود که این موضوع ذهنمو درگیر کرده بود که چرا مگس ها این قدر سریع واکنش نشون میدن اول اینطوری توجیه کردم که شامد سرعت انتقال پیامهای عصبی مگسها بالاست و همین باعث تسریع عملشون میشه تا اینکه این مطلب رو خوندم زمان برای حیوانات کوچک کند تر میگذرد(اگه دوس داشتین بخونینش) http://jahannews.com/vdcb9fbf5rhb80p.uiur.html

اینکه کند گذشتن زمان رو چی تعریف کنیم بحث خودش رو داره ولی چیزی که میخواستم بگم اینه که اگر به جای دستگاه 3 ناظر زنده داشته باشیم بسته به اینکه سه ناظرمون چه موجودی باشه در چه سنی باشه چه جنسیتی داشته باشه میتونیم نتایج متفاوتی رو ثبت کنیم یا حتی اگر یک نفر رو در سه موقعیت پست بالا قرار بدیم و آزمایش رو چند بار تکرار کنیم بسته به حالت روحی فردمون و شاید ساعتی از روز که داره آزمایش انجام میشه ممکنه نتیجه ها با هم اختلاف داشته باشه چیزی که خیلی ساده بهش میگیم خطای اندازه گیری و خیلی ساده از کنارش میگذریم درصورتی که شاید حقیقتی درون همین خطاهای اندازه گیری نهفته باشه.:)

حالا یه سوال یه ناظر انسان داریم یه ناظر مگس!:دی با فرض اینکه زمان برای مگس 4 برابر کند تر از انسان میگذره تحت چه شرایطی دو ناظر یکی از موقعیت های پست بالا رو به صورت همزمان درک میکنن؟!!:دی

راستش فیزیک دانان همیشه برای اندازه گیری از راههایی استفاده می‌کنند که بستگی به ناظرِ زنده نداشته باشه و اصطلاحا نتایجِ مستقل از ناظر و تکرار پذیر داشته باشه. یعنی یه سری راههای اندازه گیری قرص و محکم درست می‌کنند تا همه بتونن اندازه گیری کنند و در اندازه گیریش هم اتفاقِ نظر داشته باشند (مثلِ همین ساعتِ آیینه ای که ربطی به این نداره که ناظر مگسه یا آدم :دی)


در هر صورت مگس هم یک ناظرِ مشخص هست که گفتیم منظورِ ما از ناظر یه موجودِ زنده نیست :)

مداد رنگیهام
12-01-2013, 01:02 PM
خوب گذشت زمان کند یا تند نشده که،در واقع این ساعت ماست که داره کند کار میکنه یعنی این ساعت ماست که خراب شده مثله این میمونه که من یه آکواریم بزرگ داشته باشم بعد با ساعتم که ضد آب نیست برم تو آکواریم و چند ساعت بمونم و بعد بگم که تو آکواریم زمان متوقفه چون ساعت من عقربه هاش حرکت نکرده :دی بعد بیام نتیجه بگیرم که آکواریم و اتاقم دو تا دستگاه جدان که قوانین توشون صدق نمیکنه!:دی در حالی که این ساعت منه که با این شرایط درست کار نمیکنه:)

از طرفی میشه اصلاحش کرد تا ساعتمون زمان رو مثله حالت عادیش نشون بده یعنی بیایم تو ساعت آینه ایمون تغییری ایجاد کنیم که متناسب با افزایش سرعت حرکتمون ،فاصله بین دوتا آینه هم کم بشه تا همیشه مسافتی که نور باید طی کنه همون یک باشه البته این اصلاح تازمانی که سرعت حرکتمون کمتر از نصف سرعت نور باشه امکان پذیره چون وفتی دوتا آینه به هم برسن ما نمیتونیم فاصلشونو کمتر کنیم...

اینکه بگیم ساعتمون در حالت متحرک در هر ثانیه مثلا سه بار تیک تاک میکنه در حالی که در حالت ایستاده در هر ثانیه 5 بار تیک تاک میکنه رو نمیفهمم،این ثانیه چه مفهومی داره؟!مگه قرار نیست زمان رو از روی ساعتمون بفهمیم؟منظورم اینه که اگر در حالت ایستادمون فاصله بین یک تیک تاک رو یک ثانیه در نظر بگیریم در حالت حرکت هم فاصله یک تیک تاک (هر چند که طولانی تر از مقدار ایستادش باشه ) باز هم یک ثانیست فقط اینجا ساعتمون داره خراب کار میکنه.....:)

Ehsan
12-17-2013, 10:41 PM
خوب گذشت زمان کند یا تند نشده که،در واقع این ساعت ماست که داره کند کار میکنه یعنی این ساعت ماست که خراب شده مثله این میمونه که من یه آکواریم بزرگ داشته باشم بعد با ساعتم که ضد آب نیست برم تو آکواریم و چند ساعت بمونم و بعد بگم که تو آکواریم زمان متوقفه چون ساعت من عقربه هاش حرکت نکرده :دی بعد بیام نتیجه بگیرم که آکواریم و اتاقم دو تا دستگاه جدان که قوانین توشون صدق نمیکنه!:دی در حالی که این ساعت منه که با این شرایط درست کار نمیکنه:)

از طرفی میشه اصلاحش کرد تا ساعتمون زمان رو مثله حالت عادیش نشون بده یعنی بیایم تو ساعت آینه ایمون تغییری ایجاد کنیم که متناسب با افزایش سرعت حرکتمون ،فاصله بین دوتا آینه هم کم بشه تا همیشه مسافتی که نور باید طی کنه همون یک باشه البته این اصلاح تازمانی که سرعت حرکتمون کمتر از نصف سرعت نور باشه امکان پذیره چون وفتی دوتا آینه به هم برسن ما نمیتونیم فاصلشونو کمتر کنیم...



واقعیت اینه که ساعتِ ما هر طوری کار بکنه (مکانیکی، آونگ، نوری، نوسانِ کوارتز، رزونانسِ اتمی و.....) باز هم همین اتفاق می‌افته، یعنی ساعتی که حرکت می‌کنه از دیدِ ناظرینِ ساکن کندتر حرکت می‌کنه




اینکه بگیم ساعتمون در حالت متحرک در هر ثانیه مثلا سه بار تیک تاک میکنه در حالی که در حالت ایستاده در هر ثانیه 5 بار تیک تاک میکنه رو نمیفهمم،این ثانیه چه مفهومی داره؟!مگه قرار نیست زمان رو از روی ساعتمون بفهمیم؟منظورم اینه که اگر در حالت ایستادمون فاصله بین یک تیک تاک رو یک ثانیه در نظر بگیریم در حالت حرکت هم فاصله یک تیک تاک (هر چند که طولانی تر از مقدار ایستادش باشه ) باز هم یک ثانیست فقط اینجا ساعتمون داره خراب کار میکنه.....:)

شما این جا جوابتون رو خودتون دادین:

(هر چند که طولانی تر از مقدار ایستادش باشه )

طولانی تر نسبت به چی؟ به کدوم ساعت؟ واضحه! ساعتهای ثابتِ روی زمین که تیک تاکشون یک ثانیه هستش! یعنی ما دقیقا یک ساعت مشابهِ اون ساعت داریم که معیارِ ما تیک تاکِ اون ساعت هستش و با اونه که زمان رو می‌سنجیم.

حالا برای این که ببینید مشکل از ساعتِ خرابِ ما نیست، آزمایش رو جورِ دیگه انجام بدیم، کنارِ آینه ی پایینی یک شخص نشسته که با هر بار رسیدنِ نور به آینه ی کنارِ شخص، با یک چکش یک ضربه روی یک میز می‌زنه، اگر نرخِ برخوردِ نور به آینه ها با افزایشِ سرعت کاهش پیدا کنه، ما باید مشاهده کنیم که بازه های زمانی بینِ ضرباتِ چکشِ شخصِ متحرک طولانی تر شده، یعنی شخص کندتر ضربه می‌زنه، حالا تناقضِ داستان اینجاست، اگر ما شخصِ رو در یک «دورِ کُند» تماشا می‌کنیم طبیعاتا اون هم باید ما رو در یک دورِ تند تماشا کنه! اما این نقضِ نسبیت هستش چون ما می‌تونیم این طوری بفهمیم که کدوم دستگاه ایستاده و کدوم دستگاه داره حرکت می‌کنه، در واقع اتفاقی که می‌افته اینه که شخصِ چکش به دست هم ما رو کندتر از حالتِ عادی خواهد دید (اون ما رو روی دورِ کند می‌بینه) حالا سوال اینه که چرا؟

kkk
12-18-2013, 09:25 PM
فکر کنم اینجا عامل اصلی اندازه گیری زمان . یعنی رسیدن نوری که نشون میده ساعت نوسان دیگه ای کرده در نظر گرفته نمیشه .
توی فرموی که احتمالا به نظر میاد این تناقض از اون نشات میگیره یعنی :http://up.avastarco.com/images/97vqroxhhxiiucpi4g.png (http://up.avastarco.com/images/97vqroxhhxiiucpi4g.png)

احتمالا تناقض اینه که الان اگه تغییرات زمانی ناظر اولیه رو بخایم بدست بیاریم میشه تغییرات زمانی اولی تقسیم بر گاما که اینجوری زمان ناظر دوم تند تر درمیاد .
ولی باید گفت که تغییرات زمانی در نظر گرفته شده در این معادله صرفا تعداد تیک تاک کردن ساعت ناظر دوم از نظر خودش هست و نه بازه ای که برای تیک تاک اولی در نظر گرفته .

Ehsan
12-20-2013, 04:23 PM
فکر کنم اینجا عامل اصلی اندازهگیری زمان . یعنی رسیدن نوری که نشون میده ساعت نوسان دیگه ای کرده در نظر گرفته نمیشه .
توی فرموی که احتمالا به نظر میاد این تناقض از اون نشات میگیره یعنی :http://up.avastarco.com/images/97vqroxhhxiiucpi4g.png

احتمالا تناقض اینه که الان اگه تغییرات زمانی ناظر اولیه رو بخایم بدست بیاریم میشه تغییرات زمانی اولی تقسیم بر گاما که اینجوری زمان ناظر دوم تند تر درمیاد .
ولی باید گفت که تغییرات زمانی در نظر گرفته شده در این معادله صرفا تعداد تیک تاک کردن ساعت ناظر دوم از نظر خودش هست و نه بازه ای که برای تیک تاک اولی در نظر گرفته .

فکر کنم دارین جوابِ درست رو می‌دین، فقط جملاتِ آخرتون رو باید یه کمی دقیقتر بیان کنید، کمی با تفضیلِ بیشتر و کمی هماهنگ با زبانِ نسبیتِ خاص :)

Ehsan
12-26-2013, 08:43 PM
خوب جوابِ تناقض رو تقریبا دادند فقط کمی با زبانی متفاوت بیانش می‌کنم:

داستان از این قرار بود که اگر ما ناظرِ متحرکی رو کندتر مشاهده می‌کنیم طبیعتا اون باید ما رو تندتر تماشا کنه اما واقعیت اینه که اگر این اتفاق بی‌افته همگنی و همسانگردی نقض میشه و در واقعیت اتفاقی که می‌افته اینه که اون هم ما رو کندتر مشاهده میکنه! چرا؟

بیاید دستگاهها رو اسم بگذاریم: دستگاهِ 1 و دستگاهِ 2 . ساعت در دستگاهِ 2 ساکنه و در دستگاهِ یک متحرک

کلیدِ بازی در تعریفِ رویدادهاست: شما در دستگاهِ 2، چندین رویدادِ متوالی دارین که در یک مکان اما با فاصله ی زمانی ثابت اتفاق می‌افتن (تیک تاکِ ساعت: tik tak tik tak.....) حالا می‌رید به یک دستگاهِ دیگه و تیک تاکِ اون ساعتها رو تماشا می‌کنید و میبینید که فاصله ی زمانی (تاکید می‌کنم تنها فاصله ی زمانی) بینِ تیک تاکها بیشتر شده (کندتر شده) (tiiik.....taaaak.....tiiiiiiiiik.....taaaaaak) اما فراموش نکنید که تیکِ اول در یک مکان و تیکِ دوم در مکانی دیگه اتفاق می‌افته چون ساعت متحرکه، و فراموش نکنید همچنان منظور از ناظر همون مجموعه ی رباطها هستند.

این تمامِ اتفاقی بود که ناظرِ دستگاهِ یک مشاهده می‌کنه، اما دستگاهِ دو چه طور می‌تونه راجع به گذرِ زمان در دستگاهِ یک اظهارِ نظر کنه؟ باید یک ساعت رو تماشا کنه و تیک تاکهاش رو بسنجه، پس این ناظر هم داره راجع به یک سلسله رویدادِ متوالی در دستگاهِ دو اظهارِ نظر میکنه، در واقع دو ناظرِ متفاوت دارند راجع به دو رویدادِ متفاوت که هیچ ربطی به هم ندارند اظهارِ نظر میکنند! پس لزومی نداره که به جهتِ کندِ شدنِ ساعت یک از دیدِ 2، ساعتِ 2 از دیدِ یک تندتر بشه و برعکس، چون این دو رویداد به هم بسته نیستند و مستقل اند.

اگر همچنان قبولش براتون سخته از دیدگاهی که راجع به زمان دارید باید دست بکشید :دی همینه که وقتی از اینشتین میپرسن زمان چیه میگه چیزی که ساعتِ مچی من نشون میده (یعنی زمان یک چیزِ شخصیه)


ــــــــــــــــــ
دیگه نوبتِ پاراداکس دوقولو هاست :)

violet20
12-27-2013, 03:57 PM
ببخشید این قانون لورنتس رو میشه بیشتر توضیح بدین ممنون میشم

Ehsan
12-27-2013, 04:40 PM
ببخشید این قانون لورنتس رو میشه بیشتر توضیح بدین ممنون میشم

کدوم قانونِ لورنتس؟ (یکی دو تا نیستن آخه :دی)

narcissus flower
12-29-2013, 02:53 AM
خب من پارادوکس دوقلو ها رو شروع میکنم.



فرض کنیم که در روی کره ی زمین دو قلوی متولد بشه و در همین موقع هم در یک نقطه ی دیگر از جهان(فرض کنید یک ستاره) یک کودک دیکر متولدبشه .این دو نقطه از جهان رو در یک چارچوب در نظر میگیریم که در آن ساکن هستند .خب?!

ولی فکر کنید که وقتی دوقلوها به دنیا اومدند ما یک سیگنال فرستادیم به اون ستارهه که بچه ی دیگه به دنیا اومده .خب سرعت سیگنال ما محدود هست دیگه و طول میکشه که به مقصد برسه .مثلا فرض کنید 49 سال! یعنی 49 سال نوری .یعنی وقتی سیگنال به ستاره میرسه , اون کودک سنی ازش گذشته.
خب تا اینجا که هیچ چی...بقیه ش جالبه!!! :)

یکی از این دوقلوها نیت میکنه که بره یه سر به اون ستارهه بزنه.مثلا در سن 20 سالگی. دقت کنید وقتی میگم 20 سالگی یعنی هر سه گزینه 20 سال دارند.میان میگن که خب ببینیم اگر بخواهیم بریم به اون ستاره , چه مسافت و چه زمانی رو باید صرف کنیم.؟؟

میاید یک خطکش میزاره میگه خب فاصله که فلان قدره .و زمان هم مثلا اگر با سرعت نزدیک به نور حرکت کنه مثلا 50 سال طول میکشه...(دقت کنید)
خیلی خب ...

حالا این قل سوار سفینه ش میشه و شروع به حرکت میکنه...برای اینکه به اون سرعت ثابت برسه , نیاز داره که یک بازه ی زمانی حرکت شتابدار رو تجربه کنه...این دوقلو ها قبل از سفر ساعتشاشون رو با هم کوک میکنند.

سفر اغاز میشه....حالا قل قصه ی ما داره سرعتی نزدیک سرعت نور(مثلا0.98سرعت نور)
رو تجربه میکنه.اگر به موضوع اتساع زمان و انقباض طول توجه کنید , فکر کنم ماجرا دستتون بیاد که چه جوریه.
اگر ساعتها رو نگاه کنید میبینید که این دو دیگه کوک هم نیستند.یعنی ساعت قل در حال سفر از ساعت قل روی زمین جا میمونه( اینو قل روی زمین میگه) ولی قل در حال حرکت فکر میکنه که همه چیز عادی هست.

از طرفی قل درحال سفر میاد دوباره خط کش میزاره و مسافتی رو که قراره طی کنه رو اندازه میگیره.....

ایا همون مقدار قبلی روکه رو زمین تخمین زده به دست میاره؟؟
به نظر خودش چه قدر تو راه بوده؟؟به عبارتی وقتی میرسه به ستاره چندسالشه؟؟
یه بچه ای هم تو اون ستاره زندگی میکنه.وقتی قل میرسه اونجا,بچه ه چند سالشه؟؟ :دی

m.Sadat
12-29-2013, 06:33 PM
خب من پارادوکس دوقلو ها رو شروع میکنم.

.
.
.

سفر اغاز میشه....حالا قل قصه ی ما داره سرعتی نزدیک سرعت نور(مثلا0.98سرعت نور)
رو تجربه میکنه.اگر به موضوع اتساع زمان و انقباض طول توجه کنید , فکر کنم ماجرا دستتون بیاد که چه جوریه.
اگر ساعتها رو نگاه کنید میبینید که این دو دیگه کوک هم نیستند.یعنی ساعت قل در حال سفر از ساعت قل روی زمین جا میمونه( اینو قل روی زمین میگه) ولی قل در حال حرکت فکر میکنه که همه چیز عادی هست.

از طرفی قل درحال سفر میاد دوباره خط کش میزاره و مسافتی رو که قراره طی کنه رو اندازه میگیره.....

ایا همون مقدار قبلی روکه رو زمین تخمین زده به دست میاره؟؟
به نظر خودش چه قدر تو راه بوده؟؟به عبارتی وقتی میرسه به ستاره چندسالشه؟؟
یه بچه ای هم تو اون ستاره زندگی میکنه.وقتی قل میرسه اونجا,بچه ه چند سالشه؟؟ :دی

البته پارادوکس دوقولو ها از اینجا شکل نمیگیره و از اونجایی بهش میگن پارادوکس که ناظر اول میبینه که ناظر موجود در سفینه با سرعت زیادی از او دور میشه پس ساعت اون کند تر کار میکنه و باید جوان تر به نظر برسه و ناظر توی سفینه هم میبینه که ناظر اول با سرعت زیادی از او دور میشه پس اون جون تر به نظر میرسه
حال سوال اینه که کدوم یک از دو ناظر واقعا جوان تر است ؟:دی

narcissus flower
12-29-2013, 08:46 PM
جناب اندکی صبر..بزارید با حوصله پیش بریم(البته با حضور شما)

اول باید بستر رو ایجاد کنیم , بعد اصل ماجرا رو بیان کنیم
خب بیاییم قلی که رو ی زمینه 1 در نظر بگیریم.چار چوب مرجعی رو در نظر بگیریم s1. در این چارچوب 1 کلا ساکن هست .و در طول سفر قلش (فرد2) همین جوری نشسته.از نظرفرد1 سفر رفتن قلش 50 سال (مثلا) طول میکشه چون میدونه برادرش با سرعت 0.98 سرعت نور قرار اون مسافت رو طی کنه....اما قلش چون خودش باسرعتی نزدیک به نور داره حرکت میکنه, این سفر رو از دید خودش 10 ساله طی میکنه.چون دچار اتساع زمان شده است...یعنی بره و برگرده 20 سال طول میکشه .اما این سفر از دید برادرش 100 سال طول میکشه تا بره و برگرده
خب اینجوریه؟؟
یعنی وقتی دو برادر , همدیگر رو مبینند اختلاف سنی وجود داره؟ جواب اینه که قل در حال حرکت مثل اینکه جوونتره ...واقعا؟؟ کی گفته؟؟

تصور اینکه فرد2 با سرعت از زمین دور بشه با اینکه زمین با سرعت ازش دور بشه , هم ارزه.
فرد تو سفینه فکر میکنه که من الان دارم میبینم که زمین وداداشم دارند باسرعت از من دور میشوند .واینکه ا....ساعتش داره کندترمیزنه .پس داداشم از من جوونتره . چون خودش رو در دستگاه سفینه ساکن در نظر میگیره و از این دیدگاه میگه داداشم از من جوونتر خواهد بود...

خب اون یکی هم میگه که چون سفینه داره با سرعت نور میره ...پس فرد 2 جوانتر خواهد بود..
حالا باید بپرسیم:
اینجوری که معلومه دوتای شون فکر میکنند برادرشون جوونتر خواهد ماند .کی درست میگه؟؟ ایا هر دوتاشون درست میگویند؟؟
(به نظر حضور ناظر و چارچوب داره نقش مهمی رو بازی میکنه...)


دوقلوها هم حتما پسر هستند , چون طبق یک اصل فیزیکی سن خانمها از 20 کلا تغییر نمیکنه :دی

narcissus flower
01-15-2014, 01:23 AM
خب به اینجا رسیدیم که کدوم جوون تر میمونند؟؟
پاسخ این هست که شخصی که مسافرت کرده جوان تر باقی میمونه به همون دلایلی که گفتیم یعنی ساعتش دچار اتساع زمان میشه و از ساعت ناظر روی زمین عقب میمونه . و چون زمان عمرش داره با اون ساعت میگذره بنابراین از ادم زمینی عمر کمتری رو گذرونده اما شاید این ابهام پیش بیاد خب چرا مسافر حق نداره بگه من در ساکن هستم در حالیکه ادم زمینی داره حرکت میکنه؟؟

نکته ی خوشگل قضیه به نظر من همین جاست...

مسافر ما برای شروع سفر نیاز داره یک حرکت شتاب دار رو در اول سفر تجربه کنه .بعد از این مرحله یک حرکت یکنواخت رو تا نزدیک مقصد طی میکنه بنابراین در این جا هم یک بار دیگه حرکت شتاب دار داره تا سرعتش رو تغییر بده تا بتونه دور بزنه و دوباره حرکت یکنواخت رو داره تا به زمین برسه و باز تغییر سرعت و شتاب منفی و اینا...
با این اوصاف فکر نمیکنم مسافره دیگه فکر کنه که در حال سکون هست و زمین داره دور میشه...پس داره حرکت میکنه

از طرفی ساعت ها هم در این مورد درگیر میشوند...

وقتی ساعت ها رو بررسی میکنیم .مسافر میبینه وقتی داره از زمین دور میشه چون از منبع ساعت زمینی دور میشه اونو پالس ها بهش اهسته میرسند اما وقتی برمیگرده چون داره به طرف پالس ها نزدیک میشه میبینه ساعت زمینی داره تندتر میزنه چون داره به طرف منبع میره...اما ناظر زمینی باید اول متوجه دور زدن مسافر بشه بعد ببینه ساعتش داره تند میزنه و این درک با تاخیر کسب میشه.
پس در حالیکه مسافر نصف سفر رو پالس کند و نصف دیگه رو پالس سریع دریافت میکنه , ناظر زمینی بیش از نیمی از زمان خود ش پالس اهسته از مسافر دریافت کرده و قسمتی رو هم پالس تند...پس ناظر زمینی زمان کوتاه تر و عمر کمتری را برای مسافر برحسب ساعت همراه مسافر تخمین خواهد زد. ...

شاید لازم باشه کمی خودتون هم تفکر کنید چون نسبیت رو باید بافکر کردن متوجه شد, واینکه استدلال های مختلفی از دیدگاه های برای این موضوع بیان میشه ...

narcissus flower
01-15-2014, 01:35 AM
حالا یه موضوع جالب تر...

یه ذره ی باردار مثل یه توپ کوچولو در نظر بگیرید.درحال سکون باشه. میدانش اینجوری هست.یه میدان کولنی متقارن و ساده .یعنی اگر خطوط میدان رو رسم کنیم در هیچ جهتی ارجحیتی وجود نداره که این حالت رو به هم بریزه.

حالا این ذره شروع به حرکت میکنه و سرعتی رو کسب میکنه. اگر بخواهید میدان اطرفش رو رسم کنیم چه جوری خواهد بود؟

http://irphy.com/eni-media/2012/02/2e.gif

Ehsan
02-03-2014, 05:58 PM
حالا یه موضوع جالب تر...

یه ذره ی باردار مثل یه توپ کوچولو در نظر بگیرید.درحال سکون باشه. میدانش اینجوری هست.یه میدان کولنی متقارن و ساده .یعنی اگر خطوط میدان رو رسم کنیم در هیچ جهتی ارجحیتی وجود نداره که این حالت رو به هم بریزه.

حالا این ذره شروع به حرکت میکنه و سرعتی رو کسب میکنه. اگر بخواهید میدان اطرفش رو رسم کنیم چه جوری خواهد بود؟

http://irphy.com/eni-media/2012/02/2e.gif

نمی‌دونم والا! شهودا انتظار داریم در راستای حرکت خطوط میدان فاصله بگیرن و میدان ضعیفتر بشه.

gandom
02-03-2014, 09:37 PM
میدانش رو هم میشه بگین که از چه نوعیه؟

مداد رنگیهام
02-04-2014, 12:08 AM
یه چیزی بگم؟الکترون خودش میدان الکتریکی داره حالا وقتی حرکت کنه میدان مغناطیس هم پیدا میکنه و میشه شبیه نور...
آیا داشتن این دو میدان در الکترون متحرک میتونه دلیل رفتار موجی الکترون باشه؟یا بهتر بگم آیا الکترون در حالت سکون باز هم خواص موجی رو نشون میده؟
.......................
نرگس جان ببخشید روند رو بهم زدم یه لحظه جرقه زد...

narcissus flower
02-04-2014, 04:33 PM
بیایین از این جا شروع کنیم که یک ذره ی باردار داریم. و یک چارچوب سکونی که ذره در آن هست. در حالت چه میدانی میبینیم ؟یک میدان الکتریکی (میدان کولنی)و از میدان الکترو مغناطیس خبری نیست .چون ذره ساکن هست.
حالا یک چارچوب دیگری رو در نظر بگیریم که در این چارچوب ذره با یک سرعتی مثل v حرکت میکنه...فرض کنیم برای سادگی , حرکت ما در راستای محور x باشه.( فقط برای سادگی )
در این حالت دیگه فقط میدان الکتریکی نداریم , بلکه مغناطیسی هم داریم چون داریم ذره رو حرکت میدیم...
اما میدان دیگه به اون صورتی که همیشه میخوندیم نیست یه خورده فرق داره
در این جا داریم
http://up.avastarco.com/images/u2dx83vbp5igcvayxlhr.png
هست و میدان مغناطیسی هم با یک ظریب v/c به دست میاد.
اما این چیزی که برای من جالب بود این زاویه تتا هست.تتا زاویه ی ببین خطوط میدان با محوری مثل x هست که ما داریم در اون راستا میریم
یعنی چی؟ یعنی شدت میدان هم الکتریکی و هم مغناطیسی وابسته به زاویه هست.
حالا چی میشه ؟
بیایید زاویه رو صفر بزاریم , چی شد؟
وبعد زاویه رو بزرگ کنیم مثلا بزاریم 90 درجه .حالا چی شد؟

یعنی در صفحه ی عمود به سرعت و حرکت بیشترین مقدارو در جلو و عقب حرکت , کمترین مقدار.
در واقع تصور من اینه که در حال سکون خطوط میدان در اطرافش کروی و متقارن هست و وقتی حرکت میکنه یک حجم بیضوی (تخم مرغی شکل به خودش میگیره)

یعنی یه جورایی انقباض طول داریم...
و طول هایی که امتدادشون عمود برراستای حرکت نسبی باشه دچار انقباض طول نمیشوند...:دی

narcissus flower
02-04-2014, 04:44 PM
یه چیزی بگم؟الکترون خودش میدان الکتریکی داره حالا وقتی حرکت کنه میدان مغناطیس هم پیدا میکنه و میشه شبیه نور...
آیا داشتن این دو میدان در الکترون متحرک میتونه دلیل رفتار موجی الکترون باشه؟یا بهتر بگم آیا الکترون در حالت سکون باز هم خواص موجی رو نشون میده؟
.......................
نرگس جان ببخشید روند رو بهم زدم یه لحظه جرقه زد...

خواهش میکنم خانم
ولی مونا جان با این حرفت که چون میدان مغناطیسی داریم پس ذره فوتونی میشه خیلی موافق نیستم...البته سوال جالبیه
ولی فقط میخوام ی موضوعی رو بگم
برداشت من اینه که هر موجودی این شرایط رو در فضا ایجاد میکنه لزوما فوتون نیست.مثلا شما یک اهنربا فرض کن داری.خب میدان مغناطیسی که داره معلومه .حالا شروع کن به تکون دادنش ....چه طور میشه؟در فضای اطراف میدان الکتریکی هم ایجاد میشه...اصلا یک مدار بزار میبینی جریان درست میشه.
یعنی الان موجودی داریم که هم میدان الکتریکی داره درست میکنه و مغناطیسی ...حالا با حرف شما این اهنربا رو من فوتون بگیرم؟؟

بزار یه خورده فکر کنیم:دی

مداد رنگیهام
02-04-2014, 09:41 PM
حرف شما متین ولی بحث ، بحثِ ذره است:)
خب بذار یه جور دیگه بپرسم موج الکترو مغناطیسی از دو میدان عمود بر هم الکتریکی و مغناطیسی ایجاد شده درسته الکترون هنگام حرکت دارای میدان الکتریکی و مغناطیسی میشه آیا میدان ها در الکترون مثل امواج عمود بر همه؟آیا الکترون در حالت سکون خاصیت موجی از خودش نشون میده؟
اصلا آقا الکترون ساکن موجودیت داره؟یا مثل فتون در حال سکون جرم سکون براش مفهومی نداره؟:)

narcissus flower
02-05-2014, 08:56 PM
افرین !نکته ی مهمی رو اشاره کردی...پس اول باید بگیم در چه مقیاسی این ادعا رو میخواهیم بکنیم...
در مقیاس میکروسکوپی ...
خب ما برای الکترون جرم سکون تعریف میکنیم ولی اینکه همین جوری یک الکترون رو توی بشقاب بگذاریم من جایی ندیدم...تو اتم که کاملا در حال حرکت هست و مغناطیس هم داره...
چون در هر حالتی الکترون یک تکان هایی داره(نوسان و چرخش داره) پس یه موجود ثابت باشه رو من ندیدم...
حتی در صفر کلوین هم ...

از طرفی بیا از دید عدم قطعیت هایزنبرگ به قضیه نگاه کنیم...
اگر الکترون ثابت سر جاش باشه پس محل و تکانه اش معلومه پس کل عدم قطعیت میرود زیر سوال چون به محض اینکه بخواهیم در یک ویژگی اش دقت کنیم باید فوتونی به سمتش برود و به محض اینکه فوتون بهش برخورد کنه همه ی سیستم تغییر میکنه و الکترون دیگه سرجاش نیست....

در مورد موجی بودنش هم باز به نظر من اینکه ما اونو مثل یک فوتون بگیریم خیلی درست نیست چون ماهیت و ساختارش با نور فرق داره (بالاخره یک موجود مادی هست)...درسته که موج داریم ولی اینکه عین فوتون باشه من بعید میدانم...ولی فکر کنم میدان هاش بر هم عمود هستند...

تو حتی در مورد فوتون هم نمیتونی بگی کاملا موجی هست بسته به شرایط میتونه کاملا ذره باشه (پدیده هایی وجود داره که اثبات کننده این موضوع هست)

پس ابزار اندازه گیری و نحوه ی مشاهده و شرایط میتونه در پاسخ به این موضوع اثر گذار باشه

این چیزایی که من بلد بودم اگر مطلب جدیدی یافتم عرض خواهم کرد...ولی عدم قطعیت فکر کنم توجیه قابل قبولی باشه:)

مداد رنگیهام
04-29-2014, 09:04 PM
اتر برگشته؟! :دی
http://www.hupaa.com/20140429092937001/جهان-یک-ابرشاره-مایع-است

fatemeh.zar
04-30-2014, 06:06 PM
سلام
آقا من یه چند تا سوال داشتم می پرسم:

1- تو کتاب رزنیک درباره نسبیت اون اولش که میخواسته درباره مکانیک نیوتنی صحبت کنه و تبدیلات گالیله رو توضیح بده این جمله رو نوشته: " رویداد چیزی است که مستقل از چارچوب مرجعی که برای توصیف آن به کار میرود، اتفاق می افتد." کلا با این تعریف یکمی مشکل دارم!! یعنی چی؟ آیا حرکت یه رویداد حساب نمیشه؟؟

2- فرض کنید یه ناظر ساکن توی مبدا داریم. یه ناظر دیگه داره با سرعت نور حرکت میکنه و در لحظه ی صفر رو مبدئه و بعد ازش دور میشه. آیا ناظر اول بعد از زمان صفر امکان ارتباط با ناظر دوم رو داره؟ یعنی فوتونی که در لحظه ی غیر صفر از مبدا خارج شده میتونه به ناظر دوم برسه؟؟ (با توضیحات لطفا!)

3-از کجا میتونیم بفهمیم چارچوب ما لخته یا نه؟ طیق تعریف، به چارچوبی که در اون قانون اول نیوتن صادق باشه لخت میگویند. خب، از کجا می فهمیم که ما (چارچوبمون) شتاب نداریم؟ شاید به اون جسمی که به عنوان ذره آزاد در نظر گرفتیم، نیرو وارد میشه و ما هم شتاب داشته باشیم، در نتیجه قانون اول نیوتن فقط ظاهرا برقرار باشه.

4-آیا شتاب هم یک چیز(!) نسبی است؟؟ اصل هم ارزی در این باره چی میگه؟ اگر نسبی هست، پس چرا ما چارچوبی که نسبت به ستارگان ثابت باشه رو لخت می گیریم؟ در واقع این فرضی بود که نیوتن انجام داد. انیشتین هم همین را میگوید؟؟

5- تعریف نیرو چیه؟ قانون دوم نیوتن؟ اگه قانون دوم هست، این یعنی ما نیرو رو با استفاده از چارچوب لخت تعریف کردیم (که توش قانون دوم نیوتن صادقه) و از طرف دیگه ما چارچوب لخت رو هم با قانون اول نیوتن که در واقع یه حالت خاص از قانون دومه تعریف کردیم. مثل این میمونه که ما بگیم به هر جا که روش میخوابیم بگیم تخت. و خواب رو روی تخت بودن تعریف کنیم!!!!:28: هنگ کردم اساسی!! کمک لطفا:help:

Ehsan
04-30-2014, 09:25 PM
سلام
آقا من یه چند تا سوال داشتم می پرسم:

1- تو کتاب رزنیک درباره نسبیت اون اولش که میخواسته درباره مکانیک نیوتنی صحبت کنه و تبدیلات گالیله رو توضیح بده این جمله رو نوشته: " رویداد چیزی است که مستقل از چارچوب مرجعی که برای توصیف آن به کار میرود، اتفاق می افتد." کلا با این تعریف یکمی مشکل دارم!! یعنی چی؟ آیا حرکت یه رویداد حساب نمیشه؟؟

حرکت به خودی خود رویداد حساب نمی‌شه، حضورِ ذره در یک نقطه رویداد محسوب می‌شه، مثلا اگر ذره از دیدِ ناظری در زمانِ 3 ثانیه در مبدا باشه این یک رویداد هستش با مختصاتِ (0,0,0,3s) و این یک رویداده، اما حرکت یک سلسله از رویدادهاست که کنارِ هم قرار گرفته‌اند، نه یک رویدادِ تنها


2- فرض کنید یه ناظر ساکن توی مبدا داریم. یه ناظر دیگه داره با سرعت نور حرکت میکنه و در لحظه ی صفر رو مبدئه و بعد ازش دور میشه. آیا ناظر اول بعد از زمان صفر امکان ارتباط با ناظر دوم رو داره؟ یعنی فوتونی که در لحظه ی غیر صفر از مبدا خارج شده میتونه به ناظر دوم برسه؟؟ (با توضیحات لطفا!)


چرا باید بتونه؟ هر سیگنالی که بفرسته با سرعتِ نور حرکت خواهد کرد پس از دیدِ ناظرِ ساکن سیگنال هیچ وقت بهش نمی‌رسه



3-از کجا میتونیم بفهمیم چارچوب ما لخته یا نه؟ طبق تعریف، به چارچوبی که در اون قانون اول نیوتن صادق باشه لخت میگویند. خب، از کجا می فهمیم که ما (چارچوبمون) شتاب نداریم؟ شاید به اون جسمی که به عنوان ذره آزاد در نظر گرفتیم، نیرو وارد میشه و ما هم شتاب داشته باشیم، در نتیجه قانون اول نیوتن فقط ظاهرا برقرار باشه.

این شتاب نسبت به چی سنجیده میشه؟ اگر در دنیایی باشید که فقط شما باشید و اون ذره این نگرانی نابجاست چون چیزی غیر از شما وجود نداره که شما نسبت به اون شتاب داشته باشید، اما در دنیای ما کلی جرم وجود داره که می‌شه با نگاه به اونها فهمید شتاب داریم یا ساکن هستیم. تبصره: گرانش از این قاعده مستثناست، یعنی اشخاصی که توی ایستگاه فضای احساسِ بی وزنی می‌کنند و مرجعِ لخت محسوب میشن ولی در واقع همگی با شتابِ 8 متر بر مجذورِ ثانیه در حالِ سقوط هستند! اما در این موردِ اصلِ هم ارزی اینشتین به ما می‌گه این افراد می‌تونن تمامی قوانینی رو که در مرجعِ لخت درسته برای خودشون درست بدونن (این یک اصله، واضحه که می‌شه زیرِ سوال بردش)




4-آیا شتاب هم یک چیز(!) نسبی است؟؟ اصل هم ارزی در این باره چی میگه؟ اگر نسبی هست، پس چرا ما چارچوبی که نسبت به ستارگان ثابت باشه رو لخت می گیریم؟ در واقع این فرضی بود که نیوتن انجام داد. انیشتین هم همین را میگوید؟؟


بله شتاب نسبی‌ست (بالا گفتم) و اصلِ هم ارزی هم دقیقا همین رو میگه


5- تعریف نیرو چیه؟ قانون دوم نیوتن؟ اگه قانون دوم هست، این یعنی ما نیرو رو با استفاده از چارچوب لخت تعریف کردیم (که توش قانون دوم نیوتن صادقه) و از طرف دیگه ما چارچوب لخت رو هم با قانون اول نیوتن که در واقع یه حالت خاص از قانون دومه تعریف کردیم. مثل این میمونه که ما بگیم به هر جا که روش میخوابیم بگیم تخت. و خواب رو روی تخت بودن تعریف کنیم!!!!:28: هنگ کردم اساسی!! کمک لطفا:help:


نه چاچوبِ لخت رو با قانونِ دوم نمی‌فهمیم با قانونِ اول می‌فهمیم، درسته قانونِ اول حالتِ خاص ِ قانونِ دومه ولی در واقع قانونِ اوله! یعنی بدونِ ارجاع به قانونِ دومِ نیوتون هم معتبره و استقلال داره به اصطلاح واسه خودش قانونیه! بنا بر این ما مرجعِ لخت رو با قانونِ اول می‌شناسیم. این پست رو هم نگاهی بندازید شاید ابهامتون کم شد:

http://forum.avastarco.com/forum/showthread.php?1443-%D9%81%D9%8A%D8%B2%D9%8A%D9%83-%D8%AC%D8%AF%D9%8A%D8%AF&p=64611&viewfull=1#post64611

fatemeh.zar
05-02-2014, 10:02 AM
حرکت به خودی خود رویداد حساب نمی‌شه، حضورِ ذره در یک نقطه رویداد محسوب می‌شه، مثلا اگر ذره از دیدِ ناظری در زمانِ 3 ثانیه در مبدا باشه این یک رویداد هستش با مختصاتِ (0,0,0,3s) و این یک رویداده، اما حرکت یک سلسله از رویدادهاست که کنارِ هم قرار گرفته‌اند، نه یک رویدادِ تنها

خب وقتی رویدادها هر کدوم مستقل از دستگاه مختصات هستند، آیا مجموعه ای از اونها (حرکت) هم نباید مستقل باشه؟؟


چرا باید بتونه؟ هر سیگنالی که بفرسته با سرعتِ نور حرکت خواهد کرد پس از دیدِ ناظرِ ساکن سیگنال هیچ وقت بهش نمی‌رسه

از دید چارچوبی که با ناظر دوم حرکت میکنه، ناظر دوم ساکنه و نور هم با سرعت نور حرکت میکنه. این یعنی نور باید بتونه بهش برسه،نه؟

یه چیز دیگه! اگه نیرو با قانون دوم نیوتن تعریف میشه پس جرم با چی تعریف میشه؟؟؟ مسلما یه معادله نمی تونه دو تا کمیت رو تعریف کنه!

ایول! ممنون:) بقیشو فهمیدم فک کنم:دی

Ehsan
05-03-2014, 09:23 PM
خب وقتی رویدادها هر کدوم مستقل از دستگاه مختصات هستند، آیا مجموعه ای از اونها (حرکت) هم نباید مستقل باشه؟؟


حرکت مستقله اما نحوه‌ی نمایشِ حرکت در یک دستگاه مختصات به خودِ دستگاه مختصات بستگی داره



از دید چارچوبی که با ناظر دوم حرکت میکنه، ناظر دوم ساکنه و نور هم با سرعت نور حرکت میکنه. این یعنی نور باید بتونه بهش برسه،نه؟

شما تو این مسئله با یک حالت بسیار خاص مواجه هستید، ناظری که نسبت به یک دستگاه با سرعتِ نور حرکت بکنه نسبت به همه‌ی دستگاه‌های لختِ دیگر (تاکید می‌کنم، نسبت به «همه»ی دستگاه‌های لختِ دیگر) با سرعتِ نور حرکت خواهد کرد. نکته اینه که وقتی ناظری با سرعتِ نور حرکت می‌کنه همه‌ی زمانهای از منفی بینهایت تا مثبتِ بی‌نهایتِ ناظرِ ساکن، برای ناظرِ متحرک «یک لحظه» خواهد بود، این وضعیت خیلی عجیبه، یعنی هر میزانِ متناهی گذرِ زمان برای ناظرِ ساکن، از نظرِ ناظرِ متحرک صفره. بنا بر این شاید ناظرِ متحرک ببینه که سیگنال بعد از مدتی بهش برسه اما این زمانِ متنهای از نظرِ ناظرِ ساکن بی‌نهایته و هیچ وقت این سیگنال بهش نخواهد رسید.

شاید بگید این نقضِ تقارنه، اما نیست، این حالت خیلی خاصه، هیچ ناظری تا الان دیده‌نشده که با سرعتِ نور حرکت بکنه.


یه چیز دیگه! اگه نیرو با قانون دوم نیوتن تعریف میشه پس جرم با چی تعریف میشه؟؟؟ مسلما یه معادله نمی تونه دو تا کمیت رو تعریف کنه!

این دور در قوانینِ نیوتون وجود داره، یعنی شما یا باید جرم رو بپذیرید و نیرو رو تعریف کنید یا باید نیرو رو بپذیرید و جرم رو از روی اون تعریف کنید (البته صحیح‌ترش به جای نیرو، تکانه هستش) تعریفِ جرم یکی از بزرگترین چالشهای فیزیکدانها بوده، گرچه نیوتون از این تعریف چشمپوشی کرد (در قبالِ اون به نتایجِ بسیار موفقیت‌آمیزی هم رسید) اما واقعا این دوری که گفتید تو قوانینِ نیوتون هست :)

Ehsan
08-05-2014, 10:12 PM
قرار بود به انقباضِ لورنتس بپردازیم، ایده‌ای که خانم قلمبر مطرح کردند رو پی می‌گیریم:

یه بارِ نقطه‌ای تصور کنید، اگر این بار ساکن باشه میدانِ الکتریکی اطرافش به طورِ متقارن حولش پخش شده، اگر از اثرِ فاصله صرفِ نظر کنیم، بردارهای میدانِ الکتریکی یک کره تشکیل می‌دن:



http://up.avastarco.com/images/elbor9jiwu4svvdhpw1.jpg

آما بیاید همین کار رو با میدانِ الکتریکی یک ذره ی متحرک انجام بدیم، یعنی نوک بردارهای میدان الکتریکی رو به هم وصل کنیم تا ببینیم چه شکلی به دست میاد، برای حرکت با یک دوم سرعتِ نور این شکل به دست میاد:


http://up.avastarco.com/images/dvnu0b65qsautbrzsouy.jpg

می‌بینید؟ شکل فشرده تر شده! این یک ایده به ما میده که بفهمیم ممکنه در حین حرکت طولها منقبض بشن، مخصوصا این ایده وقتی معنا پیدا می‌کنه که بدونیم معادلاتِ ماکسول تحتِ حرکت ناوردا هستند، این یعنی چی؟

بیاید ظریفتر به این ایده نگاه کنیم: باری که از دیدِ ما متحرکه، از دیدِ دستگاهِ خودش ساکنه، و در اون دستگاه یک میدانِ متقارن و کروی داره، اما گویا با حرکت میدانها در راستای حرکت ضعیفتر میشن، و کره هم در راستای حرکت فشرده تر. اما این هنوز هیچ چیزِ دقیقی راجع به میدانهای الکتریکی نمی‌گه چون ما هنوز راجع به ارتباطِ میدانهای الکتریکی و «طول» چیزی نمی‌دونیم بنا بر این نمی‌تونیم راجع به انقباضِ لورنتس هم قضاوتی داشته باشیم.

ما به روشی مستقیمتر برای مشاهده‌ی این پدیده نیاز داریم، بعد از بیانِ این روشِ مستقیم برمی‌گردیم به تدقیقِ ایده‌ی خانم قلمبر که اتفاقا حرفهای بسیار جالبی برای گفتن داره :)

Ehsan
08-12-2014, 11:31 PM
یادتون هست تو این پست (http://forum.avastarco.com/forum/showthread.php?1443-%D9%81%D9%8A%D8%B2%D9%8A%D9%83-%D8%AC%D8%AF%D9%8A%D8%AF&p=70770&viewfull=1#post70770) (و پستهای بعدیش) راجع به فرقِ دیدن و اندازه گرفتن حرف زدیم؟

و این که برای اندازه گرفتنِ طولِ یک جسم باید سر و تهِ جسم رو همزمان اندازه بگیریم وگرنه اندازه‌گیری ما نتیجه‌ی مهملی خواهد داشت. از طرفی یادمون هست که مفهومِ همزمانی در نسبیتِ خاص کاملا متفاوت هستش، پس هر مفهومِ دیگری مرتبط با همزمانی باید دوباره بررسی بشه تا ببینیم آیا نسبت به حالتِ کلاسیک تفاوتی رخ داده یا خیر، و الان می‌خواهیم همین کار رو بکنیم اون هم راجع به اندازه‌گیری طولِ اجسام:

تکنیکِ ما باز هم به ساعت مربوطه، ساعتهای آینه‌ای (http://forum.avastarco.com/forum/showthread.php?1443-%D9%81%D9%8A%D8%B2%D9%8A%D9%83-%D8%AC%D8%AF%D9%8A%D8%AF&p=71607&viewfull=1#post71607) رو که ساخته بودیم یادتون هست؟ دو آینه‌ی رو به روی هم که راستای قرار گیری‌شون عمود بر مسیرِ حرکت هستش، به کمکِ این ساعتها ما تونستیم انبساطِ زمان رو بسنجیم و تحقیق کنیم، حالا بیاید این ساعتها رو بچرخونیم، طوری که راستای بین دو آینه موازی راستای حرکت باشه، حالا چه اتفاقی می‌افته؟


http://up.avastarco.com/images/xfnrprww3bb51m6gd70.png

فرض کنید فاصله‌ی بین دو آینه از دیدِ ناظرِ ساکن l و از دیدِ ناظرِ متحرک'l باشه، هنوز نمی‌دونیم رابطه‌ی بینِ l و'l چیه، رفت و برگشتِ نور رو همچنان تیک تاک در نظر بگیرید، حالا فاصله‌ی زمانی بین یک رفت و برگشتِ نور رو در دو دستگاه محاسبه می‌کنیم با فرضِ این که سرعتِ نور در هر دستگاه ثابت و c هستش.

به وضوح زمانِ تیک تاک در دستگاهِ ساکن t=2l/c خواهد بود، اما به خاطرِ تفاوت سرعتِ نور نسبت به آینه‌های متحرک، زمانِ رفت و برگشتِ نور بین دو آینه‌ی متحرک کاملا متفاوت خواهد بود و با این فرمول حساب می‌شه


http://up.avastarco.com/images/0oirfe9qpdvw98bekw7w.jpg

که جمله‌ی شاملِ c+v مالِ وقتی هستش که نور و ساعت در خلافِ هم حرکت می‌کنند و c-v هم وقتی که ساعت و نور به هم‌جهت حرکت می‌کنند. خوب، یک سوال، آیا با چرخوندنِ آینه‌ها، چیزی از ساعت بودنِ این ساختار کم می‌شه؟ یعنی این که من می‌تونم همچنان به رفت و برگشتِ نور در آینه به عنوانِ تیک تاک نگاه کنم؟ بله (جزئیاتش رو خودتون استدلال کنید، راهنمایی هم این که دو ساعتِ عمود بر هم رو کنارِ هم تصور کنید که با هم همزمان کار می‌کنند و در مکانِ یکی از آینه ها مشترک هستند)
خوب حالا که این هم ساعت هستش و تیک تاکش هم باید از اون فرمولِ اتساعِ زمانی که به دست آورده بودیم تبعیت کنه، پس دارم:


http://up.avastarco.com/images/vh1glndl28kgvqx0b0ca.png

که این یعنی طولِ متحرک باید از طولِ ثابت کوتاه‌تر باشه، این همون فرمولِ انقباضِ لورنتس هستش، یعنی طولِ های متحرک کوتاه‌تر می‌شوند (نه این که کوتاه تر به نظر می‌رسند، بلکه طبقِ تعریفِ اندازه‌گیری دقیقا کوتاه‌تر می‌شوند)

باز ما با یک پدیده‌ی عجیب رو به رو هستیم، توضیحاتش بمونه برای بعد، تازه بعدش باید روشِ خانم قلمبر رو هم بسط بدیم و ببینیم اون چه طور می‌تونه این نتیجه رو تایید کنه، بعدش پارادکس زیبای نردبان و دستِ آخر هم پدیده‌ی خارق‌العاده زیبای «چرخشِ ظاهری»

Ehsan
09-07-2014, 10:30 PM
خوب قبل از این که واردِ توضیحِ این پدیده و نتایجِ بسیار عجیب و زیباش برسیم، می‌خوام به تدقیقِ ایده‌ی خانمِ قلمبر بپردازم، این ایده حرفهای جالبی برای گفتن داره:

ایده‌ی خانمِ قلمبر از اینجا شروع می‌شه که چون ما می‌دونیم معادلاتِ ماکسول تحتِ حرکت ناوردا هستند، (یعنی قوانینِ ماکسول هم در دستگاهِ ساکن صادق هستند و هم در دستگاهِ متحرک) بنا بر این بدونِ این که از نسبیت اطلاع داشته باشیم و تصور کنیم که قوانینِ حرکت کلاسیک هستند و قوانینِ ماکسول در هر دستگاهی صادق اند می‌تونم با مقایسه‌ی میدانِ ایجاد شده به وسیله‌ی بارهای ساکن و متحرک، به نوعی انقباضِ فضا رو در یک جهت پیدا کنیم. اما چه‌طور می‌تونیم از این که خطوطِ میدان تغییر شکل می‌دن و تقارنِ کروی‌شون به هم می‌خوره راجع به انقباضِ لورنتس نتیجه‌ی دقیق بگیرم؟ باید به نحوی خطوطِ میدان رو به طول مرتبط کرد.

یک راه اینه که پوسته‌ای کروی رو حولِ یک بارِ الکتریکی ساکن تصور کنیم که به همه جای پوسته شتابِ یکسانی وارد می‌شه، و بعد نگاه کنیم به نیرویی که به بارها در حالتِ متحرک با میدانِ دِفُرمه وارد می‌شه و با مقایسه‌ی این دو حالت بتونیم نتیجه بگیریم که فضا منقبض شده. اما این ایده پیچیدگی‌های محاسباتی وحشتناکی داره که که کلا بی‌خیالش میشم.*

اما یک راهِ بهتر که توسطِ آقای حسین براتی ارائه شده اینه که با سه تا بارِ الکتریکی، نقطه‌ای بسازیم که برایندِ میدانِ الکترکی در اون نقطه صفر باشه، این طوری:

http://up.avastarco.com/images/laxqneepbtduwa5hdkxu.jpg

در نقطه‌ی b که خیلی نزدیکِ به بارِ مثبت هستش، میدان مثبت و به سمتِ بیرونه، در نقطه‌ی a که خیلی دوره، اثرِ دو بارِ منفی می‌چربه و انگار یک بارِ منفی در مرکز قرار داره و میدان منفی و به سمتِ بیرونه، طبقِ قضیه‌ی رول (همون قضیه که میگه خطِ پیوسته سوراخ نداره! :دی) یک نقطه‌ای مثلِ c وجود داره که میدان اونجا صفره. پس هر بارِ الکتریکی بگذارم از اونجا تکون نمی‌خوره.

حالا فرض کنید باری کوچک در نقطه‌ی c قرار بدم و به دستگاهی چون s’ برم که این بارها با سرعتِ v به سمتِ راست حرکت می‌کنند، میشه با قوانینِ الکترومغناطیس میدانِ الکتریکی بارهای متحرک رو حساب کرد (همون فرمولِ خانمِ قلمبر) و بعد با اعمالِ اون فرمول روی این حالت (بارهای متحرک) خواهیم دید که نقطه‌ی میدانِ صفر جا به جا میشه و با ضریبِ

http://upload.wikimedia.org/math/7/4/a/74a74e4bf40e9fe7240f8d1437ba5c95.png

به مبدا نزدیک می‌شه.

یعنی چی؟ در حالتی که بارها ساکن هستند (چارچوبِ s) ، اگر بارِ کوچکی در نقطه‌ی c قرار بدم، تکان نمی‌خوره، حالا اگر همه‌ی بارها با سرعتِ v به سمتِ راست حرکت کنند (چارچوبِ s’) ، قوانینِ الکترومغناطیس به ما میگن در دستگاهِ مختصاتِ متحرک، خواهیم دید اگر فاصله‌ی بارِ کوچک از بارِ مثبتِ مرکزی همان فاصله‌ی قبلی باشه، ذره دستخوشِ شتاب میشه و اگر دستخوشِ شتاب بشه یعنی در دستگاهِ s ساکن نیست (که می‌دونیم هست) اما اگر ذره کمی نزدیکتر باشه (به اندازه‌ی ضریب لورنتس) دستخوشِ شتاب نمیشه و با همان سرعتِ v به حرکتش ادامه میده انگار که در دستگاهِ s ساکنه!

به یک معنی خطکشهای الکترومغناطیسی بدونِ این که ما چیزی از قوانینِ نسبیت بدونیم، دارن اثراتِ نسبیتی رو به ما نشان میدن. جالب نیست؟



* اون تیکه‌ی بیخیال میشیم دو هفته وقتِ منو گرفت تا بفهمم باید بیخیالش بشیم! :|

dideh bane javan
09-24-2014, 09:36 PM
سلام دوستان من دانشجوی برقم به فیزیک هم علاقه دارم میتونین یه کتاب فیزیک برام معرفی بکنین که ساده باشه(مهم نیس چن جلد باشه) کتابی که از کلاسیک تا مدرن رو شامل بشه؟سوال بعدیم این بود که کتاب فیزیک عمومی فریمن داریم؟

narcissus flower
09-25-2014, 04:36 PM
سلام دوستان من دانشجوی برقم به فیزیک هم علاقه دارم میتونین یه کتاب فیزیک برام معرفی بکنین که ساده باشه(مهم نیس چن جلد باشه) کتابی که از کلاسیک تا مدرن رو شامل بشه؟سوال بعدیم این بود که کتاب فیزیک عمومی فریمن داریم؟
سلام به جمع ما خوش امدید
والا کتاب که زیاده اما از دید اکادمیک مجموعه کتب هالیدی هست که اگر بتونید با ادبیاتش کنار بیایید کتاب های سختی نیستند و تقریبا جامع فصول فیزیک پایه رو پوشش میده .که فکر کنم دو جلدش رو شما تو دانشگاه میخونید.
اما پیشنهاد من اینه که اول از همه تو سایت های معتبر در باره ی فیزیک گردشی بکنید تا کمی سر ذوق بیایید و کلی کیف کنید.:)
و اینکه کتاب نمایش شگفت انگیز فیزیک برای انتشارات اراکس رو نگاهی بیاندازید ،کتابی هست که با ازمایش های جالب قوانین فیزیک رو بیان میکنه.
بعد از اینکه کمی پایه تون قوی شد از اون به بعد باید برحسب علاقه تون کتاب انتخاب کنید مثلا اگر دوست دارید مکانیک کوانتوم رو به زبان ساده بخونید پیشنهاد من زتیلی هست.چون اموزش روان و تمرین های با پاسخ داره.
اگر هم مثلا نسبیت دوست دارید کتاب نسبیت خاص و عام و کیهان شناختی ولفگانگ ریندلر کتاب بدی نیست ،
و یا فیزیک نوین وایدنر وسلز .این ها اکادمیک هستند و نیازمند دقت. و حوصله
اما
اگر برای اوقات فراغت میخواید
الان داریم به صورت گروهی کتاب پس از نخستین سه دقیقه رو میخونیم که یک کتاب کیهان شناسی به زبان ساده هست و کتابی مثل کیهان در پوست گردو ویا طرح بزرگ استفان هاوکیینگ هست که میتونه یه دید مطلوبی در مورد دانش های روی میز امروز کسب کنید.
البته در همین فروم خودمون هم گشتی بزنید کلی مطلب خوندنی پیدا میکنید بهتون قول میدم:)

Ho33ein.kh
09-25-2014, 10:14 PM
سلام
خب ، من به صحبت های دوست عزیزمون خانم قلمبر (اگه اشتباه نکنم) اضافه میکنم:
با توجه به اینکه شما دانشجوی مهندسی بودین ، و ریاضیات مهندسی رو مطالعه کردین کمتر به مشکل بر می خورید. اما باز هم با یه سری مباحث مثل تانسور ها و عملگر های ماتریسی آشنایی ندارید. در صورتی که به مشکلی برخوردید باید به کتاب "روش های ریاضی در فیزیک" یا به قول ما فیزیکی ها ریاضی-فیزیک مراجعه کنید.

بعد از خودن فیریک پایه ( سه جلد فیزیک هالیدی) برای ادامه :
توی حوزه مکانیک : کتاب مکانیک تحلیلی نوشته فولز ( انتشارات نوپردازان خوبه و روان - البته یه کتاب قدیمی نشر دانشگاهی هم هست یه سری مباحث توش داره که توی نسخه های جدید حذف شده) رو بخونید خیلی جالبه
می تونید در ادامه اش هم برید کتاب های مکانیک دیگه توی حوزه استاتیک یا دینامیک رو بخونید. (البته می دونم دنبالش نیستید. ولی مکانیک تحلیلی یکی از جالب ترین درس های بچه های فیزیکه )

توی حوزه نورشناسی (اپتیک) : کتاب های اپتیکی زیاد هستن ، مباحث مربوط به اپتیک هندسی و موجی و همچنین اپتیک خطی و غیر خطی و... زیاده برای شروع توی اپتیک باید الکتر مغناطیس بلد باشید که توی رشته برق فکر کنم یه چند واحدی پاس میکنیئ.
تی این حوزه (اپتیک) کتاب "رهیافت حل مسائله در نورشناخت نوین" کتاب خوبیه

توی حوزه مکانیک کوانتمی همون کتاب زیتیلی مناسبه البته پیشنهادم ترجمه دکتر محمد صبائیان نشر نواندیشه

توی رمینه ترمو دینامیک هم سراغ هر کتابی میرید پیشنهاد میکنم سراغ زیمانسکی نرید که اصلا جالب نیست

به هر حال توی هر شتخه ای کتب مختلف هست ، البته برای بیشتر گرایش ها تاب ترجمه شده نیست مثلا توی زمینه اتمی مولکولی (اسپکتروسکپی ، لیزر ، شتاب دهنده ها ، آشکار ساز ها و ...)